Sunteți pe pagina 1din 96

ISSN 2601-0305

ISSN-L 2601-0305

REVISTA DE MATEMATICĂ
MARINESCU-GHEMECI OCTAVIAN
PUBLICAT, IE ANUALĂ
PENTRU ELEVI S, I PROFESORI

Liceul ,,S, tefan Diaconescu” Potcoava

Anul III, nr. 1/2019

Editura Hoffman
Editori:
Costel BĂLCĂU (redactor s, ef) Costel ANGHEL (redactor)
Florea BADEA (redactor) Mihai Florea DUMITRESCU
(secretar general de redact, ie)
Comitetul s, tiint, ific:
Membri de onoare:
Prof. univ. dr. Victor ALEXANDRU - Universitatea din Bucures, ti
Prof. univ. dr. Tudor BĂLĂNESCU - Universitatea din Pites, ti
Prof. univ. dr. Corneliu UDREA - Universitatea din Pites, ti
Membri:
Stelian-Corneliu ANDRONESCU - Universitatea din Pites, ti
Costel BĂLCĂU - Universitatea din Pites, ti
Cezar JOIT, A - I.M.A.R.
Ruxandra MARINESCU-GHEMECI - Universitatea din Bucures, ti
Radu MICULESCU - Universitatea Transilvania din Bras, ov
Cristinel MORTICI - Universitatea Valahia din Târgovis, te
Cristian NICULESCU - Universitatea din Bucures, ti
Comitetul de redact, ie:
Leonard GIUGIUC Marian HAIDUCU Marin IONESCU
Daniel JINGA Marius MÂINEA Marius PERIANU
Florin STĂNESCU Adrian T, URCANU Sorin ULMEANU

Colaboratori:
Delia Ileana BASCH-NAIDIN Eduard BUZDUGAN Luigi Ionut, CATANA
Aurel CHIRIT , Ă Dumitru ILIE Lavinia DUMITRANA
Iuliana ION-IONESCU Oana KRISZTA Cosmin MANEA
Ileana MARINESCU-GHEMECI Iulia MARINESCU-GHEMECI George MIHAI
Adriana MIU Constantin MOGOS , ANU Florin NĂSUI
Mariana NĂSUI Grat, iela POPA Dorin POPA
Sorin PELIGRAD Dragos, PETRICĂ Florina PETRUS , CĂ
Valentin RĂDULESCU Florin Adrian RIS
, CĂ Cristina SMARANDACHE
Valentin SMARANDACHE Manuela STROE Daniela TACLIT
Marian TELER Nicolae TOMESCU Iuliana TRAS , CĂ

Tehnoredactare computerizată: Mihail TĂNASE, e-mail: mihaimit@yahoo.it

Redact, ia: Liceul ,,S, tefan Diaconescu”, Str. Principală, nr. 197, cod 237355,
Potcoava, tel. 0742123058, e-mail: florin14mihai@yahoo.com
Revista este editată ı̂n colaborare cu Departamentul de Matematică-Informatică,
Universitatea din Pites, ti.
Forma digitală a revistei poate fi accesată la adresa: http://rmgo.upit.ro

Publicată de: Editura Hoffman, www.EdituraHoffman.com, www.LibrariaHoffman.ro


Anul III, Nr. 1, 2019
Cuprins 3

Cuprins

COLT, IS, ORUL CU AMINTIRI 5

Dumitru LAZĂR
Cum am ajuns profesor de franceză la Liceul Potcoava. Odiseea ı̂nceputurilor! 5

ARTICOLE S, I NOTE MATEMATICE 15

Thanos KALOGERAKIS
Geometric constructions, with the compass alone, the straightedge alone or
neither of them . . . . . . . . . . . . . . . . . . . . . . . . . . . . . . . . . . 15

Florin STĂNESCU
Utilitatea unei formule ı̂n rezolvarea unor probleme de calcul matriceal . . . 23

Leonard GIUGIUC and Bogdan SUCEAVĂ


On a Geometric Theorem . . . . . . . . . . . . . . . . . . . . . . . . . . . . 27

Victor ALEXANDRU s, i Stelian Corneliu ANDRONESCU


Lema lui Hensel. Aplicat, ii . . . . . . . . . . . . . . . . . . . . . . . . . . . . 31

CONCURSUL DE MATEMATICĂ ,,MARINESCU-GHEMECI OCTAVIAN”39

Florea BADEA
Prezentarea Concursului Interjudet, ean de Matematică ,,MARINESCU–GHEMECI
OCTAVIAN”, Edit, ia a VII-a, Potcoava, 5 mai 2018 . . . . . . . . . . . . . 39

Costel ANGHEL
Prezentarea Concursului Judet, ean de Matematică ,,MARINESCU–GHEMECI
OCTAVIAN”, Edit, ia a VIII-a, Potcoava, 11 mai 2019 . . . . . . . . . . . . 47

TESTE PENTRU EXAMENE 55

Costel ANGHEL s, i Florea BADEA


Teste pentru examenul de Evaluare Nat, ională . . . . . . . . . . . . . . . . . 55
4 Cuprins

Mihai Florea DUMITRESCU


Teste pentru examenul de Bacalaureat, specializarea s, tiint, e ale naturii . . . 59

Costel BĂLCĂU
Teste pentru examenul de Bacalaureat, specializarea matematică-informatică 63

PROBLEME PENTRU CONCURSURI 68

Rezolvarea problemelor din numărul anterior 68

Probleme propuse 89
COLT, IS, ORUL CU AMINTIRI

Cum am ajuns profesor de franceză la Liceul


Potcoava. Odiseea ı̂nceputurilor!

Dumitru LAZĂR 1

Am absolvit Facultatea de Filologie sect, ia Limba s, i literatura română - secundar,


Limba s, i literatura franceză, ı̂n cadrul Universităt, ii ,,Babes, Bolyai” din Cluj-
Napoca, ı̂n anul 1972, fiind ultima promot, ie cu 5 ani de studiu (după cum fusesem
s, i ultima promot, ie cu 7 clase ,,elementare” s, i 11 clase de liceu). Repartit, iile, ı̂n acel
an, au fost făcute pe centre universitare: posturile oferite erau, ı̂n general, pentru
zona Ardealului. As, adar, acestea nu erau avantajoase pentru cei care veneam
din ,,vechiul regat” (cum ne spuneau ardelenii). De altfel, ı̂n general, posturile
disponibile nu erau prea atractive, ı̂ntrucât conducerea facultăt, ii nu mersese ı̂n
timp optim la Ministerul Învăt, ământului ca să aibă posibilitatea să le aleagă pe
cele favorabile. Eu, ı̂mpreună cu alt, i patru colegi, cu medii bune de absolvire, am
refuzat oferta Comisiei de repartizare a Ministerului, motivând că, având părint, ii
bolnavi sau cu diverse probleme, dorim catedre mai aproape de locuint, a acestora.
Ment, ionez că cei cu medii de repartit, ie peste nouă aveau dreptul să ia post s, i ı̂n
oras, e (ı̂n limita numărului de catedre scoase la repartit, ie). Am avut posibilitatea
să aleg post, la gimnaziu, ı̂n trei oras, e: Res, it, a, Petros, ani, Hunedoara, toate foarte
poluate ı̂n vremea aceea. Având ı̂n vedere că sot, ia mea Valeria Minodora Lazăr
(care a terminat Facultatea de Filologie ı̂n acelas, i an - fiind prima promot, ie cu
patru ani de studiu - dar primea repartit, ia cu o lună ı̂n urma mea) avea, ı̂n acea
vreme, unele probleme de sănătate, am refuzat - ı̂mpreună cu cei patru colegi - să
luăm post ı̂n prima zi de repartit, ie, motivând că avem părint, ii cu situat, ii precare
(tatăl meu, bolnav grav, s-a s, i stins din viat, ă la mai put, in de un an, ı̂n martie 1973)
s, i am solicitat să ni se obt, ină de la Minister posturi pentru zona Munteniei s, i/sau
Olteniei. A doua zi, luând act de cazul nostru, distinsul profesor universitar doctor
Dumitru Pop (Dumnezeu să-l ierte!), ı̂nt, elegător, a obt, inut câteva posturi mai
convenabile, la licee, dar pentru specializarea secundară (Limba franceză). Unul
dintre acestea era la Liceul Teoretic Potcoava, judet, ul Olt. Consultând Catalogul
Ministerului Învăt, ământului, cuprinzând posturile oferite pentru repartit, iile din
1972, am constatat că la Liceul Potcoava erau, de fapt, două posturi de Franceză
1
Profesor dr. ı̂n Istorie, pensionar

5
6 Dumitru LAZĂR

libere (unul la liceu, celălalt la gimnaziu). Socotind că pe al doilea post ar putea
veni, peste o lună, sot, ia mea (care avea aceeas, i specialitate), am ales, fires, te, dar
NEINSPIRAT, postul de Franceză la clasele de liceu. Mărturisesc că alegerea nu a
fost ı̂ntâmplătoare, ci, mai degrabă, a fost subiectivă: de Potcoava auzisem ı̂ncă
din copilărie, tatăl meu mergea, ı̂mpreună cu alt, i oameni din sat, la Vale, ı̂n zona
Potcoava, cu fructe, t, uică, ot, et etc., pe care le schimba cu grâu s, i porumb. În
plus, părintele unei colege de facultate, care lucrase ı̂n zonă, mi-a vorbit atât de
frumos despre Potcoava ı̂ncât m-a convins (dar mi-a s, i marcat destinul, spunând
că ,,cine bea apa de Potcoava nu mai pleacă”, ceea ce s-a cam ı̂ntâmplat, ı̂ntrucât
am rămas ı̂n satul, azi oras, ul de pe Plopcea - ı̂ntr-adevăr cu cea mai bună apă
potabilă naturală - 25 de ani, până la 31 august 1997, s, i recunosc că nu mi-a fost
deloc rău. De aceea, am regretat ulterior transferul ı̂n Râmnicu-Vâlcea, fiindcă, ı̂n
Potcoava, ı̂mi trăisem tineret, ea, eram cunoscut s, i apreciat, avusesem performant, e
profesionale deosebite (la care mă voi referi ı̂n numărul următor al revistei), adică
eram, cum se spune, cineva, ı̂n timp ce ı̂n Vâlcea (unde m-am simt, it o bună
perioadă intrus, mai ales ı̂n funct, ia de director al Bibliotecii Judet, ene, domeniu
diferit de cel al ı̂nvăt, ământului) a trebuit să iau viat, a profesională de la capăt. Nu
mi-a fost us, or, probabil s, i pentru că nu mai aveam fort, a s, i elanul tineret, ii, aveam
48 de ani, mi se părea că nici timpul nu mai avea aceeas, i consistent, ă s, i răbdare; de
aceea, a trebuit să sporesc eforturile ca să mă păstrez la ı̂nălt, ime. Un alt argument
pro Potcoava mi l-a oferit mărturisirea marelui nostru profesor clujean de Sintaxa
Limbii române, Dumitru Dras, oveanu (ierte-l Dumnezeu s, i pe dumnealui!), fiu de
ceferist, care mi-a spus că, ı̂n copilărie, mergea cu trenul ı̂nsot, indu-s, i tatăl (asta se
ı̂ntâmpla cu cel put, in 20 de ani ı̂nainte, deci prin anii ’50) s, i admira Potcoava de
sus, de pe dealul Gării, luminată ca un oras, , cu asfalt, cu blocuri s, i case mari etc.
Tentant, nu-i as, a? Mai ales pentru un fost copil trăit la t, ară, ı̂n satul Corbii din
Deal (comuna Nicolae Bălcescu, judet, ul Vâlcea), unde noaptea, s, i atunci, ı̂n anii
’70, ne luminam cu lampa cu gaz.

Am spus mai sus că alegerea mea a fost, totus, i, ,,neinspirată” - asta din mai
multe motive. În primul rând, deoarece, când am descins ı̂n Potcoava, ı̂nainte de
ı̂nceperea cursurilor s, colare, ca să ı̂mi anunt, repartit, ia, am aflat de la directorul
s, colii (profesorul S, tefan Diaconescu - odihnească-l Dumnezeu ı̂n pace! -, al cărui
nume ı̂l poartă astăzi liceul din localitate), că, pe catedra pentru care optasem la
repartit, ie, funct, iona, de mult timp, un profesor al locului, cu experient, ă, din păcate
netitularizat ı̂ncă pe post. La prima ı̂ntâlnire cu Domnul Director (Dumnezeu să
ı̂l ierte s, i fie-i numele lăudat pentru ceea ce a realizat, ı̂n cei 15 ani de directorat,
pentru s, coala pe care a identificat-o cu propriul destin, până ı̂n ultima clipă a viet, ii
sale, părăsind lumea noastră ı̂n timp ce mergea spre s, coala sa, pe care a slujit-o
până la jertfă supremă. Eroică s, i admirabilă moarte!), am constatat, surprins
neplăcut atunci, că nu eram dorit as, a cum, tânăr entuziast, mă as, teptam: când a
aflat că am specialitatea secundară pentru catedra de Franceză la liceul la care
urma să predau, a react, ionat prompt, fără curtoazie s, i menajamente: ,,Domnule, te
fluieră elevii!” Aici predă franceza un cadru cu multă experient, ă, care a fost s, i ghid
Cum am ajuns profesor de franceză la Liceul Potcoava. 7

turistic ı̂n Deltă etc., etc. Fără să mă las intimidat, dar contrariat, replica mea, la
fel de promptă, a fost simplă: ,,Domnule director vă rog să nu facet, i presupuneri
... avant la lettre!. Vet, i avea ocazia s, i timpul necesar să vă pronunt, at, i după ce
mă vet, i asista la clasă”. În momentul când i-am spus că dorim ca sot, ia mea (care
se afla de fat, ă) să ocupe celălalt post de Franceză (la gimnaziu), descumpănit,
mi-a răspuns pe un ton destul de dur: ,,Dar vret, i să ne dat, i afară pe tot, i?”. Însă,
dându-s, i seama că a exagerat, a ı̂ndulcit tonul s, i a spus că postul de Franceză a fost
transformat ı̂n catedra de Limba engleză s, i (fiind informat că postul de Engleză
devenize vacant prin faptul că absolventa repartizată pe el luase negat, ie) că acest
post va trebui transformat din nou pe Franceză, dar numai cu decizia Ministerului,
ceea ce este foarte dificil. Am plecat... eu hotărât să respect repartit, ia (nici
nu aveam altă solut, ie), sot, ia, derutată s, i descurajată, plângând. Am linis, tit-o,
spunându-i că un asemenea om care spune franc ı̂n fat, ă ceea ce simte ı̂ntr-un
moment de provocare, nu poate fi decât un om cinstit, corect, ı̂ntr-un final, bun,
considerând că este de apreciat s, i admirabil faptul că, ı̂n funct, ia sa de director,
t, ine la calitatea ı̂nvăt, ământului din s, coală, precum s, i la cadrele didactice pe care
le reprezintă, ceea ce nu este deloc de neglijat.

Am făcut aceste mărturisiri, care par a pune ı̂ntr-o lumină defavorabilă pe


Domnul Director Diaconescu (departe de intent, ia mea!), pentru a arăta efectul,
ı̂n timp, al atitudinii sale asupra parcursului meu profesional. Ment, ionez că
react, iile Domniei Sale, efect al temperamentului său coleric, nu m-au descurajat,
ba, dimpotrivă, m-au ambit, ionat, făcându-mă să cons, tientizez că trebuie să ı̂mi
iau, ı̂ncă din prima zi de s, coală, meseria de dascăl ı̂n foarte serios. Am recunoscut
ulterior, s, i o spun s, i astăzi, că directorul S, tefan Diaconescu m-a făcut veritabil
profesor de Franceză, zic veritabil, ı̂ntrucât, cunoscându-mi limitele (motiv pentru
care am rugat sot, ia să preia ea ı̂n catedră clasele a XI-a s, i a XII-a Uman, ea s, tiind
foarte bine - mai bine decât mine - limba), generate de faptul că ı̂n facultate nu
m-am gândit niciodată că as, putea ajunge să predau Limba franceză, m-am apucat
să mă pregătesc serios pentru fiecare oră de clasă, cu planuri de lect, ie, scenarii
didactice detaliate, urmând firul consacrat, concret s, i complet al celor 50 de minute
ale lect, iei: la fiecare ı̂ntrebare a profesorului, răspundeam cu mai multe solut, ii
posibile ale elevilor, până la epuizarea subiectului. S, i asta nu pentru că devenisem
subit fan, ca să folosesc un cuvânt la modă astăzi, al Limbii franceze (se va ı̂ntâmpla
s, i asta mai târziu, când am constatat că pot obt, ine rezultate performante), ci ca
să nu ,,mă fluiere elevii”, contrazicând previziunile s, i dovedind conducerii s, colii s, i
mie ı̂nsumi că pot fi profesor de Franceză la liceu, că pot obt, ine rezultate bune
cu elevii, ceea ce pe parcurs s-a ı̂ntâmplat, ajungând să am două eleve (una la
liceul Potcoava, Pandelică Iuliana, ı̂n clasele a X-a, a XI-a s, i a XII-a, s, i cealaltă,
fiica mea, Lazăr Minodora-Laura, ı̂n clasa a XI-a la Liceul Pedagogic) participante
la Olimpiada nat, ională de Franceză, care au obt, inut premii, premii speciale s, i
ment, iuni. Drept răsplată, i-am purtat Domnului Director Diaconescu respect s, i
recunos, tint, ă cât a trăit s, i postum, am spus-o s, i ı̂n cuvântul de ı̂ngropăciune, când
ne-a părăsit, apoi l-am pomenit ı̂n rugăciunile mele printre mort, ii mei.
8 Dumitru LAZĂR

În al doilea rând ,,neinspirată” a fost alegerea, la repartit, ie, a specialităt, ii


secundare, Franceza (la clasele de liceu) s, i din cauză că, atâta timp cât am predat
această disciplină (18 ani), nu am fost validat de Inspectoratul S, colar Olt ca titular
pe post, pe motiv că pe Diploma de licent, ă scrie ,,secundar Limba s, i literatura
franceză” ceea ce, ı̂n opinia autorităt, ilor judet, ene, nu ı̂mi dădea dreptul să predau
la clasele de liceu (ment, ionez că, la promot, iile următoare ale Facultăt, ii de Filologie,
inclusiv aceea a absolvent, ilor cu patru ani de studiu din acel an - 1972, nu s-a
mai făcut distinct, ie ı̂ntre cele două specializări, acestea nemaifiind discriminate ı̂n
,,principală” s, i ,,secundară”). Mai mult de atât, paradoxal, Inspectoratul S, colar,
fără să cer eu, ı̂n repetate rânduri, mi-a trimis adrese prin care eram ı̂ns, tiint, at că mi
s-a aprobat, ,,la cererea mea”, titularizarea pe o catedră de Franceză sau de Română
la anumite s, coli din judet, : Generală Sines, ti (Potcoava), apoi numărul 4 Slatina,
Liceul Teoretic Corabia, ı̂nsă eu, prevalându-mă de repartit, ia guvernamentală s, i,
de felul meu, fiind statornic, am refuzat tacit ofertele s, i nu mi s-a ı̂ntâmplat nimic
(nu neg, totus, i, faptul că această situat, ie ı̂mi genera de fiecare dată când sosea o
adresă de transfer, o stare de nelinis, te, de provizorat, ceea ce m-a determinat ca
ı̂n al optsprezecelea an de ı̂nvăt, ământ să mă titularizez pe catedra de Română,
devenită vacantă la aceeas, i s, coală, oportunitate de care nu am beneficiat prea
mult, deoarece, ı̂ncepând cu februarie 1990 până ı̂n 1997 (31 august) am lucrat la
Inspectoratul S, colar Olt, ı̂n calitate de inspector la specialitatea Limba s, i literatura
română s, i, tot ı̂n acest ultim an, m-am transferat, prin concurs, la Râmnicu-Vâlcea.

O altă cauză a lipsei mele de inspirat, ie privind repartit, ia la Potcoava, a fost


faptul că nu am prevăzut că al doilea post de Franceză (pe care ı̂l vizam pentru
repartit, ia sot, iei) se va transforma ı̂n Engleză, post care, din fericire, a redevenit
vacant s, i care trebuia schimbat din nou ı̂n Franceză. A necesitat să facem, eu s, i
sot, ia mea, multe demersuri (s, i drumuri costisitoare, ı̂n condit, iile ı̂n care părint, ii
nos, tri, t, ărani, nu aveau posibilităt, i să ne ajute) la Inspectoratul S, colar pentru a
obt, ine acceptul s, i, apoi, adresă către Minister, prin care să se solicite revenirea la
postul de Franceză. În cele din urmă, am obt, inut adresa, grat, ie inspectorului de
la cadre, Domnul Merer, de care ne amintim ı̂ntotdeauna ca fiind binefăcătorul
nostru. Din fericire, la Minister am rezolvat totul ı̂ntr-o singură zi, nu fără mici
peripet, ii create de inspectorul general care răspundea de judet, ul Olt, un anume
domn birocrat, grăbit să plece acasă ı̂ntrucât ziua de lucru era pe sfârs, ite, care
ne-a expediat sfătuindu-ne să revenim ı̂n ziua de mart, i a săptămânii viitoare.
Noroc cu un mare domn de bine din acelas, i birou, care ne-a sfătuit să mergem
la directorul general, Domnul Bondrea, care ne-a aprobat, ı̂n ultimele minute ale
zilei, transformarea postului din Engleză ı̂n Franceză, ı̂ndrumându-ne să mergem
rapid la Domnul Inspector general de la personal, Floricel, care, aflând că sot, ia
avea media de absolvire 9,14, i-a oferit satisfact, ia să-s, i scrie singură noua repartit, ie
(prima, la terminarea facultăt, ii, fusese la Liceul Teoretic Osica, tot ı̂n judet, ul Olt,
după ce renunt, ase la un post de Franceză, cu mult mai bun, ı̂n Târgu-Mures, , ca
să vină, prin schimb de repartit, ie cu o colegă din promot, ia mea, ı̂n judet, ul ı̂n
care luasem eu repartit, ia cu o lună ı̂nainte). Dumnezeu să-i aibă ı̂n grijă, ı̂n viat, a
Cum am ajuns profesor de franceză la Liceul Potcoava. 9

asta sau ı̂n cea de dincolo, pe tot, i binefăcătorii nos, tri, fiindcă au ı̂nt, eles s, i ne-au
ajutat ı̂ncă de la ı̂nceput de drum ı̂n carieră, permit, ându-ne, la ı̂nceput de an
s, colar (1972-1973), să fim prezent, i ı̂n s, coală s, i să ı̂ncepem profesia cu dreptul. În
treacăt fie spus, oportunitatea alegerii posturilor de la Potcoava, al meu s, i a sot, iei,
m-a obsedat mult timp, ı̂ntrucât, fără voie am afectat catedrele domnilor profesori
Marin s, i Olimpia Bălcău (sot, s, i sot, ie). Însă, mai târziu, m-am eliberat, de fapt,
Doamna Profesoară Olimpia Bălcău m-a absolvit de ,,vină”, asigurându-mă că
nu ne-au purtat niciodată pică (se ı̂ntâmpla după plecarea mea din Potcoava, la
cât, iva ani, ı̂n ziua ı̂n care potcovenii conduceau pe ultimul drum pe distinsa s, i
buna Doamnă Profesoară Olimpia Diaconescu). Când i-am cerut iertare, Doamna
Olimpia Bălcău a react, ionat ca s, i cum nimic din ceea ce mă apăsa sufletes, te nu
s-ar fi ı̂ntâmplat. Mult, umesc familiei Bălcău pentru generozitate!
Încă din prima zi de s, coală, mi s-a inclus ı̂n catedră Dirigent, ia la clasa a XII-a
Real, o provocare pentru mine, care aveam cu patru ani mai mult decât elevii mei.
În prima lect, ie de Dirigent, ie, pentru cunoas, terea acestora, dar s, i pentru a testa
opt, iunile s, i preferint, ele lor, ca să ı̂nt, eleg cum ar trebui să fie un profesor s, i un
diriginte, ı̂n eventualitatea de a mă adapta la cerint, ele clasei, am supus elevii la
un chestionar scris. Una din ı̂ntrebări le cerea să precizeze care a fost/care este cel
mai bun s, i cel mai agreat a) profesor; b) diriginte pe care l-au avut ı̂n gimnaziu sau
ı̂n liceu s, i care le-a devenit model pentru viat, ă. Unul dintre cei mai buni elevi din
clasă (la matematică), Ioncea Mircea (care absolvise gimnaziul la S, coala Generală
Mărgineni-Scornices, ti, a răspuns fără echivoc că ..cel mai bun profesor s, i diriginte,
unic pentru mine, a fost Marinescu Octavian, de Matematică, pentru că ne era s, i
profesor ideal s, i diriginte ı̂nt, elegător, bun educator s, i prieten ı̂n s, coală s, i ı̂n afara
s, colii, pe terenul de sport”. Răspunsul elevului mi-a suscitat curiozitatea de a-l
cunoas, te pe profesorul Marinescu (s, ansa a venit repede, fiindcă ı̂n 1975 avea să
vină prin transfer la Liceul Potcoava), despre care voi mai vorbi.
Precizez că la ı̂nceputurile carierei mele de dascăl am fost antrenat s, i ,,călit”
ı̂ntr-o activitate s, colară s, i extras, colară nonstop (s, i as, a am rămas până la pensionare,
ı̂n anul 2014): efectuam zilnic cinci sau s, ase ore la clasă, ı̂n 6 zile lucrătoare (făceam
s, i consultat, ii s, i meditat, ii dimineat, a, ı̂nainte de ı̂nceperea cursurilor, ı̂ntre orele 7 s, i
8, fie pentru activitatea de performant, ă - pregătire pentru concursurile s, colare - fie
pentru recuperarea elevilor cu ,,ritm mai lent” de ı̂nvăt, are), având, ı̂n primii ani, ı̂n
catedră, 24 de ore (25 cu Dirigent, ia) de Franceză la clasă, dintre care 20 obligat, ie
de normă s, i patru suplimentare fără plată, ı̂ntrucât cele două grupe de ı̂ncepători
de la clasele a XI-a s, i a XII-a Uman (cu 5, respectiv 4 elevi) nu ı̂ntruneau condit, iile
finant, abile de minim 15 elevi de grupă. Trebuie să spun, ı̂nsă, că cea mai mare parte
din săptămâna de lucru o consacram activităt, ii cu tineretul, deoarece, ı̂ncă din
prima zi de s, coală (1 septembrie 1972), Domnul Director mi-a pus ı̂n brat, e sarcina
de ı̂ndrumător al organizat, iei Uniunii Tineretului Comunist din s, coală - funct, ie care
presupunea, ı̂n vremea aceea, să activezi aproape toată ziua ı̂n mijlocul tinerilor,
efectuând variate activităt, i educative, cultural-artistice (antrenarea elevilor ı̂n
concursuri cu diverse format, ii artistice, ,,brigăzi de agitat, ie”, teatru, montaje
10 Dumitru LAZĂR

literar-artistice, dansuri, recitări, cântece etc.), concursuri sportive, activităt, i


patriotice nenumărate (colectarea de plante medicinale, fructe de pădure s, i, mai
ales, de materiale refolosibile: maculatură, fier vechi, des, euri textile etc.) care se
efectuau zilnic, inclusiv (sau mai ales) duminica (ı̂n această zi, uneori, se efectueau
s, i plantat, ii forestiere s, i de pomi fructiferi), s, edint, e de organizat, ie cu nemiluita
(pe clase sau la nivel de s, coală), tematice sau de primire ı̂n U.T.C., pregătire
militară (eram s, i comandant de Centru de P.T.A.P. - Pregătirea Tineretului pentru
Apărarea Patriei - ı̂n cadrul Liceului), activităt, i distractive (reuniuni ,,tovarăs, es, ti”,
cel put, in o dată pe săptămână - sâmbăta sau ı̂n ,,Joia Tineretului”, după cursuri)
etc. Seara s, i o bună parte din noapte, ı̂nvăt, am s, colăres, te la Franceză, sarcină
dificilă, la ı̂nceput, pentru că trebuia să mă pregătesc ore ı̂n s, ir, ı̂ntocmind s, i
planurile de lect, ii - rămânându-mi numai patru, maxim cinci ore de somn - dar s, i
frumoasă, deoarece, la clasele a XI-a s, i a XII-a, se studia Istoria literaturii franceze
de la origini (,,Jurământul din Strasbourg”, anul 842) până ı̂n secolul al XX-lea
inclusiv. Era o plăcere să extinzi s, i să raportezi literatura franceză la literatura
universală, ı̂n cadrul căreia includeam s, i literatura română. Pe lângă aceste
multiple s, i diverse activităt, i la nivel de s, coală, noi, dascălii, trebuia să onorăm (ı̂n
fapt, să asigurăm) s, i activităt, ile culturale s, i educative din comună, la Căminul
cultural (unde jucam teatru, cântam ı̂n cor, ı̂n grupuri vocale, brigăzi artistice ,,de
agitat, ie”, ı̂n montaje literar-artistice s, i altele, format, ii participante la Festivalul
Nat, ional ,,Cântarea României” (ca s, i format, iile cultural-artistice din cadrul s, colii),
să desfăs, urăm ı̂nvăt, ământul politico-ideologic al diferitelor organizat, ii de U.T.C.
sau de partid, din cadrul Cooperativelor Agricole de Product, ie Sines, ti s, i Potcoava,
Cooperat, iei, S.M.A.-ului, Gării etc.

Am prezentat mai sus, activitatea mea s, colară s, i extras, colară (care, ı̂nsă, nu
era doar a mea, ci a majorităt, ii dascălilor de tradit, ie haretiană din t, ară) nu pentru
a mă ridica ı̂n slăvi, nici pentru a mă victimiza, ci pentru a arăta tinerilor (sau
mai put, in tinerilor) dascăli de astăzi, din ı̂nvăt, ământul preuniversitar, care era
programul unui cadru didactic (adevărat dascăl-apostol ı̂n tradit, ia s, colii românes, ti,
mos, tenită de la ı̂naintas, i, ı̂ndeosebi de la Spiru Haret) ı̂nainte de 1989 (anii
’70-’80), cum ı̂s, i exercita profesia s, i ı̂n câte activităt, i era angajat s, i se implica
direct (am omis să includ, aici, s, i campaniile agricole de primăvară, de vară s, i de
toamnă efectuate la C.A.P. Sines, ti s, i Potcoava, I.A.S. Corbu, la Baza de Recept, ie
Potcoava etc.), activităt, i utile pentru comunitate, care aveau rolul lor benefic (ı̂n
educarea, cres, terea şi formarea copiilor s, i tinerilor, prin muncă, pentru muncă),
ı̂n schimbul unui salariu mult mai mic decât al dascălilor de astăzi (bunăoară, ı̂n
primi ani de ı̂nvăt, ământ aveam 1050 lei, impozabili, pe lună, ceea ce ı̂nsemna
cu mult sub valoarea miei de lei de astăzi). Însă, ı̂n vremea aceea, nu partea
materială ne motiva (s, tiam de la ı̂nceput că remunerat, ia ı̂n ı̂nvăt, ământ nu este
mare - nu era pe măsura misiunii la care ne angajasem de bunăvoie - dascăl
ajungea doar cine avea chemare), ci ne motiva pasiunea pentru meseria aleasă,
vocat, ia, responsabilitatea fat, ă de viitorul generat, iilor de copii s, i de tineri care ni
se ı̂ncredint, au s, i pe care ı̂i cres, team, instruindu-i s, i educându-i, formându-i cu
Cum am ajuns profesor de franceză la Liceul Potcoava. 11

deprinderi practice s, i cetăt, enes, ti, cu dragoste de viat, ă, de muncă, de frumos, de
cultură, cu spirit competitiv s, i, mai ales, cu dragoste de T, ară s, i de Popor (ı̂nainte
de 1990, ,,Patria” s, i ,,Poporul” erau realităt, i sacre, ı̂n slujba cărora ne desfăs, uram,
sincer s, i fără menajamente, activitatea; aceste entităt, i fiint, iale românes, ti nu erau
doar vorbe goale, de circumstant, ă, de ,,campanii electorale” ca ı̂n ultimii 30
de ani), cons, tientizându-i s, i responsabilizându-i fat, ă de viitorul lor s, i al T, ării,
determinându-i s, i capacitându-i să-s, i construiască singuri s, i să-s, i asume acest
viitor.

Luând act de această realitate, cadrele didactice de astăzi care nu mai sunt
angajate ı̂n multitudinea s, i varietatea activităt, ilor de ı̂nainte de anii ’90 ai secolului
trecut - au obligat, ia firească să-s, i canalizeze toate eforturile s, i priceperea pentru
ridicarea la cele mai ı̂nalte cote a calităt, ii ı̂nvăt, ământului, pentru eficient, a activităt, ii
instructive s, i educative desfăs, urate la clasă s, i ı̂n afara acesteia. Din nefericire,
să-mi fie iertat, o spun eu care am slujit ı̂nvăt, ământul (s, i cultura) 42 de ani, nu se
ı̂ntâmplă aceasta ı̂ntotdeauna astăzi - mă refer, ı̂n primul rând la disciplina pe care
am predat-o ı̂n ultimii 24 de ani, Româna, regina ı̂nvăt, ământului românesc, la
care unii profesori (din păcate, destui) nu mai parcurg integral materia prevăzută
de programa s, colară, nu mai lucrează cu s, i după manualul s, colar (pe care elevii ı̂l
primesc gratuit, statul cheltuind sume mari de bani pentru a-l pune la dispozit, ia
elevilor s, i profesorilor), neluând ı̂n seamă un lucru elementar: faptul că etimonul
acestui cuvânt, latinescul ,,manus” (mână) ı̂i obligă să aibă manualul permanent
,,ı̂n mână” s, i ,,la ı̂ndemână”, ca instrument de lucru, sine qua non, ı̂n s, coală (cu
manual s, colar s-a făcut ı̂nvăt, ământul ı̂ntotdeauna s, i peste tot ı̂n lume!). Aces, ti
,,funct, ionari” de circumstant, ă (pe care nu ı̂i pot numi dascăli de s, coală) lucrează
cu elevii ,,după ureche” (ba se ı̂mpăunează că sunt inventivi s, i creativi!) măcinând
ı̂n moara comodităt, ii s, i iresponsabilităt, ii menirii lor, ı̂n cele patru clase de liceu,
numai 17 autori as, a-zis, i canonici, parcurgând, din creat, ia acestora, câte o singură
operă literară (prevăzută ı̂n programa examenului de Bacalaureat), ignorând nu
doar cultura literară, ci s, i cultura ı̂n general a elevilor, viitorul profil spiritual al
lor, sfidând orice moralitate deontologică. Nu mai aduc ı̂n discut, ie insuficienta
pregătire profesională a absolvent, ilor de facultăt, i, aceasta din cauza sistemului de
ı̂nvăt, ământ superior care a renunt, at la stagiul de patru sau cinci ani de studiu,
acesta reducându-se la trei ani ı̂n ideea continuării pregătirii prin studii de masterat,
pe care ı̂nsă absolvent, ii, de regulă, nu le fac ı̂n specialitatea obt, inută anterior, ci
aleg un alt profil, pentru a obt, ine, zic ei, o nouă specializare, ceea ce contrazice
ı̂nsăs, i semantica termenului ,,master”, care presupune studii aprofundate ı̂n ceea
ce au studiat anterior, or as, a-zisul profesionist nu poate fi performant ı̂n niciuna
dintre specializări. Superficialitatea s, i insuficient, a prin care se abordează (de
fapt, se mutilează) programa s, colară, generează efecte păgubitoare ı̂n formarea
profilului spiritual al elevilor, viitorii absolvent, i, care nu sunt solicitat, i s, i obis, nuit, i
să gândească, să interpreteze un fenomen cultural s, i literar, o operă artistică,
nu sunt supus, i la eforturi intelectuale, fiind stimulat, i doar să memoreze mecanic
comentariile celor 17-18 opere literare prevăzute pentru Bacalaureat.
12 Dumitru LAZĂR

Cu aceste carent, e s, i inconvenient, e m-am confruntat s, i eu, profesional, ı̂n


momentul când, ı̂n 2006, m-am ı̂ntors la catedră (la Colegiul Nat, ional de Informatică
,,Matei Basarab” Râmnicu-Vâlcea) după nouă ani (perioadă ı̂n care am exercitat
funct, ia de director la Biblioteca Judet, eană Vâlcea) s, i am avut neplăcuta surpriză
să nu mai găsesc elevii pe care-i s, tiam dintotdeauna, elevii tradit, ionali, care erau
dispus, i s, i simt, eau nevoia să-s, i urmeze profesorul ı̂n demersul didactic, doritori să
ı̂nvet, e s, i să s, tie cât mai mult s, i cât mai multe, politicos, i, cu bun-simt, , cu respect
fat, ă de s, coală s, i profesori s, i as, a mai departe. Am aflat nis, te tineri distant, i, blazat, i,
veleitari, cu pretent, ii de atots, tiutori, cu predispozit, ii pentru cancan, obis, nuit, i
să comunice s, i să se comunice ı̂n afara subiectului lect, iei (cu vorbe spuse de
dragul vorbelor pentru trecerea timpului), contestatari, neobis, nuit, i să respecte
,,ritualul lect, iei”. A trebuit, o perioadă să mă confrunt cu rezistent, a acestora la
obligat, iile de elev, să-i determin să cons, tientizeze că s, coala (indiferent de evolut, ia
celorlalte institut, ii ale statului) rămâne s, coală, să le demonstrez cât s, i câte nu
s, tiu din câte ar trebui să s, tie, s, i, evident, notele mai mici nu au ı̂ntârziat să vină,
ceea ce a generat nemult, umire totală (fuseseră obis, nuit, i cu note mari, obt, inute
facil, pe care le revendicau ca un drept, pentru că le trebuiau pentru admitere ı̂n
facultate, fără, ı̂nsă, a face efortul să ı̂nvet, e să ı̂s, i ı̂nsus, ească materia prevăzută
de manualele s, colare, care le erau străine), au ı̂nceput să cârtească, ba chiar să
se plângă la diriginte, la director, să reclame, inclusiv la Inspectoratul S, colar, că
predau ,,academic”, că le cer prea mult, că vorbesc ı̂n Limba franceză, ı̂n Limba
latină, ı̂n Limba greacă etc. (ı̂n realitate, nu făceam decât conexiuni cu literatura
franceză s, i universală sau explicam etimonul unor cuvinte provenite din latină sau
greacă). Evident că nu am cedat (cum, din păcate, au făcut mult, i profesori din
generat, ia postdecembristă). Aveam destulă experient, ă ca să rămân Profesor, iar
elevii erau destul de vulnerabili (le-am dovedit-o prin testele de evaluare periodică,
uneori chiar săptămânale, a căror notă s, i-o propuneau singuri, pe baza baremului
de corectare s, i evaluare pe care li-l dădeam), destul de ,,necopt, i” ca să le ,,ı̂nfrâng”
rezistent, a (s, i reticent, a) s, i să-i ı̂ntorc (să-i aduc) la condit, ia de elev. Nu a fost
deloc us, or (ı̂ntrucât nu aveam de partea mea nici părint, ii, nici dirigint, ii s, i nici
conducerea s, colii, fiind chiar luat ı̂n discut, ie ı̂n consiliul de administrat, ie, cu scopul
de a mă modela, ,,reeduca” ı̂n spiritul liniei noi a ,,faire ce qu’on veut” sau a
unui ,,dolce far’niente”, recomandându-mi-se, ,,grijuliu”, să nu mă complic, ca să
nu am probleme) s, i nici timpul de revenire la standardelele tradit, ionale ale s, colii
românes, ti nu a fost scurt - a durat un an, ba chiar doi, până când elevii - după
ce au epuizat toate armele de rezistent, ă - s-au ,,dat pe brazdă” s, i au acceptat
să ı̂nvet, e, parcurgând ı̂ntreaga programă s, colară, prevăzută de manualul cel mai
complet (coordonat de academicianul Eugen Simion, pentru clasele a XI-a s, i a
XII-a) ajungând ca, la bacalaureat, absolvent, ii clasei a XII-a E - S, tiint, e sociale
(prima promot, ie, după ce m-am ı̂ntors la catedră, din 2010) să obt, ină media clasei
nouă fără cinci sutimi, adică 8,95, iar patru elevi au obt, inut cele mai mari note
din judet, (evident alături de alt, i absolvent, i din Colegiile Nat, ionale ,,Alexandru
Lahovari” s, i ,,Mircea cel Bătrân”). Era momentul de bilant, ı̂n care elevii s, i
părint, ii trăiau momentul sublim al recunos, tint, ei (exuberant, a s-a instituit peste
Cum am ajuns profesor de franceză la Liceul Potcoava. 13

tot, satisfact, ia mea, a muncii ı̂mplinite s, i răsplătite, era totală, iar elevii nu mai
s, tiau cum să-mi mult, umească s, i să ı̂s, i ceară scuze pentru atitudinea lor refractară).
Mărturisesc că, ı̂n continuare, elevii mei din promot, ia următoare (2014, când am
fost pensionat) cunoscându-mi exigent, a s, i stilul, s-au conformat calităt, ii de s, colar
fără să mai cârtească s, i rezultatele au fost tot la ı̂nălt, ime, chiar ı̂n condit, iile ı̂n care
,,s, i-au irosit timpul”, cum ziceau colegii lor din clasele altor profesori, studiind tot, i
scriitorii prevăzut, i de manualele s, colare s, i toate elementele de teorie s, i de istorie
literară, de limbă, de cultură s, i de artă. Elevii clasei a XII-a (Filologie bilingv),
cărora le-am fost diriginte, au obt, inut la Bacalaureat, ı̂n majoritate, note bune
s, i foarte bune (doar trei elevi, dintre cei cinci pe care trebuia să-i las corigent, i,
au luat medii sub 8: 7,45, 7,75, 7,80 - aces, tia, cum au ies, it de la examen, m-au
sunat să-mi mult, umească pentru faptul că i-am obligat să ı̂nvet, e poeziile pe de
rost - subiectul de ,,Bac” ı̂n 2014 a fost din poezia interbelică) - 18 din cei 32 de
absolvent, i au obt, inut note ı̂ntre 9 s, i 9,95, inclusiv notele cele mai mari din judet, .
Am evocat aceste ,,poves, ti adevărate”, am insistat s, i detaliat aceste experient, e
personale, nu pentru ,,faima” mea, ci pentru ca să scot ı̂n evident, ă s, i faptul că
nu numai elevii sunt vinovat, i (aces, tia s, tiu să speculeze slăbiciunile profesorilor,
ale oamenilor mari; e drept că sunt ,,ajutat, i” uneori de părint, i, care derutat, i de
dezordinea generală din societate, nu văd decât necesitatea ca fiii lor să aibă note
mari, dar nu să s, i ı̂nvet, e pentru a le merita), ci s, i unii profesori care au renunt, at la
exigent, ă, care au cedat ı̂n fat, a elevilor, inversând rolurile: nu profesorul, ci elevul
este suveran ı̂n democrat, ia noastră românească, situat, ie care generează haos ı̂n
ı̂nvăt, ământ, cu reflexe negative ı̂n cons, tiint, a elevilor, care nu mai văd utilitatea
efortului de a ı̂nvăt, a, care pleacă din liceu cu mintea goală, fără cultură, fără
demnitatea cons, tiint, ei de sine, fără a discerne ı̂ntre ceea ce este rău s, i ceea ce
este bine pentru ei ı̂n viitor, nepregătit, i pentru viat, ă. Nu ı̂ntâmplător, unii dintre
aces, tia, animat, i de spiritul de aventură, la chemarea depărtărilor s, i a mirajului
Occidentului, părăsesc t, ara s, i es, uează sau se angajează ı̂n activităt, i umilitoare.
Este timpul - des, i foarte tardiv - dar nu e imposibil, ca societatea, factorii
decident, i, să se implice, hotărât s, i profund, ı̂ntr-un ,,Proiect de t, ară” al României,
ı̂n care prioritate absolută să o aibă educat, ia s, i ı̂nvăt, ământul, ı̂n primul rând,
educat, ia civică s, i, mai ales, educat, ia patriotică din s, coală, care trebuie să revină
la ı̂nălt, imea tradit, iei, sprijinită de stat, prin toate pârghiile acestuia (conducătorii
t, ării, politicienii, biserica, armata, societatea civilă etc.), care să inoculeze ı̂n
cons, tiint, a copiilor s, i tinerilor dragostea de T, ară s, i de Neam, cons, tiint, a că au
datoria sacră (as, a cum o au pentru părint, i) fat, ă de T, ară, fat, ă de pământul pe
care s-au născut s, i fat, ă de Poporul din care s-au ivit, au crescut, au fost s, colit, i cu
cheltuieli din partea statului român, să fie pătruns, i - as, a cum au fost românii de a
lungul veacurilor s, i au supraviet, uit miraculos - de faptul că T, ara, România, are
dreptul să beneficieze de munca, inteligent, a s, i aportul fiilor ei aici, ı̂ntre granit, ele
sale, nu afară ı̂n slujba s, i interesul altor t, ări (care nici măcar nu apreciază, nu
recunosc că cele aproape 4 milioane de români plecat, i ı̂n strainătate, la muncă,
au contribuit ı̂n ultimii 30 de ani, s, i contribuie s, i astăzi efectiv la sust, inerea
14 Dumitru LAZĂR

economiei s, i la prosperitatea acestora). Drepturile omului, ale individului, sunt


respectate, dar acestea nu trebuie să prevaleze drepturile nat, iunilor, ale popoarelor
din care fac parte. Se s, tie că cei mai buni dintre absolvent, ii de liceu s, i facultăt, i,
ademenit, i de cei care au nevoie de elite, de fort, ă de muncă gata pregătită, merg
să studieze s, i apoi să muncească ı̂n străinătate. Nu avem nimic ı̂mpotrivă ca
aces, tia să ı̂s, i desăvârs, ească studiile sau pregătirea profesională, dar ei trebuie
să aibă cons, tiint, a nat, ională prezentă la tinerii din generat, iile trecute, ı̂ncepând
cu jumătatea secolului al XIX-lea, pas, optis, tii, junimis, tii, până către jumătatea
secolului al XX-lea, care studiau ı̂n străinătate ca să vină ı̂n T, ară s, i să se pună ı̂n
slujba ei, ca să restituie ceea ce au primit de la Neamul lor care i-a sust, inut ı̂n
specializarea lor cu mari sacrificii. Autorităt, ile, cei care reprezintă România ı̂n
t, ară s, i ı̂n Europa unită, trebuie să renunt, e la capul plecat, la pozit, ia de drept, i,
la lipsa de demnitate, la teama de a-s, i manifesta identitatea nat, ională ı̂n cadrul
structurilor europene s, i mondiale. Plecând de la realitatea că România este a
s, aptea t, ară europeană, respectiv a s, asea fără Marea Britanie, dintre cele 28/27,
ca pondere teritorială s, i demografică, este normal să ne acordăm noi ı̂ns, ine, ı̂n
primul rând, această important, ă/acest atu s, i, apoi, s-o impunem, cu demnitate,
s, i celorlalte t, ări europene, să impunem important, a s, i rolul României de astăzi,
care asigură securitatea Europei ı̂n flancul de est al NATO, s, i al T, ărilor Române
de-a lungul veacurilor, ı̂n protejarea civilizat, iei occidentului s, i a cres, tinismului. Să
cons, tientizăm s, i să convingem (fie s, i post festum) opinia europeană s, i mondială că
România ı̂n cele două războaie mondiale a avut un rol dintre cele mai importante,
la Mărăs, es, ti, Mărăs, ti sau Oituz, la eliberarea Ungariei, alături de Frant, a, Anglia
s, i SUA, ı̂n Primul Război Mondial, s, i alături de SUA, Uniunea Sovietică s, i Anglia,
ı̂n al Doilea Război Mondial, ı̂n urma căruia România, ı̂n loc să fie considerată -
datorită contribut, iei s, i jertfelor imense materiale s, i umane - t, ară cobeligerantă ı̂n
ı̂nfrângerea Germaniei naziste, a fost obligată să plătească despăgubiri de război
Uniunii Sovietice 20 de ani, ca t, ară ı̂nvinsă, cu toate că am contribuit substant, ial
la ı̂nvingerea Germaniei s, i la retragerea ei ı̂n bârlogul propriu, luptând alături
de aliat, i până ı̂n munt, ii Tatra, ı̂ncetând ostilităt, ile ı̂n 6 mai 1945, cu doar trei
zile ı̂nainte de capitularea acesteia. Acestea sunt realităt, i pe care tinerii nos, tri
trebuie să le s, tie s, i românii nu au dreptul să le uite, dar trebuie să le s, i impună,
ca realităt, i, s, i marilor puteri din Uniunea Europeană, la care am aderat cu toată
fiint, a noastră s, i din care facem parte (ı̂nsă fără să ne bucurăm, din păcate, de
toate drepturile s, i de respectul cuvenite - aceasta din cauza lipsei de personalitate,
de demnitate s, i, poate, de patriotism a unora care ne reprezintă t, ara ı̂n lume...).
P.S. Mă opresc aici (ca să nu ocup prea mult din spat, iul Revistei) cu gândul s, i
intent, ia ca, ı̂n numărul următor al revistei care poartă numele dascălului-apostol
Marinescu-Ghemeci Octavian, să evoc trecerea mea prin Liceul Potcoava, prin
Potcoava (prin ı̂nvăt, ământ s, i prin comunitate).
Urez succes colaboratorilor s, i redactorilor Revistei (cu deosebire, init, iatorilor
ei). La multe numere ı̂ntru mult, i ani!
ARTICOLE S, I NOTE MATEMATICE

Geometric constructions, with the compass


alone, the straightedge alone or neither of them

Thanos KALOGERAKIS 1

Geometric constructions are, probably, some of the most exciting problems


among Euclidean geometry enthusiasts. Even more so, when they can be accom-
pliced using either the compass, or the straightedge alone.
Below we present three constructions, with the compass alone, three more with
the straightedge alone and one with neither of them.

Constructions with the compass alone

Problem 1 (Figure 1). Let ABCD be a trapezoid (ABkCD, AB + CD > AC)


and P the intersection point of its diagonals. Using the compass alone, construct
(at least one) point Q such that AQP
[ = CQP\.

Figure 1

Solution. If Q is the desired point, then from the Bisector Theorem in AQC,
since AQP
[ = CQP \, we get:
AQ AP
= . (1)
CQ CP
1
Mechanical Engineer, National Technical University of Athens, Kiato, Korinthia, Greece,
kalogerakis.thanos@gmail.com

15
16 Thanos KALOGERAKIS

But, from the P DC, P AB triangles’ similarity, we have:


AB AP BP
= = . (2)
CD CP DP
Combining (1), (2) we get (see Figure 2):
AQ AB
= , that implies {Q, Q0 } = (A, AB) ∩ (C, CD)
CQ CD
and
AQ BP
= , that implies {S, S 0 } = (A, P B) ∩ (C, P D).
CQ DP

Figure 2

Both pairs Q, Q0 and S, S 0 are constructed using the compass alone (four circles
with known centers and radii).
Moreover the pairs of points Q, Q0 and S, S 0 belong to the Apollonius circle
(A, C, k) where k = AP/P C.
Problem 2 (Figure 3). Given an A-righted triangle ABC, drawn on the plane.
Using the compass alone, construct the blue semicircles.
Note: altitude AD is not drawn.

Figure 3
Geometric constructions, with the compass alone, the straightedge alone or neither of them 17

Solution. First we have to construct point D on BC. Considering known the


construction of the midpoint of a segment, with the compass alone (Mascheroni
construction), we find the midpoint M of AB.
Then the circle (M, M A) intersects BC at desired point D.

Figure 4

Now we have to construct points P, Q on BD.


The observation that AP, AQ are the bisectors of the angles BAD,
\ CAD, \
implies that circle (C, CA) intersect BC at the desired point P and similarly
(B, BA) at Q.
P
\ CA
Indeed BA is tangent to (C, CA) at A and hence BAP
\=
2 which is true
(Chord Tangent Theorem) and similarly for Q.
Finally, since we know D, P and Q, we can draw the asked circles (P, P D) and
(Q, QD).
Problem 3 (Figure 5). Given two intersecting (at A, B) circles (K), (L) and
one of their centers K or L, find (construct) point P on (L) and point Q on (K),
using the compass alone, so that triangle AP Q is equilateral.

Figure 5
18 Thanos KALOGERAKIS

Solution. Suppose we know the center L.

We first draw circle (A, AL = r) which intersects (L) at C and C 0 .

Now we draw circle (C, CA = r) which intersects (K) at point Q.

Finally, we draw circle (Q, QA) which intersects (L) at point P . We claim that
Q, P are the desired points and AP Q is equilateral and we’ll prove it as follows.

Figure 6

Let AT be the common chord of congruent circles (L) and (C).

We draw T Q which intersects (L) again at P . Then clearly AQT d◦ ⇒


[ = 120
AQP
[ = 60c◦ and AP
[ c◦ .
T = 60

Thus, triangle AP Q is equilateral and since QA = QP the circle (Q, QA)


intersects (L) at point P .

Constructions with the straightedge alone

Problem 4 (Figure 7). Two circles (u), (v) have a common chord AB. A third
circle (w), through B, intersects (v) at C and (u) at D. Construct the tangents to
(w) at the points C, D, using the straightedge alone.
Note: none of the centers of the three circles are drawn.
Geometric constructions, with the compass alone, the straightedge alone or neither of them 19

Figure 7

Solution. We draw the line through CD intersecting (u) and (v) at E and F
respectively.
Next, we draw the segments AB, which intersects (w) at H, AE, which
intersects (v) at P , and AF , which intersects (u) at Q.
Now then, we’ll prove that P C and QD are the desired tangents to (w) (see
Figure 8).

Figure 8

From cyclic quadrilaterals CBDH, AEBD, AEDQ:

x1
c = x2
c = x3
c = x4⇒HCkAE.
c

From cyclic quadrilaterals CBDH, AF BC, AF CP :

z1
c = z2
c = z3
c = z4⇒HDkAF.
c

But HCkAE implies that z4


c = z5
c and since z1
c = z5,
c from the converse Chord
Tangent Theorem, we have that PC is tangent to (w).
Similarly, HDkAF implies that QD is tangent to (w).
20 Thanos KALOGERAKIS

Problem 5. Three circles (u), (v) and (w) drawn on the plane as in the Figure
9, have a common point A. Construct a transversal P QRS such that P Q = RS,
using the straightedge alone.
Note: none of the centers of the three circles are drawn.

Figure 9

Solution. Let {A, C} = (w) ∩ (u), {A, D} = (w) ∩ (v), and {A, E} = AC ∩ (v),
{A, F } = AD ∩ (u).
The line through E and F , is the desired transversal and intersects again (u)
and (v) at P and S, while Q and R are its intersections with (w) (Figure 10).
Now then, we’ll prove that P Q = RS.
An experienced solver recognizes the Haruki’s lemma configuration.

Figure 10

We apply the Intersecting Chords Theorem to AD crossed by QS in the


circles (w), (v):

QF · F R = AF · F D in (w) and EF · F S = AF · F D in (v),


Geometric constructions, with the compass alone, the straightedge alone or neither of them 21

from which (QE + EF ) · F R = EF · (F R + RS) or QE · F R = EF · RS,


and finally
QE · F R
= RS. (3)
EF
Similarly, Intersecting Chords Theorem to AC crossed by P R in the circles
(w), (u), implies
QE · F R
= P Q. (4)
EF
From (3), (4) we have P Q = RS.
Problem 6. In the Figure 11, find (construct) the midpoint of segment AB, using
the straightedge alone.
Note: It isn’t necessary for the circles’ centers to be known to complete the
construction.

Figure 11

Solution. Let {P, Q} = (u) ∩ (v), S ∈ (v) ∩ AB and T ∈ (u) ∩ AB.


We draw segments P S, P T and QS, QT .
Then let C ∈ P S ∩ (u), D ∈ P T ∩ (v), E ∈ QS ∩ (u) and F ∈ QT ∩ (v).
We draw AE, BF and AC, BD.
Then let {K} = AE ∩ BF and {L} = AC ∩ BD.

Figure 12
22 Thanos KALOGERAKIS

Clearly, a1
c = a2 c = a3
c and b1b = b2
b = b3,
b that implies triangles KAB and LAB
are isosceles and therefore K and L are lying on the perpendicular bisector of AB.
Finally we draw KL which intersects AB at the desired point M , the midpoint
of AB.

A construction with neither a compass nor a straightedge

Problem 7 (Figure 13). Given, as much cardboard squares as needed, of sides a,


b and a unit segment on the plane, construct segments with lengths:
√ √ √ √
17, 37, 65, 101.

Figure 13

Solution. We place the a-side and b-side squares, as in the Figure 14.
√ √ √ √
It’s easy to prove that c1 = 2, c2 = 5, c3 = 10, . . . , cn = n2 + 1.

Figure 14
Utilitatea unei formule ı̂n rezolvarea unor
probleme de calcul matriceal

Florin STĂNESCU 1

În cele ce urmează ne propunem prezentarea unei formule de calcul matriceal


s, i a unor aplicat, ii ce evident, iază utilitatea acestei formule.

Propozit, ia 1. Dacă A, B ∈ M2 (C) sunt două matrice s, i x ∈ C, atunci avem


egalitatea:

det (A + xB) = det A + (tr A · tr B − tr (AB)) · x + det B · x2 . (∗)

   
a b e f
Demonstraţie. Luând A = s, i B = , obt, inem:
c d g h
a + xe b + xf
det (A + xB) = = x2 (eh − gf ) + x (ed + ah − bg − cf ) + ad −
c + xg d + xh
bc = x2 det B+ x (tr A · tr B − tr (AB)) + det A.

Aplicat, ia 1. Dacă A, B ∈ M2 (C) sunt două matrice, atunci propozit, iile următoare
sunt echivalente:

p1 : tr (AB) = tr (A) · tr (B) ;


p2 : det (A + B) = det A + det B;
p3 : det (A − B) = det (A + B) .

Solut, ie. Luând ı̂n (∗) x = 1, respectiv x = −1, obt, inem că det (A + B) − det A −
det B =tr A·tr B −tr (AB) s, i det (A − B)−det A−det B = −tr A·tr B +tr (AB),
ceea ce arată că p1 ⇔ p2 ⇔ p3 .

Aplicat, ia 2. Dacă A, B ∈ M2 (C) sunt două matrice, atunci are loc loc relat, ia:

det (A + B) + det (A − B) = 2 (det A + det B) .

Solut, ie. Luând x = 1, respectiv x = −1 ı̂n relat, ia (∗) s, i adunând cele două relat, ii
concluzia este imediată.
1
Profesor, S, coala Gimnazială ,,S, erban Cioculescu”, Găes, ti, florin.florinstanescu@yahoo.com

23
24 Florin STĂNESCU

Aplicat, ia 3. Dacă A, B, C ∈ M2 (C) sunt trei matrice, atunci are loc loc relat, ia:

det (A + B + C)+det A+det B+det C = det (A + B)+det (B + C)+det (C + A) .

Solut, ie. Cu ajutorul formulei (∗), putem scrie: det (A + B + C) = det (A + B) +


tr (A + B) tr (C) − tr (AC + BC) + det C = det (B + C) + tr (B + C) tr (A) −
tr (BA + CA)+det A = det (C + A) +tr (C + A) tr (B)−tr (CB + AB)+det B.
Adunând cele trei relat, ii, avem:
3 det (A + B + C) = det (A + B) + det (B + C) + det (C + A) + det A + det B +
det C + 2 [tr A · tr B − tr (AB) + tr B · tr C − tr (BC) + tr C · tr A − tr (CA)]
(∗) 
= det (A + B)+det (B + C)+det (C + A)+det A+det B+det C+2 det (A + B)−
det A − det B + det (B + C) − det B − det C + det (C + A) − det C − det A , de
unde concluzia este imediată.
(tr (A))2 − tr A2

Aplicat, ia 4. Dacă A ∈ M2 (C) ,atunci det A = .
2

Solut, ie. Luând x = 1 ı̂n (∗) s, i ı̂nlocuind pe B cu A, concluzia este imediată.

Aplicat, ia 5. Fie A, B ∈ M2 (C) . Arătat, i că:

det [(A − B) (A + B)] = det A2 − B 2 ⇔ (AB − BA)2 = O2 .




Solut, ie. Din Teorema Hamilton-Cayley, cum tr (AB) = tr (BA), rezultă că
(AB − BA)2 = − det (AB − BA) · I2 . Cu ajutorul formulei (∗) putem scrie:
det [(A − B)(A + B)] = det A2 − B 2 + AB − BA = det A2 − B 2 +


tr A2 − B 2 · tr (AB − BA) − tr A2 − B 2 (AB − BA) + det (AB − BA).


  
| {z } | {z }
=0 =0
Astfel, avem: det [(A − B) (A + B)] = det A2 − B 2 ⇔ det (AB − BA) = 0 ⇔


(AB − BA)2 = O2 .

Aplicat, ia 6. Fie A, B ∈ M2 (C) astfel ı̂ncât A2 + B 2 = 2AB.

a) Arătat, i că AB = BA.

b) Arătat, i că tr A = tr B.

Solut, ie. a) Relat, ia din enunt, poate fi rescrisă că (A − B)2 = AB − BA, deci avem:
  (∗)
0 = det (A − B)2 − (AB − BA) = det (A − B)2 +det (AB − BA)−tr (A − B)2 ·
h i
tr (AB − BA) + tr (AB − BA) (A − B)2 = det (A − B)2 +det (AB − BA). Re-
| {z } | {z }
=0
=0
zultă că det (A − B)2 = det (AB − BA) = 0, deci det (A − B) = 0. Din Teorema
Hamilton-Cayley obt, inem (AB − BA)2 = O2 s, i (A − B)2 = tr (A − B) · (A − B),
Utilitatea unei formule ı̂n rezolvarea unor probleme de calcul matriceal 25

deci O2 = (A − B)4 = (tr (A − B))3 · (A − B), de unde (A − B)2 = O2 . Astfel


AB − BA = O2 , deci AB = BA.
b) Din (A − B)2 = O2 , obt, inem tr (A−B)·(A−B) = O2 , deci (tr (A − B))2 =
0 s, i astfel tr A = tr B.
Aplicat, ia 7. Fie A, B ∈ M2 (C) cu proprietatea A2 + B 2 = AB. Să se arate că

(AB − BA)2 = O2 .

Solut, ie. Este clar că det A2 + B 2 = det (AB) . Acum, plecând de la relat, ia din


enunt, , putem scrie:


A2 + B 2 = AB ⇒ A2 + B 2 − (AB − BA) = BA ⇒ det A2 + B 2 − (AB − BA)

(∗)
= det (AB) ⇒ det A2 + B 2 +det (AB − BA) − tr A2 + B 2 · tr (AB − BA)
 
| {z }
=0
+ tr A + B 2 (AB − BA) = det (AB) ⇒ det (AB − BA) = 0. Din Teorema
 2  
| {z }
=0
Hamilton-Cayley obt, inem (AB − BA)2 = O2 .
Aplicat, ia 8. Fie A, B ∈ M2 (C) asttfel ı̂ncât (A − B)2 = O2 .

a) Arătat, i că det A2 − B 2 = (det A − det B)2 .




b) Demonstrat, i că det (AB − BA) = 0 dacă s, i numai dacă det A = det B.

Solut, ie. a) Întrucât (A − B)2 = O2 , avem det(A − B) = 0. Din Teorema


Hamilton-Cayley obt, inem tr (A − B) · (A − B) = O2 , deci tr (A − B) = 0. Fie
tr A = tr B = α. Avem A2 = αA − det A · I2 s, i B 2 = αB − det B · I2 , prin
(∗)
urmare det A2 − B 2 = det [α (A − B) + (det B − det A) I2 ] = α2 det (A − B) +

2
(det A − det B)2 = det A − det B .
(∗)
b) Cum 0 = det [(A − B) (A + B)] = det A2 − B 2 + det (AB − BA), echiva-


lent, a din enunt, este o consecint, ă imediată a egalităt, ii de la punctul a).


Aplicat, ia 9. Fie A, B ∈ M2 (C) cu A2 + B 2 + 2AB = O2 . Să se arate că

det A = det B.

Solut, ie. Conform Aplicat, iei 6 pentru matricele A s, i −B rezultă că AB = BA s, i


tr A = −tr B. Acum relat, ia din enunt, se poate rescrie sub forma (A + B)2 = O2 ,
deci det(A + B) = 0. Fie α = tr A = −tr B. Din Teorema Hamilton-Cayley avem
A2 = αA − det A · I2 s, i B 2 = −αB − det B · I2 , de unde 0 = det(A2 − B 2 ) =
(∗)
det [α(A + B) + (det B − det A)I2 ] = α2 det(A + B) + 2α(det B − det A)tr (A + B)
−α(det B − det A)tr (A + B) + (det A − det B)2 = (det A − det B)2 , prin urmare
det A = det B.
26 Florin STĂNESCU

Aplicat, ia 10. Fie A, B ∈ M2 (C) cu A2 + B 2 + 2AB = O2 . Să se arate că

det (tr A · A − tr B · B) = 0.

Solut, ie. Din solut, ia aplicat, iei anterioare avem det(A + B) = 0 s, i tr A = −tr B,
deci det(tr A · A − tr B · B) = det(tr A · A + tr B · B) = (tr A)2 · det(A + B) = 0.

Bibliografie

[1] Gh. Andrei, C. Caragea, Gh. Bordea, Algebra pentru concursuri de admitere s, i
olimpiade s, colare, Editura Topaz, Constant, a, 1993.

[2] M. Andronache, R. Gologan, D. Schwarz, D. S, erbănescu, Olimpiada de mate-


matică 2006-2010, Editura Sigma, Bucures, ti, 2010.

[3] A. Chites, , G. Dospinescu, A. Ismail, G. Kreindler, C. Popa, C. Raicu, A.


Zahariuc, Probleme alese de matematică pentru pregătirea Olimpiadei Nat, ionale,
Editura Gil, Zalău, 2010.

[4] G.H. Golub, C.F. Van Loan, Calculul Matricial, trad. A. Cipu s, i M. Cipu,
Editura Theta, Bucures, ti, 2005.

[5] F. Stănescu, Probleme de calcul matriceal: olimpiade, concursuri s, colare s, i


bacalaureat, Editura Cartea Românească, Pites, ti, 2018.
On a Geometric Theorem

Leonard GIUGIUC 1 and Bogdan SUCEAVĂ 2

In this paper we’ll give the admissible values of k for which

(a + b + c + d)2 + kabcd ≥ 16 + k (1)

holds for all positive real numbers a, b, c and d satisfying a+b+c+d = a1 + 1b + 1c + d1 .


From a + b + c + d = a1 + 1b + 1c + d1 we get

abcd (a + b + c + d) abc + abd + acd + bcd


1= and abcd = .
abc + abd + acd + bcd a+b+c+d

So (1) writes as
 
2 abc + abd + acd + bcd (16 + k) (a + b + c + d) abcd
(a + b + c + d) + k ≥ .
a+b+c+d abc + abd + acd + bcd
(2)
Firstly, we’ll find k ≥ 0 for which (2) holds for all a, b, c, d > 0.
We may assume WLOG in (2) that a + b + c + d = 4.
Via AM-GM we get 0 < abcd ≤ 1.

Claim 1. The function f : [0, 1) → (0, 1] as f (t) = (1 − t)3 (1 + 3t) ∀t ∈ [0, 1) is


bijectively decreasing.

Proof. As f 0 (t) = −12t(1 − t)2 ≤ 0 ∀t ∈ [0, 1), we deduce that f is strictly


decreasing. On the other hand, f (0) = 1 and f (1 − 0) = 0; f is continuous, hence
it is well defined and bijective.

Claim 2. Let t ∈ [0, 1) be a fixed real number. We consider the positive numbers
a, b, c, d such that a + b + c + d = 4 and abcd = (1 − t)3 (1 + 3t). Then

min (abc + abd + acd + bcd) = 4(1 − t)2 (1 + 2t) .


1
Professor, Colegiul Nat, ional ,,Traian”, Drobeta Turnu Severin, leonardgiugiuc@yahoo.com
2
Professor, Department of Mathematics, California State University, bsuceava@fullerton.edu
This paper was presented at the 23-th Annual Conference of S.S.M.R., Pites, ti, 2019.

27
28 Leonard GIUGIUC and Bogdan SUCEAVĂ

Proof. Consider the polynomial P (x) = (x − a) (x − b) (x − c) (x − d) on (0, ∞).


We have: P (x) = x4 − 4x3 + mx2 − sx + (1 − t)3 (1 + 3t) ∀x > 0, where m and s
are positive with s = abc + abd + acd + bcd.
Consider now the function g (x) = Px(x)
2 on (0, ∞). Since g has 4 positive roots,
by Rolle’s Theorem g 0 has at least 3 positive roots. We have:
s 2(1 − t)3 (1 + 3t)
g 0 (x) = 2x − 4 +
− , ∀x > 0.
x2 x3
3
h i
Finally, let the function h (x) = x2 2x − 4 + xs2 − 2(1+t)x3(1−3t) = 2x3 − 4x2 −
2(1−t)3 (1+3t)
x + s, ∀x > 0. We have:
h0 (x) 3x4 − 4x3 + (1 − t)3 (1 + 3t)
= , ∀x > 0.
2 x2

Shall study now the polynomial Q (x) = 3x4 − 4x3 + (1 − t)3 (1 + 3t) on
(0, ∞). Q0 (x) = 12x2 (x − 1) ∀x > 0, such that Q is strictly decreasing on (0, 1]
and strictly increasing on [1, ∞). But Q (1) = (1 − t)3 (1 + 3t) − 1 ≤ 0. So
that Q has a unique root t1 in (0, 1] and a unique root t2 in [1, ∞), because
Q (0+ ) = (1 − t)3 (1 + 3t) > 0 and Q (∞) = ∞ > 0.
Also, because 1 − t ≤ 1 is root for Q, we deduce that t1 = 1 − t. We form
the Rolle’s sequence for the function h thus: 0+ < 1 − t ≤ t2 < ∞. Since
h (0+ ) = −∞ < 0, h (∞) = ∞ > 0 and h admits at least 3 positive roots, by a
consequence of Rolle’s Theorem we deduce that h (1 − t) ≥ 0 and h (t2 ) ≤ 0. By
h (1 − t) ≥ 0 we obtain that s ≥ 4(1 − t)2 (1 + 2t).
Let’s note that if a = b = c = 1 − t and d = 1 + 3t, then a + b + c + d = 4,
abcd = (1 − t)3 (1 + 3t) and abc + abd + acd + bcd = 4(1 − t)2 (1 + 2t). So indeed,
min (abc + abd + acd + bcd) = 4(1 − t)2 (1 + 2t).
 
By Claim 2, (a + b + c + d)2 + k abc+abd+acd+bcd
a+b+c+d ≥ 16 + k(1 − t)2 (1 + 2t)
(16+k)(1−t)(1+3t) (16+k)(a+b+c+d)abcd
and 1+2t ≥ abc+abd+acd+bcd , with equality for a = b = c = 1 − t
(16+k)(1−t)(1+3t)
and d = 1 + 3t, so from (2) we get 16 + k(1 − t)2 (1 + 2t) ≥ 1+2t . We
2 (16+k)(1−t)(1+3t) 48t2 4t3
have: 16 + k(1 − t) (1 + 2t) ≥ 1+2t ∀t ∈ [0, 1) ⇔ 1+2t ≥ k (1 − t) · 1+2t
48 48 48
∀t ∈ [0, 1) ⇔ 4t(1−t) ≥ k ∀t ∈ (0, 1). Hence k ≤ inf 4t(1−t) = min 4t(1−t) = 48,
t∈(0,1) t∈(0,1)
at t = 12 .
In conclusion, if 0 ≤ k ≤ 48 then (2) holds for all a, b, c, d > 0.
Moreover, if k = 48 then we have equality in (2) at (α, α, α, α) or (α, α, α, 5α)
and permutations, α > 0.
Hence if 0 ≤ k ≤ 48, then (1) holds for all positive real numbers a, b, c and d
satisfying a + b + c + d = a1 + 1b + 1c + d1 . Moreover, if k = 48 then we have equality
√ 2 2 2

in (1) at (1, 1, 1, 1) or 10, √10 , √10 , √10 and permutations.
On a Geometric Theorem 29

Shall prove that if k > 48, then (1) does not hold for all positive real numbers
1 1 1 1
a,
√b, c and d satisfying  a + b + c + d = a + b + c + d . Indeed, if we put the vector
10, √210 , √210 , √210 in (1) we get k ≤ 48, which is false.

Now we prove that if k ∈ [−16, 48] then (1) holds for all positive real numbers
a, b, c and d satisfying a + b + c + d = a1 + 1b + 1c + d1 .
h i h
We have: 64 (a + b + c + d)2 + kabcd − 16 − k − (16 + k) (a + b + c + d)2 +
i h i
48abcd − 64 = (48 − k) (a + b + c + d)2 − 16abcd .
 
Via Maclaurin’s Inequality, (a + b + c + d)2 ≥ 16 abc+abd+acd+bcd
a+b+c+d = 16abcd
h i h i
⇒ 64 (a + b + c + d)2 +kabcd−16−k ≥ (16 + k) (a + b + c + d)2 +48abcd−64 .
h
But 16 + k ≥ 0 and (a + b + c + d)2 + 48abcd − 64 ≥ 0 ⇒ 64 (a + b + c + d)2 +
i
kabcd − 16 − k ≥ 0 ⇒ (a + b + c + d)2 + kabcd ≥ 16 + k.

It remains to study the existence of k < −16 for which (1) holds for all positive
real numbers a, b, c and d satisfying a + b + c + d = a1 + 1b + 1c + d1 . As above, shall
study first the existence of k < −16 in (2), for all a, b, c, d > 0.
Again, we may assume WLOG in (2) that a + b + c + d = 4.
Via AM-GM we get 0 < abcd ≤ 1.

Claim 3. The function f : 0, 31 → (0, 1] as f (t) = (1 + t)3 (1 − 3t) ∀t ∈ 0, 13


   

is bijectively decreasing.

Proof. As f 0 (t) = −12t(1 + t)2 ≤ 0 ∀t ∈ 0, 13 , we


 
 deduce that f is strictly
decreasing. On the other hand, f (0) = 1 and f 13 − 0 = 0; f is continuous, hence
it is well defined and bijective.

Claim 4. Let t ∈ 0, 13 be a fixed real number. We consider the positive numbers


 

a, b, c, d such that a + b + c + d = 4 and abcd = (1 + t)3 (1 − 3t). Then

max (abc + abd + acd + bcd) = 4(1 + t)2 (1 − 2t) .

Proof. Consider the polynomial P (x) = (x − a) (x − b) (x − c) (x − d) on (0, ∞).


We have: P (x) = x4 − 4x3 + mx2 − sx + (1 + t)3 (1 − 3t) ∀x > 0, where m and s
are positive with s = abc + abd + acd + bcd. Consider now the function g (x) = Px(x)
2
on (0, ∞). Since g has 4 positive roots, by Rolle’s Theorem g 0 has at least 3
positive roots. We have:

s 2(1 + t)3 (1 − 3t)


g 0 (x) = 2x − 4 + − ∀x > 0.
x2 x3
30 Leonard GIUGIUC and Bogdan SUCEAVĂ
2(1+t)3 (1−3t)
h i
s
Finally, let the function h (x) = x2 2x − 4 + x2
− x3
= 2x3 − 4x2 −
2(1+t)3 (1−3t)
x + s, ∀x > 0. We have:

h0 (x) 3x4 − 4x3 + (1 + t)3 (1 − 3t)


= ∀x > 0.
2 x2

Shall study now the polynomial Q (x) = 3x4 − 4x3 + (1 + t)3 (1 − 3t) on (0, ∞).
Q0 (x) = 12x2 (x − 1) ∀x > 0, such that Q is strictly decreasing on (0, 1]
and strictly increasing on [1, ∞). But Q (1) = (1 + t)3 (1 − 3t) − 1 ≤ 0. So
that Q has a unique root t1 in (0, 1] and a unique root t2 in [1, ∞), because
Q (0+ ) = (1 + t)3 (1 − 3t) > 0 and Q (∞) = ∞ > 0. Also, because 1 + t ≥ 1 is root
for Q, we deduce that t2 = 1 + t. We form the Rolle’s sequence for the function h
thus: 0+ < t1 ≤ 1 + t < ∞. Since h (0+ ) = −∞ < 0, h (∞) = ∞ > 0 and h admits
at least 3 positive roots, by a consequence of Rolle’s Theorem we deduce that
h (t1 ) ≥ 0 and h (1 + t) ≤ 0. By h (1 + t) ≤ 0 we obtain that s ≤ 4(1 + t)2 (1 − 2t).
Let’s note that if a = b = c = 1 + t and d = 1 − 3t, then a + b + c + d = 4,
abcd = (1 + t)3 (1 − 3t) and abc + abd + acd + bcd = 4(1 + t)2 (1 − 2t). So indeed,
max (abc + abd + acd + bcd) = 4(1 + t)2 (1 − 2t).
 
By Claim 4, (a + b + c + d)2 + k abc+abd+acd+bcd
a+b+c+d ≥ 16 + k(1 + t)2 (1 − 2t)
(16+k)(1+t)(1−3t) (16+k)(a+b+c+d)abcd
and 1−2t ≥ abc+abd+acd+bcd , with equality for a = b = c = 1 + t and
d = 1 − 3t, so from (2) we get 16 + k(1 + t)2 (1 − 2t) ≥ (16+k)(1+t)(1−3t)
1−2t . We have:
2 (16+k)(1+t)(1−3t) 48t2 4t3
 1
16 + k(1 + t) (1 − 2t) ≥ 1−2t  ∀t ∈ 0, 3 ⇔ 1−2t ≥ −k (1 + t) · 1−2t
 1 48
∀t ∈ 0, 3 ⇔ 4t(1+t) ≥ −k ∀t ∈ 0, 13 . Hence −k ≤ inf 4t(1+t) 48
= 27.
t∈(0, 31 )

In conclusion, if −27 ≤ k < −16 then (2) holds for all a, b, c, d > 0 and hence
(1) holds for all a, b, c, d > 0 satisfying a + b + c + d = a1 + 1b + 1c + d1 .
Let k < −27. We define on (0,∞)4 the function 
(16+k)(a+b+c+d)abcd
ϕ (a, b, c, d) = (a + b + c + d)2 + k abc+abd+acd+bcd
a+b+c+d − abc+abd+acd+bcd .
k
Then lim ϕ (1, 1, 1, x) = 9 + 3 < 0 ⇒ ∃x > 0 for which ϕ (1, 1, 1, x) < 0 ⇒
x&0
ϕ (α, α, α, αx) < 0 ∀α > 0. Let
qus find α > 0 for which α + α + α + αx = α1 + α1 +
q q q q
1 1 3x+1 3x+1 3x+1 3x+1 3x+1
α + αx ⇒ α = x(x+3) ⇒ ϕ x(x+3) , x(x+3) , x(x+3) , x · x(x+3) < 0. We
q q
3x+1 3x+1
set a = b = c = x(x+3) and d = x · x(x+3) . Then a + b + c + d = a1 + 1b + 1c + d1
and (a + b + c + d)2 + kabcd < 16 + k.
In conclusion, the required set of admissible k equals the interval [−27, 48].
Lema lui Hensel. Aplicat, ii

Victor ALEXANDRU 1 s, i Stelian Corneliu ANDRONESCU 2

Fie p un număr natural prim. Într-o notă precedentă [3] a fost expusă con-
struct, ia inelului ı̂ntregilor p-adici notat cu Op . Are loc reprezentarea p-adică (sau
reprezentarea canonică):

Op = a0 + a1 p + a2 p2 + . . .

0 ≤ an ≤ p − 1, an ∈ N, ∀n ∈ N .

În particular, dacă ı̂ntregul p-adic α̂ ∈ N, atunci reprezentarea sa p-adică este chiar
reprezentarea sa ı̂n baza p. Grupul unităt, ilor (elementelor inversabile) U (Op ) din
Op este format din acei intregi p-adici

α̂ = a0 + a1 p + . . . + an pn + . . .

pentru care (a0 , p) = 1 (adică 1 ≤ a0 ≤ p − 1). Acest fapt rezultă astfel: dacă
(a0 , p) = 1 atunci se poate determina β̂ = b0 + b1 p + . . . + bn pn + . . . unic, astfel
ı̂ncât (b0 + b1 p + . . . + bn pn ) (a0 + a1 p + . . . + an pn ) ≡ 1 (mod pn+1 ), pentru orice
n ≥ 0 (coeficient, ii bi se determină prin induct, ie după i ≥ 0).
Reamintim că are loc reprezentarea p-adică

−1 = (p − 1) + (p − 1)p + . . . + (p − 1)pn + . . . ,

iar inversul elementului 1 − p este 1 + p + p2 + . . . + pn + . . . ∈ Op .


De asemenea, orice x = 6 0 din Op se reprezintă ı̂n mod unic sub forma x = pm · ε
cu ε ∈ U (Op ) s, i m ∈ N. S-a definit vp (x) = m (valuarea sau exponentul p-adic al
def
lui x, v(0) = ∞). În plus, x divide y ı̂n Op dacă s, i numai dacă vp (x) ≤ vp (y).
Dacă x ∈ Z obt, inem exponentul p-adic uzual din Z.
Fie x, y ∈ Op s, i n ∈ N. Notăm x ≡ 0 (mod pn ) dacă s, i numai dacă vp (x) ≥ n;
notăm x ≡ y (mod pn ) dacă s, i numai dacă x − y ≡ 0 (mod pn ).
Fiind domeniu de integritate, inelul Op are un corp de fract, ii, notat Qp , denumit
corpul numerelor p-adice. Elementele sale nenule se reprezintă ı̂n mod unic sub
forma x = pn · ε cu ε ∈ U (Op ) s, i n ∈ Z. Desigur, vp (x) = n.
1
Prof. univ. dr., Universitatea din Bucures, ti, vralexandru@yahoo.com
2
Lect. univ. dr., Universitatea din Pites, ti, corneliuandronescu@yahoo.com

31
32 Victor ALEXANDRU s, i Stelian Corneliu ANDRONESCU

Reprezentarea p-adică a numerelor rat, ionale se obt, ine ı̂n felul următor: Fie x
un număr rat, ional al cărui numitor nu se divide cu p. Putem reprezenta x = c − ab
cu c ∈ Z, a, b ∈ N, 0 ≤ a < b s, i (p, b) = 1. Din faptul că (p, b) = 1 rezultă că există
t ∈ N∗ astfel ı̂ncât pt − 1 = bu, u ∈ N. Într-adevăr, există i, j ∈ N, i < j astfel
ı̂ncât pi ≡ pj (mod b), deci pj−i ≡ 0 (mod b). Rezultă că
a au
− = = au(1 + pt + p2t + . . .),
b 1 − pt

iar 0 ≤ au < bu = pt − 1. Fie acum au = a0 + a1 p + . . . + at−1 pt−1 reprezentarea


lui au ı̂n baza p. Atunci reprezentarea p-adică a lui − ab este

a0 + a1 p + . . . + at−1 pt−1 + a0 pt + . . . + at−1 p2t−1 + a0 p2t + . . . ,

o serie periodică simplă cu perioada de lungime t. Este clar acum că, ı̂n general,
x ∈ Q admite o reprezentare p-adică printr-o serie de puteri ale lui p, serie periodică
mixtă. Se poate proba că s, i reciproc, o serie de puteri a0 + a1 p + . . . + an pn + . . .
periodică, cu 0 ≤ ai ≤ p − 1, reprezintă un număr rat, ional. De remarcat analogia
cu reprezentarea zecimală a numerelor rat, ionale.
Observat, ii. 1) Din proprietăt, ile funct, iei vp se deduce us, or că definind
 vp (x)
1
|x|p =
p

avem |x + y|p ≤ max(|x|p , |y|p ) s, i |x · y|p = |x|p · |y|p pentru orice x, y ∈ Qp (cu
convent, ia |x|p = 0 dacă x = 0). Astfel, definind

d(x, y) = |x − y|p

obt, inem d(x, y) = 0 ⇔ x = y s, i d(x, z) ≤ max (d(x, y), d(y, z)) pentru orice
x, y, z ∈ Qp . Prin urmare funct, ia d : Qp × Qp → Qp este o distant, ă pe Qp , iar
acesta este un spat, iu metric complet relativ la d (a se vedea [5]).
2) În particular, dacă F (X) ∈ Op [X] cu grad F ≥ 1, (αn )n≥1 este un s, ir
de ı̂ntregi p-adici astfel ı̂ncât vp (αn+1 − αn ) → ∞, adică |αn+1 − αn |p → 0, iar
vp (F (αn )) → ∞ atunci, considerând α ∈ Op limita s, irului Cauchy (αn )n , avem
vp (F (α)) = ∞, adică F (α) = 0.
3) O altă notat, ie frecvent folosită pentru inelul ı̂ntregilor p-adici este Zp , caz
ı̂n care Z/pZ, corpul claselor de resturi modulo p, se notează cu Fp .

Lema lui Hensel

Teorema cunoscută sub denumirea de Lema lui Hensel este, probabil, cea mai
importantă proprietate algebrică a numerelor p-adice.
Lema lui Hensel. Aplicat, ii 33

Teorema 1 (Lema lui Hensel). Fie F (X) = a0 + a1 X + a2 X 2 + . . . + an X n un


polinom cu coeficient, i ı̂n Op . Presupunem că există ı̂ntregul p-adic α1 ∈ Op astfel
ı̂ncât
F (α1 ) ≡ 0 (mod p)
s, i
F 0 (α1 ) 6≡ 0 (mod p)
unde F 0 (X) este derivata (formală) a lui F (X). Atunci există un ı̂ntreg p-adic
α ∈ Op astfel ı̂ncât α ≡ α1 (mod p) s, i F (α) = 0.

Demonstraţie. Vom arăta că rădăcina α există construind un s, ir Cauchy de ı̂ntregi


care converge la ea (metoda lui Newton). Vom construi un s, ir de ı̂ntregi p-adici
α1 , α2 , . . . , αn , . . . astfel ı̂ncât, pentru orice n ≥ 1 avem

i) F (αn ) ≡ 0 ( mod pn );

ii) αn ≡ αn+1 ( mod pn ).

Acest s, ir este un s, ir Cauchy, s, i limita sa satisface F (α) = 0 (prin continuitate) s, i


α ≡ α1 (mod p) (prin construct, ie). Deci, odată ce avem s, irul αn , teorema este
demonstrată.
Principala presupunere a teoremei este că α1 există. Pentru a-l găsi pe α2 , ı̂l
căutăm de forma
α2 = α1 + b1 p
pentru un b1 ∈ Op . Introducând această expresie ı̂n polinomul F (X) s, i dezvoltând,
obt, inem

F (α2 ) = F (α1 + b1 p)
= F (α1 ) + F 0 (α1 )b1 p + termeni ı̂n pn , cu n ≥ 2
≡ F (α1 ) + F 0 (α1 )b1 p ( mod p2 ).

Pentru al găsi pe α2 trebuie să arătăm că putem găsi b1 astfel ı̂ncât

F (α1 ) + F 0 (α1 )b1 p ≡ 0 ( mod p2 ).

S, tim că F (α1 ) ≡ 0 (mod p), deci F (α1 ) = px pentru un x. Ecuat, ia devine

px + F 0 (α1 )b1 p ≡ 0 ( mod p2 ),

care devine (după ce am ı̂mpărt, it prin p)

x + F 0 (α1 )b1 ≡ 0 (mod p).

Observăm că F 0 (α1 ) nu este divizibil cu p, deci este inversabil ı̂n Op s, i putem lua

b1 ≡ −x(F 0 (α1 ))−1 (mod p).


34 Victor ALEXANDRU s, i Stelian Corneliu ANDRONESCU

Pentru această alegere a lui b1 fixăm α2 = α1 + b1 p care are proprietăt, ile cerute.
Se procedează ı̂n continuare prin induct, ie după n ≥ 2. Fiind dat αn căutăm
αn+1 = αn + bn pn . Se obt, ine F (αn+1 ) = F (αn + bn pn ) = F (αn ) + F 0 (αn )bn pn +
termenii divizibili cu pn+1 ≡ F (αn ) + F 0 (αn )bn pn (mod pn+1 ).
Deoarece F (αn ) = pn · xn cu xn ∈ Op congruent, a F (αn+1 ) ≡ 0 (mod pn+1 )
este echivalentă cu xn + F 0 (αn )bn ≡ 0 (mod p) care are solut, ie unică modulo p
deoarece F 0 (αn ) ≡ F 0 (α1 ) 6≡ 0 (mod p).
Deoarece vp (αn+1 − αn ) ≥ n rezultă că există α = lim αn ∈ Op s, i F (α) ≡
n
0 (mod pn ), ∀n ≥ 1. Rezultă că vp (F (α)) = ∞ adică F (α) = 0.
Se poate arăta că α este unic cu proprietăt, ile din enunt, .

În continuare vom considera o formă mai generală a Lemei lui Hensel. Vom
porni de la interpretarea primei forme a Lemei lui Hensel ı̂n felul următor: dacă un
polinom este reductibil modulo p s, i unul din factori este de forma (X − α) astfel
ı̂ncât
f (X) ≡ (X − α)g(X) ( mod p),
atunci există o descompunere similară ı̂n Op [X]. Generalizare evidentă: să con-
siderăm descompuneri arbitrare. Condit, ia asupra derivatei spune, ı̂n esent, ă, că
rădăcina α nu este o rădăcină dublă, deci factorul al doilea, g(X), nu se divide
prin X − a. Pentru descompuneri generale, presupunerea trebuie să fie că factorii
sunt relativ primi (ca polinoame) modulo p. Mai precis:

Definit, ia 1. Fie g(X) s, i h(X) două polinoame ı̂n Op [X]. Spunem că g(X) s, i
h(X) sunt relativ prime modulo p dacă există polinoamele a(X), b(X) ∈ Op [X]
astfel ı̂ncât
a(X)g(X) + b(X)h(X) ≡ 1 ( mod p)
unde congruent, a este ı̂nt, eleasă coeficient cu coeficient, i.e. două polinoame sunt
congruente modulo p dacă fiecare coeficient al unuia este congruent modulo p cu
coeficientul corespunzător al celuilalt.

Teorema 2 (Lema lui Hensel, forma a doua). Fie f (X) ∈ Op [X] un polinom
s, i presupunem că există polinoamele g1 (X) s, i h1 (X) din Op [X] astfel ı̂ncât

i) g1 (X) este monic;

ii) g1 (X) s, i h1 (X) sunt relativ prime modulo p s, i

ii) f (X) ≡ g1 (X)h1 (X) (mod p) (ı̂nt, eles coeficient cu coeficient).

Atunci există polinoamele g(X),h(X) ∈ Op [X] astfel ı̂ncât

i) g(X) este monic;


Lema lui Hensel. Aplicat, ii 35

ii) g(X) ≡ g1 (X) (mod p) s, i h(X) ≡ h1 (X) (mod p) s, i

ii) f (X) = g(X)h(X).


Teorema 3 (Lema lui Hensel, a treia formă). Fie f (X) ∈ Op [X]. Dacă
există α0 ∈ Op s, i s ≥ 0 astfel ı̂ncât

f (α0 ) ≡ 0 ( mod p2s+1 )

f 0 (α0 ) ≡ 0 ( mod ps )
f 0 (α0 ) 6≡ 0 ( mod ps+1 )
atunci ∃ α ∈ Op astfel ı̂ncât f (α) = 0 s, i α ≡ α0 (mod ps+1 ).

Demonstraţie. Construim prin induct, ie s, irul α0 , α1 , . . . , αn , . . . definind

f (αn )
αn+1 = αn − , ∀n ≥ 0.
f 0 (αn )
Arătăm că ∀n ≥ 0 avem

αn ∈ Op , f (αn ) ≡ 0 ( mod p2s+n+1 )

s, i
αn ≡ αn−1 ( mod ps+n )
pentru n ≥ 1 (adevărat pentru n = 0).
i) Să presupunem n ≥ 0 s, i afirmat, iile verificate pentru respectivul număr.
Atunci
αn ≡ αn−1 ( mod ps+n ) ⇒ f 0 (αn ) ≡ f 0 (α0 ) ( mod ps+1 ),
adică f 0 (αn ) = ps · εn cu εn unitate ı̂n Op (deoarece f 0 (α0 ) = ps · ε). Atunci
conform relat, iei de recurent, ă ce defines, te s, irul, se obt, ine αn+1 ∈ Op s, i

αn+1 ≡ αn ( mod ps+n+1 ).

Din dezvoltarea polinomului f (X) după puterile lui X − αn rezultă, grupând tot, i
termenii de grad cel put, in doi, identitatea

f (X) = f (αn ) + f 0 (αn )(X − αn ) + (X − αn )2 · G(X),

cu G(X) ∈ Op [X]. Înlocuind X = αn+1 s, i având ı̂n vedere relat, ia de recurent, ă, se
obt, ine
f (αn ) 2
 
f (αn+1 ) = · G(αn+1 ),
f 0 (αn )
iar de aici f (αn+1 ) ≡ 0 (mod p2s+2n+2 ). Prin urmare

vp (αn+1 − αn ) → ∞ s, i vp (f (αn )) → ∞ când n → ∞.


36 Victor ALEXANDRU s, i Stelian Corneliu ANDRONESCU

Fie
α = lim αn ∈ Op .
n
Atunci
vp (f (α)) = ∞, adică f (α) = 0, iar α ≡ α0 ( mod ps+1 ).
Mai mult, demonstrăm că α este unic cu proprietăt, ile din enunt, , prin reducere
la absurd. Presupunem că f (X) ar mai avea o rădăcină β astfel ı̂ncât β ≡ α0
(mod ps+1 ). Deoarece f (β) = f (α) + f 0 (α)(β − α) + (β − α)2 G(β) rezultă că
f 0 (α)(α − β) = (α − β)2 G(β), de unde vp (f 0 (α)) ≥ vp (α − β) ≥ s + 1. Dar,
pe de altă parte, avem f 0 (α) = f 0 (α0 ) + f 0 (α) − f 0 (α0 ) , vp (f 0 (α0 )) = s s, i
vp (f 0 (α) − f 0 (α0 )) ≥ vp (α − α0 ) ≥ s + 1, deci vp (f 0 (α)) = s, contradict, ie.

Observat, ii. 1) Lema lui Hensel, considerată ı̂n ansamblul celor trei forme, stabiles, te
condit, ii suficiente (criterii) de reductibilitate pentru polinoamele din Op [X].
2) Demonstrat, ia formei a treia a Lemei lui Hensel este asemănătoare metodei
tangentei (sau metodei lui Newton) de aproximare a rădăcinilor polinoamelor din
R[X].
3) O consecint, ă a Lemei lui Hensel este următoarea: Fie F (X) ∈ Z[X] un
polinom cu rădăcini distincte (deci relativ prim cu F 0 (X) ı̂n Q[X]). Există un
exponent nF astfel ı̂ncât dacă congruent, a F (X) ≡ 0 (mod pnF ) are solut, ie ı̂n Z
atunci pentru orice n ≥ nF congruent, a F (X) ≡ 0 (mod pn ) are solut, ie iar s, irul
solut, iilor congruent, elor, (xn )n , converge ı̂n Op la o rădăcină x a polinomului F (X).
Propunem cititorului spre demonstrat, ie această consecint, ă.
Indicat, ie: Deoarece F (X) s, i F 0 (X) sunt relativ prime ı̂n Q[X] rezultă (din
algoritmul lui Euclid) că există A(X) s, i B(X) ∈ Z[X] astfel ı̂ncât F (X)A(X) +
F 0 (X)B(X) = C ∈ Z∗ .

Aplicat, ii

1. Descompunerea polinomului f = X p−1 − 1 ı̂n factori liniari ı̂n Op [X].

Demonstraţie. Fie f (X) = X p−1 − 1 ∈ Zp [X]. S, tim că

αp−1 = 1 ∀ α ∈ Z∗p = {1, 2, . . . , p − 1},

deci f are p − 1 rădăcini distincte ı̂n Zp . De fapt pentru orice a ∈ Z \ pZ, avem

f (a) = ap−1 − 1 ≡ 0 ( mod p)

dar
f 0 (a) = (p − 1)ap−2 6≡ 0 ( mod p)
adică vp (f (a)) = 1 s, i vp (f 0 (a)) = 0.
Lema lui Hensel. Aplicat, ii 37

Conform Lemei lui Hensel (prima formă), rezultă că f = X p−1 − 1 are p − 1
rădăcini distincte ı̂n Op .

2. Determinarea pătratelor perfecte din Op . Fie α = ps · ε ∈ O∗p s, i α = β 2 ,


β = pt · ε1 ∈ Op .
Atunci
α = ps · ε = β 2 = p2t · (ε1 )2 ⇒ s = 2t s, i ε = ε21 .
Fie ε = a0 + a1 p + . . . s, i 1 ≤ a0 ≤ p − 1.
2
Cazul 1: Fie p ≥ 3. Atunci ε ∈ O∗p ⇔ a0 este ,,rest pătratic modulo p”.

Demonstraţie. ,,⇒”: Implicat, ia este evidentă.


,,⇐”: Fie a0 un rest pătratic modulo p i.e. ∃b0 ∈ Z, (b0 , p) = 1 astfel ı̂ncât
a0 ≡ b20 (mod p).
Fie g = X 2 − ε ∈ Op [X|. Atunci g(b0 ) ≡ 0 (mod p) s, i g 0 (b0 ) = 2b0 6≡ 0 (mod p);
conform Lemei lui Hensel rezultă că g are o rădăcină ı̂n Op .

Cazul 2: p = 2. Atunci ε = a0 + a1 · 2 + . . . ∈ (O∗2 )2 ⇔ ε ≡ 1 (mod 8), adică


ε = 1 + a3 · 23 + a4 · 24 + . . ..

Demonstraţie. ,,⇒”: Rezultă din observat, ia simplă că a ∈ Z s, i (a, 2) = 1 ⇔


a = 2k + 1, deci a2 = (2k + 1)2 = 4k 2 + 4k + 1 = 4k(k + 1) + 1 ≡ 1 (mod 8), ce
trebuia demonstrat.
,,⇐”: Dacă ε ≡ 1 (mod 8) ı̂n O2 ⇒ ε = 1 + a3 · 23 + a4 · 24 + . . . ⇒ ∃b ∈ Z \ 2Z
cu ε ≡ b2 (mod 8) (pentru că ∀b ∈ Z \ 2Z, b2 ≡ 1 (mod 8)).
Fie g = X 2 − ε ∈ Z2 [X]. Atunci

g(b) = b2 − ε ≡ 0 ( mod 8),

g 0 (b) = 2b ≡ 0 ( mod 2)
s, i
g 0 (b) = 2b ≡ 2 ( mod 4).
Deci v2 (g(b)) ≥ 3 si v2 (g 0 (b)) = 1.
Atunci din Lema lui Hensel, forma a treia, rezultă că ∃β ∈ O2 astfel ı̂ncât

g(β) = β 2 − ε = 0 iar β ≡ b ( mod 4).


38 Victor ALEXANDRU s, i Stelian Corneliu ANDRONESCU

Folosind caracterizările anterioare se poate da un exemplu de polinom care are


rădăcini ı̂n orice Qp s, i ı̂n R, dar nu are rădăcini ı̂n Q:

f = (X 2 − 2)(X 2 − 17)(X 2 − 34).

Pentru demonstrat, ia completă se poate consulta [5].


Dacă ı̂nsă f = aX 2 + bX + c ∈ Q[X] are rădăcini reale s, i ı̂n orice corp p-adic
Qp , atunci f are rădăcini rat, ionale.
O serie de aplicat, ii ale numerelor p-adice ı̂n teoria numerelor se află de exemplu
ı̂n [5]. În ultimele decenii metodele p-adice ı̂s, i găsesc aplicat, ii notabile la problemele
de modelare ı̂n s, tiint, ele experimentale (a se vedea [7]).

Bibliografie

[1] V. Alexandru, N. Gos, oniu, Elemente de teoria numerelor, Ed. Univ. din
Bucures, ti, 1999.

[2] Y. Amice, Les nombres p-adiques, Presses Univ. de France, 1975.

[3] S.C. Andronescu, Numere p-adice. Aplicat, ii (I), RMGO, nr 1/2017, pg. 38–43.

[4] G. Bachman, Introduction to p-adic Numbers and Valuation Theory, Academic


Press, New York, 1964.

[5] Z.I. Borevici, I.R. Safarevici, Teoria numerelor, Ed. S, tiint, ifică s, i Enciclopedică,
Bucures, ti, 1985.

[6] G. Groza, A. Popescu, Extensions of valued fields, Ed. Academiei Române,


Bucures, ti, 2011.

[7] A. Khrennikov, Non-Archimedean Analysis: Quantum Paradoxes, Dynamical


Systems and Biological Models, Kluwer Academic Publishers, 1997.
CONCURSUL DE MATEMATICĂ
,,MARINESCU-GHEMECI OCTAVIAN”

Prezentarea Concursului Interjudet, ean de


Matematică ,,MARINESCU–GHEMECI
OCTAVIAN”, Edit, ia a VII-a,
Potcoava, 5 mai 2018

Florea BADEA 1

I.S.J. Olt s, i Liceul ,,S, tefan Diaconescu” din Potcoava au organizat ı̂n data de 5
mai 2018, Concursul Interjudet, ean de Matematică ,,Marinescu-Ghemeci Octavian”,
edit, ia a VII-a, care s-a adrest elevilor din clasele V-VIII s, i elevilor de liceu având
examenul de Bacalaureat de tip M2. Pres, edintele concursului a fost domnul conf.
univ. dr. Costel Bălcău de la Universitatea din Pites, ti.
La concurs au participat elevi din judet, ele Arges, s, i Olt.
Premiile au fost sponsorizate de firma VISTORIA LUX SRL, reprezentată de
directorul general ing. Victor Badea, iar doamna profesoară Ileana Marinescu-
Ghemeci a acordat un premiu special elevului cu cel mai mare punctaj din concurs.
Subiectele au fost selectate de o comisie formată din conf. univ. dr. Costel
Bălcău, prof. Costel Anghel, prof. Florea Badea s, i prof. Mihai Florea Dumitrescu.
Iată subiectele propuse ı̂n concurs:

Clasa a V-a
 
2 200 h 9 i 4
1. Fie numerele a = 41009 · 327:(81:3) · 25 · 29 − 216:4:2 · 228 ,
b = 2 · 22 · 23 · ... · 22018 s, i c = 21009 · (1 + 2 + 22 + 23 + ... + 22018 ) + 21009 .
Comparat, i cele trei numere date.
Mihai Florea Dumitrescu, Potcoava

1
Profesor, S, coala Gimnazială ,,Nicolae Coculescu”, Scornices, ti

39
40 Florea BADEA

2. Un elev a avut de rezolvat 10 probleme. Pentru fiecare problemă rezolvată


corect se acordă 10 puncte. Pentru o problemă rezolvată gres, it din primele 5
probleme se scad 3 puncte, iar pentru fiecare rezolvare gres, ită din ultimele 5
probleme se scad 2 puncte. S, tiind că elevul a obt, inut 62 de puncte aflat, i:

a) Câte probleme a rezolvat corect.


b) Câte probleme a rezolvat gres, it din primele 5 s, i câte din ultimele 5.
Florea Badea, Scornices, ti

3. Se as, ează cifrele 1,2,3,6,7 s, i 9 ı̂ntr-o ordine oarecare s, i se obt, ine numărul
n. Se as, ează apoi aceleas, i cifre ı̂ntr-o altă ordine obt, inându-se numărul m,
m 6= n. Să se arate că n nu se divide cu m.
Costel Anghel, Bircii

4. Se consideră un s, ir format din 100 de cartonas, e albe s, i 100 de cartonas, e ros, ii.
Să se arate că, pentru orice ordine a cartonas, elor, există 100 de cartonas, e
consecutive care sunt jumătate albe s, i jumătate ros, ii.
Stelian Corneliu Andronescu s, i Costel Bălcău, Pites, ti, Problema MGO 1

Clasa a VI-a

1. Pentru numerele naturale nenule n considerăm numerele


a = 2016n−1 + 2017n−1 + 2018n−1 s, i b = 1007n−1 + 1008n−1 + 1009n−1 .
a
a) Determinat, i numărul n pentru care este natural.
b
b) Arătat, i că există o infinitate de numere naturale n ≥ 2 astfel ı̂ncât b
este divizibil cu 1008.
Florea Badea, Scornices, ti

2 5
2. Aflat, i numerele naturale nenule x s, i y s, tiind că + = 1.
x y
Florea Badea, Scornices, ti

3. Se dă triunghiul obtuzunghic isoscel ABC cu AB = AC. Mediatoarele


laturilor [AB] s, i [AC] intersectează latura [BC] ı̂n punctele D, respectiv E.
S, tiind că CE = ED = DB, determinat, i măsura unghiului ^BAC.
Costel Anghel, Bircii

4. Fie ABC un triunghi echilateral. Arătat, i că ı̂n interiorul acestui triunghi
există un unic punct S astfel ı̂ncât ^ASB ≡ ^BSC ≡ ^CSA.
***, Problema MGO 10
Prezentarea Concursului MGO, Edit, ia a VII-a 41

Clasa a VII-a

1. Să se arate că numărul n2019 − n2017 + 2018 nu este rat, ional, unde n ∈ N.
Florea Badea, Scornices, ti

2. Fie numerele naturale nenule a s, i b. Dacă x = a2 + 8b s, i y = b2 − 8a, se cere:

a) Determinat, i două perechi (a, b) ∈ N∗ × N∗ astfel ı̂ncât x s, i y să fie


pătrate perfecte.
b) Demonstrat, i că există o infinitate de perechi (a, b) ∈ N∗ × N∗ pentru
care x s, i y să fie simultan pătrate perfecte.
Florea Badea, Scornices, ti

3. a) Fie ABC un triunghi dreptunghic isoscel cu m (^A) = 90◦ . O dreaptă


arbitrară intersectează interioarele catetelor (AB) s, i (AC) ı̂n M , res-
pectiv N s, i ı̂nălt, imea [AD], D ∈ BC ı̂n punctul P . Să se arate că:

1 1 2
+ = .
AM AN AP
b) Fie pătratul ABCD. Pe laturile (AB) , (BC) , (CD) , (DA) se iau punc-
tele E, F, G, respectiv H. Notăm AC ∩ EH = {M } , EF ∩ BD =
{N } , F G ∩ AC = {P }, GH ∩ BD = {Q}. Să se arate că:

1 1 1 1 8 2
+ + + ≥ .
AM BN CP DQ AB
Costel Anghel, Bircii

4. Fie M un punct ı̂n interiorul triunghiului echilateral ABC. Dacă P, Q s, i R


sunt picioarele perpendicularelor duse din M pe [AB] , [BC] respectiv [AC],
arătat, i că:

a) AP 2 + BQ2 + CR2 = BP 2 + CQ2 + AR2 .


b) AP · BQ + AP · CR + BQ · CR = BP · CQ + BP · AR + CQ · AR.
***, Problema MGO 15

Clasa a VIII-a
p √ p √ 2
1. Comparat, i numerele a = 6 + 4 2 − 5 + 2 6 s, i b = p √ +
8 + 2 15
1
p √ .
5+2 6
Mihai Florea Dumitrescu, Potcoava
42 Florea BADEA

2. a) Să se arate că nu există numere naturale a, b, c, d s, i n ∈ N∗ astfel ı̂ncât


d
a + b + c + d = 2018 s, i a + n = b − n = c · n = .
n
b) Să se găsească cel mai mare număr natural n s, i numerele naturale
d
a, b, c, d cu proprietatea : a+b+c+d = 2016 s, i a+n = b−n = c·n = .
n
Florea Badea, Scornices, ti

√ 2
3. Fie tetraedrul ABCD cu AB = 5, AC = 2, AB⊥AC, CD⊥AC, tg x = ,
3
unde x = m(^(AB, CD)). Aflat, i măsura unghiului diedru dintre planele
(ABC) s, i (BCD).
Mihai Florea Dumitrescu, Potcoava

4. Într-o piramidă patrulateră regulată distant, a de la centrul bazei la una din


fet, ele laterale este egală cu 10 dm, iar măsura unghiului diedru dintre bază
s, i o fat, ă laterală este egală cu 30◦ . Calculat, i:

a) Aria laterală s, i volumul piramidei.


b) Distant, a de la centrul bazei la ortocentrul unei fet, e laterale.
Florea Badea, Scornices, ti, adaptare după Problema MGO 17

Clasa a IX-a
   
x−1 x+2
1. Se consideră funct, ia f : R → R, f (x) = + , unde {a}
6 6
reprezintă partea fract, ionară a numărului real a.

a) Arătat, i că f (x + 6) = f (x), ∀x ∈ R;


x+2
b) Rezolvat, i ı̂n mult, imea numerelor reale ecuat, ia f (x + 2018) = .
6
2. Comparat, i numerele a s, i b, unde a = cos 80◦ + cos 81◦ + . . . + cos 100◦ s, i
b = cos 80 + cos 81 + . . . + cos 100.

3. Demonstrat, i că ı̂n orice triunghi ABC are loc relat, ia cos A sin B sin C +
sin2 A + sin2 B + sin2 C
sin A cos B sin C + sin A sin B cos C = .
2
4. (Legenda s, ahului, varianta oltenească) Banul Olteniei a ı̂nvăt, at jocul de s, ah
de la logofătul Ilie. Ei stabilesc următoarea modalitate de recompensare:
pe o tablă extinsă de s, ah, de dimensiune (2n + 1) × (2n + 1), unde n ∈ N∗
este fixat, se as, ează un bob de grâu pe un pătrăt, el (ales fie de ban, fie
de logofăt), apoi câte două boabe de grâu pe fiecare pătrăt, el ı̂nvecinat cu
acesta, apoi câte 4 boabe de grâu pe fiecare pătrăt, el ı̂nvecinat cu cel put, in
Prezentarea Concursului MGO, Edit, ia a VII-a 43

un pătrăt, el ocupat, apoi câte 8 boabe de grâu pe fiecare pătrăt, el ı̂nvecinat


cu cel put, in un pătrăt, el ocupat, s, .a.m.d., pănă când toate pătrăt, ele tablei
devin ocupate. Toate aceste boabe formează recompensa pe care banul i-o
acordă logofătului. Două pătrăt, ele se consideră vecine dacă frontierele lor
au cel put, in un punct comun (deci sunt vecine pe linie, pe coloană sau pe
diagonală), iar un pătrăt, el se consideră ocupat dacă el cont, ine cel put, in un
bob de grâu.

a) Dacă logofătul este cel care alege pătrăt, elul init, ial, care este, ı̂n funct, ie
de n, recompensa maximă (măsurată ı̂n număr de boabe) pe care el o
poate obt, ine?
b) Dacă banul este cel care alege pătrăt, elul init, ial, care este recompensa
minimă pe care el o acordă?
Stelian-Corneliu Andronescu s, i Costel Bălcău, Pites, ti

Clasa a X-a

m · 2x+1 − 4y = m

1. Se consideră sistemul de ecuat, ii , unde m este un
m · 2y+1 − 4x = m
parametru real.

a) Rezolvat, i sistemul pentru m = 1;


b) Determinat, i m ∈ R pentru care sistemul are solut, ie unică (x, y) ∈ R × R.

2. Fie mult, imea A = z ∈ C | z 4 − 15|z| + 14 = 0 .




a) Arătat, i că A cont, ine exact patru numere reale;


b) Demonstrat, i inegalitatea card (A \ R) ≥ card (A ∩ R).

3. Fie funct, ia f : R → R, f (x) = cos 2x − 2m cos x + m2 unde m ∈ Z.

a) Pentru m = 1, rezolvat, i ecuat, ia f (x) = 0;


b) Determinat, i m ∈ Z pentru care ecuat, ia f (x) = 0 are solut, ii.

4. Un mus, uroi cont, ine ı̂n prima zi doar două furnici. În ziua a doua el va cont, ine
de 10 ori mai multe furnici decât ı̂n prima zi, plus ı̂ncă două furnici. În ziua
a treia el va cont, ine de 10 ori mai multe furnici decât ı̂n ziua precedentă,
plus ı̂ncă trei furnici, s, .a.m.d.

a) În a câta zi numărul de furnici din mus, uroi va depăs, i un milion?


b) Fie n ∈ N∗ . Câte furnici va cont, ine mus, uroiul ı̂n a n-a zi?
Stelian-Corneliu Andronescu s, i Costel Bălcău, Pites, ti
44 Florea BADEA

Clasa a XI-a

1. Pentru orice matrice A, B ∈ Mn (R) (n ∈ {2, 3, 4}), se consideră mult, imea


S(A, B) = {X ∈ Mn (R) | AX = B, XB = A} .

a) Demonstrat, i că dacă S(A, B) 6= ∅, atunci A2 = B 2 ;


b) Arătat, i că reciproca afirmat, iei de la punctul a) nu este ı̂ntotdeauna
adevărată;
c) Demonstrat, i că dacă det(A) 6= 0, atunci reciproca afirmat, iei de la
punctul a) este adevărată.

2. Se consideră funct, ia f : N → N, f (x) = 2x2 + 3x + 4 s, i trei puncte distincte


A, B s, i C situate pe graficul funct, iei f .

a) Arătat, i că punctele A, B s, i C nu sunt coliniare;


b) Demonstrat, i că aria triunghiului ABC nu poate fi egală cu 2018.

3. a) Determinat, i asimptotele graficului funct, iei

ln(x + 1)
f : (−1, +∞) \ {0} → R, f (x) = ;
x

b) Comparat, i numerele 20182018 s, i 20192017 .

4. Ionel are curtea ı̂n forma unui trapez dreptunghic ABCD, cu ABkCD,
BC⊥AB, AB = 70 m, BC = 90 m, CD = 50 m. El dores, te să ı̂mpartă
această curte ı̂n trei triunghiuri prin intermediul unui gard ce unes, te fiecare
dintre punctele A s, i D cu un punct M situat pe latura BC. Calculat, i
lungimea minimă a unui astfel de gard.
Stelian-Corneliu Andronescu s, i Costel Bălcău, Pites, ti

Clasa a XII-a
  
a 5b
1. Se consideră mult, imea G = a, b ∈ Z, a2 − 10b2 = 1 .
2b a

a) Demonstrat, i că (G, ·) este grup abelian;


b) Arătat, i că mult, imea G este infinită.
x1 x2 x3
2. Fie polinomul f = X 3 + X − 1 s, i fie E = + + , unde x1 , x2 , x3 ∈ C
x2 x3 x1
sunt rădăcinile lui f .

a) Arătat, i că exact una dintre rădăcinile x1 , x2 , x3 este reală;


b) Calculat, i E + E, unde E reprezintă conjugatul numărului complex E;
Prezentarea Concursului MGO, Edit, ia a VII-a 45

c) Calculat, i E.
Z x

t
3. Se consideră funct, ia f : [0, +∞) → R, f (x) = dt.
0 t2 + 2t + 1

a) Calculat, i f (1);

b) Calculat, i lim x (f (2x) − f (x));
x→+∞
Z 1
c) Calculat, i f (x) dx.
0

4. Pentru a decupa o semilună dintr-o bucată de carton, Ionel procedează astfel:


desenează un triunghi dreptunghic isoscel ABC cu ipotenuza BC de 60
cm, determină centrul de greutate G al triunghiului ABC s, i mijlocul M al
segmentului [AG], apoi trasează două parabole, prima având vârful ı̂n A
s, i trecând prin punctele B s, i C, iar a doua având vârful ı̂n M s, i trecând
de asemenea prin punctele B s, i C, după care decupează suprafat, a cuprinsă
ı̂ntre cele două parabole. Calculat, i aria semilunii obt, inute.

Stelian-Corneliu Andronescu s, i Costel Bălcău, Pites, ti

Premiant, ii concursului au fost:


Clasa a V-a: Premiul I: Popa Luca (C.N. ,,Ion Minulescu”, Slatina); Premiul al
II-lea: Dinu Mihai (C.N. ,,Ion Minulescu”, Slatina) s, i Gı̂rnit, ă Laurent, iu Mihai (S, c.
Gimn. ,,Mihail Drumes, ”, Bals, ); Premiul al III-lea: Bibirică Andrei (C.N. ,,Ion
Minulescu”, Slatina) s, i Slăvoiu Adrian (S, c. Gimn. ,,Virgil Mazilescu”, Corabia).
Clasa a VI-a: Premiul I: Dumitrescu Lucian (C.N. ,,Ion Minulescu”, Slatina);
Premiul al II-lea: Stanciu Daria (S, c. Gimn. ,,Virgil Mazilescu”, Corabia); Premiul
al III-lea: Tudorache Robert (S, c. Gimn. ,,Constantin Brâncoveanu”, Slatina).
Clasa a VII-a: Premiul I: Militaru Marius (C.N. ,,Ion Minulescu”, Slatina);
Premiul al II-lea: Popa Filip (C.N. ,,Ion Minulescu”, Slatina); Premiul al III-lea:
Asaftei Alin Raul ( S, c. Gimn. Sines, ti).
Clasa a VIII-a: Premiul I: Cucu Iulia (S, c. Gimn. ,,Mihai Eminescu”, Pites, ti),
Premiul al II-lea: Iacob Victor (S, c. Gimn. ,,Mihai Eminescu”, Pites, ti), Premiul
al III-lea: Dinut, Mădălina Elena (Liceul Tehn. Tufeni).
Clasa a IX-a: Premiul al III-lea: Nicolaescu Andreea (L.T. ,,Mihai Viteazul”,
Caracal).
Clasa a X-a: Premiul I: Ivana Alin Mihail (Liceul ,,S, tefan Diaconescu”, Pot-
coava), Florescu Georgiana Iuliana (Liceul ,,S, tefan Diaconescu”, Potcoava); Pre-
miul al III-lea: Catana Larisa Georgiana (Liceul ,,S, tefan Diaconescu”, Potcoava).
Clasa a XI-a: Premiul al III-lea: Pavel Andrei (C.N. ,,Ion Minulescu”, Slatina).
46 Florea BADEA

Clasa a XII-a: Premiul I: Dumitrescu Valentin (C.N. ,,Radu Greceanu”, Sla-


tina); Premiul al II-lea: Velcea Ana Maria (Liceul ,,S, tefan Diaconescu”, Potcoava);
Premiul al III-lea: Floreanu Julia Mădălina (C.N. ,,Ion Minulescu”, Slatina).
Doamna profesoară Ileana Marinescu-Ghemeci a acordat Premiul Special Ma-
rinescu-Ghemeci Octavian elevului Dumitrescu Lucian din clasa a VI-a de la C.N.
,,Ion Minulescu”, Slatina, acesta obt, inând chiar punctajul maxim posibil.
Prezentarea Concursului Judet, ean de
Matematică ,,MARINESCU–GHEMECI
OCTAVIAN”, Edit, ia a VIII-a,
Potcoava, 11 mai 2019

Costel ANGHEL 1

Liceul ,,S, tefan Diaconescu” din Potcoava s, i I.S.J. Olt, reprezentat de doamna
profesoară Delia Ileana Basch-Naidin - inspector s, colar de matematică - au organizat
ı̂n data de 11 mai 2019, Concursul Judet, ean de Matematică ,,Marinescu- Ghemeci
Octavian”, edit, ia a VIII-a, care s-a adrest elevilor din clasele de gimnaziu s, i elevilor
de liceu având examenul de Bacalaureat de tip M2. Pres, edintele concursului a fost
domnul conf. univ. dr. Costel Bălcău.
La această edit, ie a concursului ne-am bucurat s, i de prezent, a a doi invitat, i
speciali, colaboratori de vază s, i de bază s, i ai revistei noastre, s, i anume domnii
profesori Leonard Giugiuc de la Colegiul Nat, ional ,,Traian” din Drobeta Turnu
Severin s, i Marin Ionescu de la Colegiul Nat, ional ,,I.C. Brătianu” din Pites, ti.
Suntem onorat, i de cuvintele domnului profesor Leonard Giugiuc, postate după
concurs pe site-uri ale comunităt, ii pasionat, ilor de matematică: ,,Sâmbătă, 11
mai 2019, a avut loc ı̂n oras, ul Potcoava din judet, ul Olt a opta edit, ie a concursu-
lui de Matematică ,,Marinescu-Ghemeci Octavian”, ı̂n memoria remarcabilului
Dascăl. Concursul constă ı̂n probă tip ONM pentru Gimnaziu s, i probă tip Finală
Haimovici-S, tiint, e pentru Liceu. Am avut onoarea să particip ı̂n calitate de invitat
special s, i totodată ca propunător de probleme, dar s, i ca evaluator la clasa a
XI-a. Organizatorii sunt: Costel Bălcău - Universitatea din Pites, ti, Florea Badea
- Scornices, ti, Costel Anghel - Slatina, Mihai Florea Dumitrescu - Potcoava. De
ment, ionat că domnii Bălcău s, i frat, ii Badea (fratele domnului profesor ment, ionat
mai sus fiind sponsorul principal) au fost mentorat, i de MGO. De asemenea, un
sponsor s, i un sprijin substant, ial a fost s, i este Primăria Oras, ului. Iată ce poate
face o comunitate aparent mică, dar unită s, i puternică!”
La concurs au participat 74 de elevi de la s, coli gimnaziale s, i licee din judet, ele
Arges, , Olt s, i Vâlcea.
S, i la această edit, ie, premiile au fost sponsorizate de firma VISTORIA LUX
SRL, reprezentată de directorul general ing. Victor Badea. De asemenea, doamna
1
Profesor, Colegiul Nat, ional ,,Ion Minulescu”, Slatina, anghelcostel2012@yahoo.com

47
48 Costel ANGHEL

profesoară Ileana Marinescu-Ghemeci a acordat din nou premii speciale elevilor


care au realizat cele mai mari punctaje din concurs.
Prezentăm ı̂n continuare subiectele propuse ı̂n concurs.

Clasa a V-a

2
1. Fie k un număr natural impar. Aflat, i restul ı̂mpărt, irii numărului 3k la 41.
Florea Badea, Scornices, ti s, i Costel Anghel, Slatina

2. Câte numere naturale de forma acbabc, scrise ı̂n baza 10, sunt divizibile cu
7?
Florea Badea, Scornices, ti s, i Costel Anghel, Slatina, Problema MGO 44

3. Se consideră numerele naturale nenule mai mici decât 2020.

(a) Există o aranjare a celor 2019 numere ı̂n grupe de câte trei astfel ı̂ncât
ı̂n fiecare grupă suma a două numere să fie cu 1 mai mică decât cel
de-al treilea număr?
(b) Există o aranjare a aceloras, i 2019 numere, tot ı̂n grupe de câte trei,
astfel ı̂ncât sumele numerelor din fiecare grupă să fie egale?
Florea Badea, Scornices, ti, Costel Anghel, Slatina s, i Costel Bălcău, Pites, ti

4. Un număr Dingo este un număr natural nenul n care poate fi scris simultan
sub formele
n = 16a + 12b = 20c + 10d,
cu a, b, c, d numere naturale mai mici sau egale cu 2019.

(a) Determinat, i cel mai mic număr Dingo.


(b) Câte numere Dingo există?
Stelian-Corneliu Andronescu s, i Costel Bălcău, Pites, ti

Clasa a VI-a

1. Determinat, i numerele naturale a s, i b, s, tiind că cel mai mare divizor comun
al lor este 1005, iar suma pătratelor lor este egală cu 10100250.
Marin Chirciu, Pites, ti, Problema MGO 49

2. Demonstrat, i că:
1 1 1 1
(a) + + + ... + < 1;
8 9 10 15
Prezentarea Concursului MGO, Edit, ia a VIII-a 49

1 2 3 2046
(b) 1 + 2
+ 2 + 2 + ... + < 11.
2 3 4 20472
Costel Anghel, Slatina s, i Florea Badea, Scornices, ti

3. Fie M mult, imea numerelor naturale formate din 2019 cifre, dintre care o
singură cifră este 6 s, i celelalte 2018 cifre sunt toate egale cu 1.

(a) Arătat, i că niciun număr din mult, imea M nu este divizibil cu 41.
(b) Câte numere din mult, imea M sunt divizibile cu 7?
Costel Anghel, Slatina s, i Florea Badea, Scornices, ti

4. Fie ABC un triunghi echilateral. În exteriorul triunghiului se construiesc


două semicercuri având diametrele [AB], respectiv [AC]. Pe semicercul de
diametru [AB] se ia un punct D, iar pe semicercul de diametru [AC] se ia
un punct E.

(a) Dacă punctele D, A, E sunt coliniare s, i dreapta DE este perpendiculară


pe ı̂nălt, imea triunghiului ABC dusă din A, arătat, i că DE = BC.
(b) Stabilit, i pozit, ia punctelor D s, i E pentru care lungimea segmentului
[DE] este maximă s, i calculat, i DE, dacă AB = 10 cm.
(c) În cazul ı̂n care lungimea segmentului [DE] este maximă, calculat, i
măsura unghiului ^DAC.
Costel Anghel, Slatina s, i Florea Badea, Scornices, ti

Clasa a VII-a

3
1. Fie n ∈ N, n ≥ 2 astfel ı̂ncât fract, ia este reductibilă. Arătat, i că
n3 −1
n3 + 8
∈ N.
9
Costel Anghel, Slatina s, i Florea Badea, Scornices, ti

2. (a) Rezolvat, i ı̂n N × N ecuat, ia x2 − y 2 = 4039.


(b) Demonstrat, i că oricum am alege 2019 pătrate perfecte, există două a
căror diferent, ă este divizibilă cu 4039.
Stelian-Corneliu Andronescu s, i Costel Bălcău, Pites, ti

3. Fie segmentul [AB] s, i punctul C ∈ (AB) astfel ı̂ncât AC > BC. De


aceeas, i parte a dreptei AB se construiesc pătratele ACDE s, i BCF G. Fie
AG ∩ CD = {M }.

(a) Arătat, i că punctele E, M, B sunt coliniare.


50 Costel ANGHEL

(b) Dacă DG k BE s, i AB = 3 + 5 cm, calculat, i lungimea segmentului
[BC].
Costel Anghel, Slatina s, i Florea Badea, Scornices, ti

n(n + 1)
4. Fie numerele reale a1 , a2 , . . . , an având suma egală cu . Dacă
p p p 2
(a1 + 1)2 + (a2 + 2)2 + (a2 + 2)2 + (a 2 +...+
 3 + 3)  (an+ n)2 + (a1 +1)2 ≤
√ 3 n+1
n(n + 1) 2, să se arate că a1 − 1 + 2 a2 − + . . . + n an − = 0.
2 2
Sorin Ulmeanu s, i Costel Bălcău, Pites, ti, Problema MGO 55

Clasa a VIII-a

1. Stabilit, i dacă numărul A = (10301 − 12 ) · (10300 − 22 ) · (10299 − 32 ) · . . . ·


(1 − 103012 ) este pozitiv, negativ sau nul.
Florea Badea, Scornices, ti s, i Costel Anghel, Slatina

1 1 1
2. Fie n ∈ N, n ≥ 2 s, i x1 , x2 , . . . , xn > 0 astfel ı̂ncât + + ... + = n.
x1 x2 xn
Arătăt, i că x21 x2 + x22 x3 + . . . + x2n−1 xn + x2n x1 ≥ 2(x1 + x2 + . . . + xn ) − n.
Marin Ionescu, Pites, ti, Problema MGO 59

3. Două piramide regulate au aceeas, i bază ABCD, vârfurile V s, i S, cu V 6= S,


toate muchiile laterale sunt congruente cu muchia bazei s, i AB = 12 cm.

(a) Demonstrat, i că (V AD) k (SBC).


(b) Calculat, i distant, a dintre planele de la punctul a).
(c) Calculat, i distant, a dintre dreptele CD s, i SA.
Florea Badea, Scornices, ti s, i Costel Anghel, Slatina

4. Se consideră s, irul 1, 8, 15, 22, . . ..

(a) Calculat, i suma a cinci termeni ai acestui s, ir, cei mai mici, care sunt
pătrate perfecte.
(b) Calculat, i suma a 2n + 1 termeni ai s, irului, pătrate perfecte, cei mai
mici.
Mihai Florea Dumitrescu, Potcoava

Clasa a IX-a

1. Rezolvat, i ı̂n mult, imea numerelor reale ecuat, iile:

(a) |x − 1| + |x − 2| = x;
(b) |x − 1| + |x − 2| + . . . + |x − 2019| = x.
Prezentarea Concursului MGO, Edit, ia a VIII-a 51

2. Fie x1 s, i x2 rădăcinile ecuat, iei x2 − x − 3 = 0. Notăm Sn = xn1 + xn2 , pentru


orice n ∈ N∗ .

(a) Arătat, i că S3 = S2 + 3S1 .


(b) Demonstrat, i că există n ∈ N∗ astfel ı̂ncât Sn ≥ 2019.
(c) Demonstrat, i că nu există n ∈ N∗ astfel ı̂ncât Sn = 2019.

3. Calculat, i suma S = [sin 2019◦ ] + [cos 2019◦ ] + [tg 2019◦ ] + [ctg 2019◦ ], unde
[x] reprezintă partea ı̂ntreagă a numărului real x.

4. Dorel are 110 centimetri de sârmă din care dores, te să construiască un triunghi
echilateral s, i un pătrat astfel ı̂ncât aria totală a acestora să fie cât mai mică.
Cele două figuri nu au port, iuni comune, iar sârma trebuie să fie utilizată ı̂n
totalitate.

(a) Comparat, i ariile totale obt, inute ı̂n următoarele două variante particu-
lare:
V1) latura triunghiului este de 18 cm;
V2) latura triunghiului este de 22 cm.
(b) Determinat, i lungimile laturilor triunghiului s, i pătratului pentru care
aria totală este minimă.
(c) Rezolvat, i aceeas, i cerint, ă ca la punctul anterior, ı̂n ipoteza suplimentară
că ambele figuri au laturile exprimate prin numere ı̂ntregi de centimetri.
Stelian-Corneliu Andronescu, Pites, ti, Costel Bălcău, Pites, ti s, i
Leonard Giugiuc, Drobeta Turnu Severin

Clasa a X-a


3

3
1. În mult, imea numerelor reale se consideră ecuat, ia x + 30 + 31 − x = m,
unde m este un parametru real.

(a) Rezolvat, i ecuat, ia pentru m = 1.


(b) Determinat, i m ∈ R pentru care ecuat, ia are solut, ie unică.

2. Pentru orice n ∈ N, n ≥ 2, definim numărul

an = log23 32 · log34 43 · . . . · lognn+1 (n + 1)n .

(a) Arătat, i că a3 = 1.


(b) Arătat, i că numărul a2019 este irat, ional s, i calculat, i primele două cifre
după virgulă din scrierea sa sub formă de fract, ie zecimală infinită.

3. (a) Rezolvat, i ı̂n mult, imea numerelor complexe ecuat, ia (z + i)2 = z − i.


52 Costel ANGHEL

(b) Determinat, i câte solut, ii complexe are ecuat, ia (z + i)2019 = z − i.

4. Profesorul de sport al unei clase dores, te să organizeze o sesiune de s, ah astfel


ı̂ncât fiecare dintre cei n elevi ai clasei să joace exact câte k partide, unde n
s, i k sunt numere naturale nenule, n > k. Între oricare doi elevi nu se poate
disputa mai mult de o partidă.

(a) Este posibilă organizarea unei astfel de sesiuni pentru n = 21 s, i k = 2?


(b) Dar pentru n = 21 s, i k = 3?
(c) Pentru n fixat, determinat, i valorile lui k pentru care se poate organiza
o astfel de sesiune.
Stelian-Corneliu Andronescu, Pites, ti, Costel Bălcău, Pites, ti s, i
Leonard Giugiuc, Drobeta Turnu Severin

Clasa a XI-a
  
 a b c a b c a b c
1. Fie mult, imea M =  d e f  ∈ M3 (R∗ ) = = s, i = = .
d e f g h i
g h i

 
a b c
(a) Arătat, i că dacă A ∈ M , A =  d e f , atunci A2 = (a + e + i)A.
g h i
 
9 9 9
(b) Rezolvat, i ı̂n mult, imea M ecuat, ia X 3 =  9 9 9 .
9 9 9
2. Fie ABC un triunghi nedreptunghic, Q centrul cercului circumscris acestui
triunghi s, i A1 , B1 s, i C1 centrele de greutate ale triunghiurilor QBC, QAC,
respectiv QAB.

(a) S, tiind că, ı̂n sistemul cartezian de coordonate xOy, avem A(−1, 2),
B(0, −1) s, i C(3, 2), calculat, i coordonatele punctului Q s, i arătat, i că
punctele A1 , B1 , C1 nu sunt coliniare.
(b) Demonstrat, i că punctul Q este centrul de greutate al triunghiului
A1 B1 C1 dacă s, i numai dacă triunghiul ABC este echilateral.
 √


2019
ax, dacă x ≤ 1
3. Fie funct, ia f : R → R, f (x) = x 2019 −1 , unde a ∈ R.

 , dacă x > 1
x−1
(a) Determinat, i a ∈ R pentru care funct, ia f este continuă.
(b) Determinat, i a ∈ R pentru care funct, ia f este injectivă.
Prezentarea Concursului MGO, Edit, ia a VIII-a 53

(c) Determinat, i a ∈ R pentru care funct, ia f este surjectivă.

4. Doi colegi de bancă, Dorel s, i Gigel, scriu ı̂n fiecare zi câte un număr. În
prima zi Dorel a scris numărul 2, iar Ionel numărul 3. Începând cu ziua a
doua, fiecare scrie suma dintre triplul numărului său din ziua precedentă s, i
numărul celuilalt din ziua precedentă.

(a) Comparat, i numerele scrise de cei doi colegi ı̂n cea de-a s, asea zi.
(b) În a câta zi numerele scrise de cei doi colegi sunt ambele mai mari decât
zece milioane?
Stelian-Corneliu Andronescu, Pites, ti, Costel Bălcău, Pites, ti s, i
Leonard Giugiuc, Drobeta Turnu Severin

Clasa a XII-a
 
3 b
b 4
1. Fie matricea A = ∈ M2 (Z5 ).
0 b
b 2

(a) Calculat, i A2019 .


(b) Rezolvat, i ı̂n mult, imea M2 (Z5 ) ecuat, ia X 3 = A.

2. Se consideră polinoamele f = X 3 + X + 1 s, i g = X 3 + X 2 + 1.

(a) Arătat, i că polinoamele f s, i g au fiecare exact câte o rădăcină reală.


(b) Demonstrat, i că polinoamele f s, i g nu au rădăcini comune.
(c) Determinat, i numărul de valori reale pe care le poate lua expresia
E = x1 y1 + x2 y2 + x3 y3 , unde x1 , x2 , x3 ∈ C sunt rădăcinile lui f iar
y1 , y2 , y3 ∈ C sunt rădăcinile lui g.

 ln(1√+ x)
, dacă x > 0
3. Fie funct, ia f : [0, ∞) → R, f (x) = x .
0, dacă x = 0

(a) Demonstrat, i că funct, ia f admite primitive.


(b) Determinat, i primitiva F a funct, iei f pentru care axa Ox este tangentă
ı̂n origine la graficul funct, iei F .

4. Doi prieteni, Dorel s, i Gigel, au fiecare câte o mas, inut, ă teleghidată. Mas, inut, a
lui Dorel este programată să se deplaseze ı̂n linie dreaptă cu viteza instantanee
v1 (x) = sin2 (πx) m/s, iar mas, inut, a lui Gigel cu viteza instantanee v2 (x) =
sin2 (2πx) m/s.

(a) Ce distant, ă a parcurs fiecare mas, inut, ă după primele 20 de secunde?


(b) Cine câs, tigă o cursă de 10,8 metri?
54 Costel ANGHEL

Se cunoas, te că dacă S(x) este distant, a parcursă de la momentul init, ial
t = 0 la momentul t = x de un mobil având viteza instantanee v(x), atunci
v(x) = S 0 (x). Cursa ı̂ncepe la momentul init, ial t = 0 s, i traseul ales este ı̂n
linie dreaptă.
Stelian-Corneliu Andronescu, Pites, ti, Costel Bălcău, Pites, ti s, i
Leonard Giugiuc, Drobeta Turnu Severin

Premiant, ii concursului au fost următorii:


Clasa V-a: Premiul I: Mazăre Emanuel (S, c. Gimn. ,,Mihai Eminescu”, Pites, ti);
Premiul al II-lea: Mateescu Matei (C.N. ,,Ion Minulescu”, Slatina) - elev ı̂n clasa
a IV-a!; Premiul al III-lea: Ciobănică Alexandru (S, c. Gimn. ,,Virgil Mazilescu”,
Corabia).
Clasa a VI-a: Premiul I: Bilauschi Simina (S, c. Gimn. Nr. 3 Slatina); Premiul
al II-lea: Popa Luca (C.N. ,,Ion Minulescu”, Slatina); Premiul al III-lea: S, tefan
Alexandru (C.N. ,,Ion Minulescu”, Slatina).
Clasa a VII-a: Premiul I: Dumitrescu Lucian (C.N. ,,Ion Minulescu”, Slatina);
Premiul al II-lea: Tudorache Robert (S, c. Gimn. ,,Constantin Brâncoveanu”,
Slatina); Premiul al III-lea: Vintilescu Oana (C.N. ,,Ion Minulescu”, Slatina).
Clasa a VIII-a: Premiul I: Militaru Marius (C.N. ,,Ion Minulescu”, Slatina);
Premiul al II-lea: Popa Filip (C.N. ,,Ion Minulescu”, Slatina); Premiul al III-lea:
Asaftei Alin Raul ( S, c. Gimn. Sines, ti).
Clasa a IX-a: Premiul I: Firfirică Mihai Călin (C.N. ,,Zinca Golescu”, Pites, ti);
Premiul al II-lea: Dimian Rares, Nicolae (C.N. ,,Zinca Golescu”, Pites, ti); Premiul
al III-lea: Negoescu Andreea Denisa (C.N. ,,Zinca Golescu”, Pites, ti).
Clasa a X-a: Premiul I: Smarandache Maria (L.T. ,,Mihai Viteazul”, Caracal);
Premiul al II-lea: Catrina Alexandra (Liceul Tehn. ,,Constantin Brâncoveanu”,
Scornices, ti); Premiul al III-lea: Călina Oana Denisa (Liceul Tehn. ,,Constantin
Brâncoveanu”, Scornices, ti).
Clasa a XI-a: Premiul I: Florescu Georgiana Iuliana (Liceul ,,S, tefan Dia-
conescu”, Potcoava); Premiul al II-lea: Nicolae Sara Elisabeth (Liceul ,,S, tefan
Diaconescu”, Potcoava); Premiul al III-lea: Zamos, teanu David (C.N. ,,Radu
Greceanu”, Slatina).
Clasa a XII-a: Premiul I: Pavel Andrei (C.N. ,,Ion Minulescu”, Slatina);
Premiul al II-lea: Ispir Dragos, Ionut, (C.E. Râmnicu Vâlcea); Premiul al III-lea:
Vlad Iuliana (Liceul Tehn. ,,Constantin Brâncoveanu”, Scornices, ti).
Doamna profesoară Ileana Marinescu-Ghemeci a acordat Premiul Special Ma-
rinescu-Ghemeci Octavian elevilor Bilauschi Simina (S, c. Gimn. Nr. 3 Slatina),
Mazăre Emanuel (S, c. Gimn. ,,Mihai Eminescu”, Pites, ti), Mateescu Matei (C.N.
,,Ion Minulescu”, Slatina) s, i Florescu Georgiana Iuliana (Liceul ,.S, tefan Diacones-
cu”, Potcoava).
TESTE PENTRU EXAMENE

Teste pentru examenul de Evaluare Nat, ională

Costel ANGHEL 1 s, i Florea BADEA 2

Testul 1

SUBIECTUL I

1. Rezultatul calculului 3, 5 + 7 : 2 este . . ..

2. Solut, ia ecuat, iei x − 5 = 2x − 7 este x = . . ..

3. Volumul unui cub cu muchia de 4 m este . . . m3 .

4. Numerele naturale de forma 1x9 divizibile cu 3 sunt . . ..



5. Un pătrat cu diagonala de 11 2 cm are latura egală cu . . . cm.

6. Volumul unui con circular drept cu raza de 2,5 cm s, i ı̂nălt, imea de 4 cm este
. . . cm3 .

SUBIECTUL al II-lea

1. Desenat, i un pentagon ABCDE.

2. Rezolvat, i ı̂n R inecuat, ia |x − 7| · (x − 7) ≤ 0.

3. Rezolvat, i ı̂n Q ecuat, ia ||5x − 1| − 5| = 1.

4. Se dă funct, ia f : R → R, f (x) = 6x − 4.

a) Reprezentat, i grafic funct, ia f .


b) Aflat, i aria triunghiului determinat de originea sistemului de axe de
coordonate s, i punctele de intersect, ie ale graficului cu axele.
c) Stabilit, i dacă punctele A(1; 2), B(−1; 5) s, i C(3; 14) sunt coliniare.
1
Profesor, Colegiul Nat, ional ,,Ion Minulescu”, Slatina, anghelcostel2012@yahoo.com
2
Profesor, S, coala Gimnazială ,,Nicolae Coculescu”, Scornices, ti

55
56 Costel ANGHEL s, i Florea BADEA

SUBIECTUL al III-lea

1. Pe cercul de centru O s, i rază R se consideră punctele A s, i B astfel ı̂ncât


m(^AOB) = 60◦ . S, tiind că AB = 10 cm să se afle:

a) lungimea razei cercului;


b) aria discului;
c) aria triunghiului AOB.

2. Un con circular drept are raza bazei de 12 cm. Desfăs, urarea suprafet, ei
laterale a conului este un sector de cerc corespunzător unui arc de cerc cu
măsura de 240◦ . Se cer:

a) lungimea generatoarei conului;


b) lungimea razei cercului circumscris sect, iunii axiale a conului;
c) volumul conului.

Testul 2

SUBIECTUL I

1. Rezultatul calculului 3,5 + 5 : 2 este . . ..

2. Perimetrul triunghiului echilateral cu latura de 3,5 cm este . . ..

3. Scriet, i două numere naturale de forma 5a3 nedivizibile cu 3 . . ..

4. Volumul cilindrului circular drept cu raza de 5 cm s, i ı̂nălt, imea de 8 cm este


. . . cm3 .

5. Media geometrică a numerelor 4 s, i 9 este . . ..



6. Un cub cu diagonala de 8 3 m are volumul de . . . m3 .

SUBIECTUL al II-lea

1. Desenat, i o piramidă hexagonală V ABCDEF .

2. Se consideră mult, imea A = {1, 5, 9, 13, . . .} s, i card A = 200.

a) Aflat, i două elemente ale lui A, care apart, in intervalului (401; 601].
b) Aflat, i cel mai mare element al mult, imii A.
c) Calculat, i suma elementelor mult, imii A.
√ √ √ √
3. Se consideră numerele reale a = 2 5 + 2 s, i b = 2 5 − 2.
Teste pentru examenul de Evaluare Nat, ională 57

a) Calculat, i produsul numerelor a s, i b.


a
b) Aflat, i numărul natural n astfel ı̂ncât ∈ (n; n + 1).
b

SUBIECTUL al III-lea

1. Fie [AB] un segment, AB = 12 cm s, i O mijlocul său. Pe semicercul


determinat de diametrul [AB] se consideră punctele C s, i D astfel ı̂ncât
_ _ _
AD≡DC≡CB. Determinat, i:

a) natura s, i perimetrul triunghiului COD;


b) natura s, i aria patrulaterului OBCD;
c) natura s, i aria patrulaterului ABCD.

2. Se dă un cilindru circular drept cu generatoarea de 10 cm s, i diametrul bazei


de 12 cm, iar punctele O s, i O0 sunt centrele bazelor. Dacă ABB 0 A0 este o
sect, iune axială a cilindrului, punctele A s, i B apart, inând cercului de centru
O, aflat, i:

a) volumul cilindrului;
b) aria totală a cilindrului;
c) valoarea tangentei unghiului dintre dreapta AO0 s, i planul unei baze a
cilindrului dat.

Testul 3

SUBIECTUL I

1. Rezultatul calculului 6 · (−2)−1 + 13 este egal cu . . ..


x 12
2. Dacă = , y 6= 0, atunci x · y este egal cu . . ..
6 y

3. Aria unui pătrat cu latura de 3 5 m este . . . m2 .

4. Volumul unui con cu raza bazei de 5 dm s, i ı̂nălt, imea de 100 cm este egal cu
. . . dm3 .

5. Produsul numerelor ı̂ntregi din intervalul [−15; 1) este . . ..


√ √
6. Media geometrică a numerelor 5 s, i 20 5 este . . ..

SUBIECTUL al II-lea

1. Desenat, i un trunchi de con.


58 Costel ANGHEL s, i Florea BADEA

15
2. Aflat, i numerele ı̂ntregi x s, tiind că ∈ N.
x+5
3. Suma a două numere este 350. Aflat, i cele două numere s, tiind că o treime
din primul număr este egală cu o pătrime din al doilea număr.
1
4. Dacă x − = 10, x 6= 0, calculat, i:
x
1
a) x2 + .
x2
1
b) x4 + 4 .
x
5. Determinat, i numerele reale x, s, tiind că expresia

x2 − 1 x2 − 4
E(x) = + + |2x − 3|
x+1 x+2
este constantă, oricare ar fi x ∈ R \ {−1; −2}.

SUBIECTUL al III-lea

1. Se consideră cercurile cu centrele O1 , O2 , O3 , razele de 4 cm, 6 cm respectiv


8 cm s, i tangente exterioare, două câte două. Notăm cu T1 , T2 , T3 cele trei
puncte de tangent, ă. Calculat, i:

a) suma ariilor celor trei cercuri (discuri);


b) aria triunghiului O1 O2 O3 ;
c) aria triunghiului T1 T2 T3 .

2. O sferă, cu raza R = 5 dm, este sect, ionată cu un plan situat la distant, a de 3


dm fat, ă de centrul său. Calculat, i:

a) aria s, i volumul sferei;


b) aria sect, iunii determinate de planul de sect, iune cu sfera;
c) volumul tetraedrului regulat ı̂nscris ı̂n sferă.
Teste pentru examenul de Bacalaureat,
specializarea s, tiint, e ale naturii

Mihai Florea DUMITRESCU 1

Testul 1

SUBIECTUL I

1. Să se determine numerele reale a s, i b, s, tiind că numerele 4, a s, i b sunt ı̂n


progresie aritmetică, iar media aritmetică a numerelor a s, i b este 7.

2. Să se determine punctele de intersect, ie dintre parabola y = x2 + 7x − 8 s, i


dreapta y = 5x − 5.

3. Să se rezolve ı̂n mult, imea numerelor reale ecuat, ia logx (x + 5 x − 5) = 1.

4. Determinat, i probabilitatea ca, alegând o submult, ime cu două elemente din


mult, imea {1, 2, 3, 4, 5}, aceasta să aibă suma elementelor egală cu 6.

5. Să se determine numărul real m, astfel ı̂ncât dreptele y = (m + 1)x + 3 s, i


y = −2x + 7 să fie paralele.
1
6. Rezolvat, i ecuat, ia sin 2x = − ı̂n intervalul (0, π).
2

SUBIECTUL al II-lea

x2 2
 
1. Se consideră matricea A(x) = , unde x este un număr real.
8 x2

a) Calculat, i det A (1).


b) Rezolvat, i ı̂n mult, imea numerelor reale ecuat, ia det A (x) = det A (5).
c) Aflat, i inversa matricei A(−1).

2. Fie legea de compozit, ie x ∗ y = axy − 3x − 3y + 12, ∀x, y ∈ R, unde a ∈ R.


1
Profesor, Liceul ,,S, tefan Diaconescu”, Potcoava, florin14mihai@yahoo.com

59
60 Mihai Florea DUMITRESCU

a) Aflat, i numărul real a, astfel ı̂ncât legea ,,∗” să fie asociativă.
b) Pentru a = 1, arătat, i că x∗y = (x − 3) (y − 3)+3, pentru orice x, y ∈ R.
c) Pentru a = 1, aflat, i x ∈ R pentru care x · x0 = e, unde e este elementul
neutru al legii ,,∗”, iar x0 este simetricul lui x ı̂n raport cu legea ,,∗”.

SUBIECTUL al III-lea

x+1
1. Se consideră funct, ia f : R → R, f (x) = .
ex
a) Calculat, i f 0 (x), x ∈ R.
b) Determinat, i asimptota orizontală spre +∞ la graficul funct, iei f.
3
c) Arătat, i că f (x) ≤ 2 , pentru orice x ∈ [2, +∞).
e
x
2. Se consideră funct, ia f : R → R, f (x) = x .
e
Z 1
f (x)
a) Calculat, i dx.
0 x
Z 1
b) Calculat, i f (x)f 0 (x) dx.
0
c) Să se afle volumul corpului de rotat, ie obt, inut prin rotirea ı̂n jurul axei
Ox a graficului funct, iei g : [1, 2] → R, g(x) = f (x).

Testul 2

SUBIECTUL I

1. Determinat, i numărul real x, pentru care numerele x + 2, x + 6 s, i x + 14 sunt


termeni consecutivi ai unei progresii geometrice.

2. Determinat, i coordonatele vârfului parabolei asociate funct, iei

f : R → R, f (x) = x2 − 4x + 1.

3. Rezolvat, i ı̂n mult, imea numerelor reale ecuat, ia 2x+1 = 9x+2 .

4. Calculat, i probabilitatea ca, alegând un număr din mult, imea {1, 2, 3, . . . , 50},
acesta să aibă divizor pe 6.

5. Determinat, i a ∈ R, s, tiind că vectorii →



u = 7~i + ~j s, i →

v = (a + 1)~i + 4~j sunt
perpendiculari.

6. Calculat, i raza cercului circumscris triunghiului ABC, s, tiind că AB = 6 cm


1
s, i cos C = − .
2
Teste pentru examenul de Bacalaureat, specializarea s, tiint, e ale naturii 61

SUBIECTUL al II-lea
 
1 2x 2x
1. Se consideră matricea A(x) =  −x 1 2x , unde x ∈ R.
−x −x 1

a) Calculat, i det A(1).


b) Aflat, i x ∈ R, pentru care 3 det A (x) = 5tr A (x).
c) Arătat, i că A (x) · A (−x) = I3 dacă s, i numai dacă x = 0.

2. Se consideră inelul (Z12 , +, ·).

a) Determinat, i suma pătratelor elementelor inversabile ale inelului.


b) Rezolvat, i ı̂n Z12 ecuat, ia b
7x + b
7=b
3.

3x + b
b 5y = b
4
c) Rezolvat, i ı̂n Z12 × Z12 sistemul de ecuat, ii .
x + 4y = 8
b b

SUBIECTUL al III-lea
ln (x + 1)
1. Se consideră funct, ia f : (−1, +∞) → R, f (x) = .
x+1
f (x) − f (1)
a) Calculat, i lim .
x→1 x−1
b) Determinat, i ecuat, ia asimptotei verticale la graficul funct, iei f.
c) Arătat, i că e ln (x + 1) ≤ x + 1, pentru orice x ∈ (−1, +∞).

2. Se consideră funct, ia f : R → R, f (x) = x x2 + 1.
Z 2
x
a) Calculat, i dx.
1 f (x)
Z 1
xf 0 (x) + f (x) dx.

b) Calculat, i
0
Z x
f (t) dt
0
c) Calculat, i lim .
x→∞ x3 + 1

Testul 3

SUBIECTUL I

1. Se consideră numărul complex z = 2 − 4i. Arătat, i că z 2 + (z̄)2 = −24.

2. Rezolvat, i ı̂n mult, imea numerelor ı̂ntregi inecuat, ia −x2 + 3x − 2 ≥ 0.

3. Rezolvat, i ı̂n mult, imea numerelor reale ecuat, ia (2x + 1)2 − 6 (2x + 1) + 8 = 0.
62 Mihai Florea DUMITRESCU

4. Calculat, i probabilitatea ca, alegând la ı̂ntâmplare un număr din mult, imea


{10, 11, 12, . . . , 70}, acesta să aibă rădăcina pătrată un număr rat, ional.
5. În reperul cartezian xOy se consideră dreptele d1 : x + y + 1 = 0,
d2 : x − y + a = 0 s, i d3 : 2x + y + 4 = 0, unde a este un număr real.
Determinat, i a ∈ R pentru care dreptele d1 , d2 s, i d3 sunt concurente.
6. Triunghiul ABC are AB = 9, AC = 12 s, i cos A = 0. Aflat, i lungimea razei
cercului circumscris triunghiului ABC.

SUBIECTUL al II-lea
   
x 0 2x 0 x 0
1. Fie matricele A(x) =  0 x 0  s, i B(x) =  −x 0 −x , unde
2x 0 x 0 x 0
x ∈ R.
a) Calculat, i det [A (1) + B (−1)].
b) Arătat, i că A(x) · B(x) + B(x) · A(x) = 4x2 · B(1), pentru orice x ∈ R.
c) Rezolvat, i ecuat, ia det [A (x) + B (x) + I3 ] = 0, x ∈ R.
2. Se consideră polinomul f = X 3 − 4X 2 − 3X + 2, cu rădăcinile x1 , x2 , x3 .
a) Calculat, i câtul s, i restul ı̂mpărt, irii polinomului f la polinomul 2X − 1.
b) Arătat, i că polinomul f nu are toate rădăcinile rat, ionale.
c) Calculat, i x41 + x42 + x43 .

SUBIECTUL al III-lea

1. Se consideră funct, ia f : R → R, f (x) = x5 − 5x3 + 10x.


f 0 (x) − f 0 (1)
a) Calculat, i lim .
x→1 x−1
b) Stabilit, i intervalele de monotonie pentru funct, ia f .
"√ !
6
c) Arătat, i că funct, ia f0 este strict crescătoare pe intervalul , +∞ .
2

2. Se consideră funct, ia f : R → R, f (x) = sin x.


Z π
a) Calculati 2 f (x) dx.
,
0
Z π
b) Calculat, i 2 f 2 (x) dx.
0
Z 1
1
c) Arătat, i că f 4 (x) dx ≤ .
0 5
Teste pentru examenul de Bacalaureat,
specializarea matematică-informatică

Costel BĂLCĂU 1

Testul 1

SUBIECTUL I

1. Fie (an )n≥1 o progresie geometrică. S, tiind că a4 = 4, calculat, i a1 a7 + a2 a6 +


a3 a5 .

2. Se consideră funct, ia f : R → R, f (x) = 2x + m, unde m ∈ R. Determinat, i


numărul real m, s, tiind că f (1) + f (2) + . . . + f (10) = 21m.

3. Rezolvat, i ı̂n mult, imea numerelor reale ecuat, ia 2 log25 x − 3 log5 (x2 ) = 8.

4. Calculat
√, i probabilitatea
√ √ √ ca, alegând la ı̂ntâmplare un număr din mult, imea
A= 0, 1, 2, . . . , 2019 , acesta să fie pătratul unui număr natural.

5. Determinat, i numărul real m pentru care vectorul →−u = 2~i + m~j formează cu

− ~ ~
vectorul v = −i + j un unghi având măsura de 135◦ .

3 6 π π
6. Fie triunghiul ABC cu AB = ,A= s, i B = . Calculat, i lungimea
2 12 4
laturii AC.

SUBIECTUL al II-lea

1 a 2
1. Se consideră matricea A(a) =  a 2 1  s, i sistemul de ecuat, ii
1 0 −1

x + ay + 2z = 8

ax + 2y + z = 10 , unde a este un număr real.

x − z = −4

a) Arătat, i că det(A(1)) = −4.


1
Conf. univ. dr., Universitatea din Pites, ti, cbalcau@yahoo.com

63
64 Costel BĂLCĂU

b) Determinat, i valorile reale ale lui a pentru care sistemul este compatibil
determinat.
c) Determinat, i numărul real a pentru care sistemul are o solut, ie (x0 , y0 , z0 )
cu proprietatea că x0 , y0 s, i z0 sunt, ı̂n această ordine, termeni consecutivi
ai unei progresii aritmetice.

2. Se consideră m, n ∈ Z s, i polinomul f = X 3 − mX 2 − X + n, având rădăcinile


x1 , x2 , x3 ∈ C.

a) Pentru m = n = 1, calculat, i restul ı̂mpărt, irii polinomului f la polinomul


X + 2.
b) Determinat, i numărul ı̂ntreg m, pentru care are loc egalitatea
x21 + x22 + x23 = 2.
c) Pentru m = 0, determinat, i numărul ı̂ntreg n pentru care rădăcinile x1 ,
x2 s, i x3 sunt numere ı̂ntregi.

SUBIECTUL al III-lea

x2 + 4x − 4
1. Se consideră funct, ia f : R → R, f (x) = .
ex
(2 − x)(x + 4)
a) Arătat, i că f 0 (x) = , x ∈ R.
ex
b) Demonstrat, i că tangenta dusă prin punctul A(0, −4) la graficul funct, iei
1
f este perpendiculară pe dreapta de ecuat, ie y = − x + 2019.
8
c) Determinat, i m ∈ R pentru care ecuat, ia f (x) = m are exact trei solut, ii
reale.

2. Se consideră funct, ia f : [0, +∞) → R, f (x) = arctg x.

a) Demonstrat, i că orice primitivă a funct, iei f este convexă pe intervalul


(0, +∞).
Z 1
f (x)
b) Calculat, i dx.
0 1 + x2
c) Determinat, i numărul real pozitiv a, s, tiind că volumul corpului obt, inut
√ a graficului funct, iei g : [0, a] → R,
prin rotat, ia ı̂n jurul axei Ox
π(π 3 − ln 8)
g(x) = f (x) este egal cu .
3
Teste pentru examenul de Bacalaureat, specializarea matematică-informatică 65

Testul 2

SUBIECTUL I
 
7
1. Determinat, i elementele mult, imii k∈Z ∈N .
k+3

2. Fie x1 s, i x2 solut, iile ecuat, iei x2 + mx − 8 = 0, unde m ∈ R. Determinat, i


numărul real m pentru care x1 (x1 − 8) = x2 (8 − x2 ).

3. Rezolvat, i ı̂n mult, imea numerelor reale inecuat, ia 0,251+x ≤ 0,1251−x .

4. Calculat, i probabilitatea ca, alegând un număr n din mult, imea numerelor


naturale de două cifre, acesta să verifice relat, ia A2n ≤ 90.

5. Se consideră pătratul ABCD s, i fie M , O s, i N mijloacele segmentelor [AB],


[AC], respectiv [OD]. Determinat, i numerele reale a s, i b pentru care are loc
−−→ −→ −−→
egalitatea M N = a AC + b BD.

6. Fie x ∈ R astfel ı̂ncât 2 sin x = 3 cos x. Calculat, i tg 2x.

SUBIECTUL al II-lea
 
1 0 1−x
1. Se consideră matricea A(x) =  0 2019x−1 0  , unde x este număr
0 0 1
real.

a) Calculat, i det (A(0) · A(3)).


b) Arătat, i că, pentru orice număr real x, matricea A(x) este inversabilă s, i
inversa ei este matricea A(2 − x).

n , i perechile (x, n) ∈ R × N pentru care are loc egalitatea
c) Determinat
A(x) = A(x).

2. Pe mult, imea numerelor ı̂ntregi se defines, te legea de compozit, ie asociativă

x ◦ y = 6(x + y) − 2xy − 15.

a) Demonstrat, i că x ◦ y = 3 − 2(3 − x)(3 − y), pentru orice numere ı̂ntregi


x s, i y.
b) Arătat, i că legea de compozit, ie ,,◦” nu admite element neutru.
c) Determinat, i numărul ı̂ntreg m, s, tiind că ecuat, ia x ◦ x ◦ x ◦ x = m are
solut, ie unică ı̂n Z.
66 Costel BĂLCĂU

SUBIECTUL al III-lea
√ √ 
1. Se consideră funct, ia f : R → R, f (x) = x2 + 5 − x + ln x2 + 5 + x .

a) Calculat, i lim f (x).


x→+∞
b) Arătat, i că graficul funct, iei f are un punct de inflexiune.

c) Demonstrat, i că f (− 5) > 5.
Z 1
xn
2. Pentru fiecare număr natural nenul n, se consideră In = 2
dx.
−1 x − 9

a) Arătat, i că I2 = 2 − 3 ln 2.
b) Calculat, i I2020 + 8I2019 − 9I2018 .
c) Demonstrat, i că s, irul (In )n≥1 este convergent.

Testul 3

SUBIECTUL I

(6 + 8i)2
1. Calculat, i modulul numărului complex z = .
(2 − i)4

2. Se consideră funct, ia f : R → R, f (x) = x2 − 2x + 1. Determinat, i numerele


reale a pentru care (f ◦ f )(a) = 0.
√ √
3. Rezolvat, i ı̂n mult, imea numerelor reale ecuat, ia x + 2 = 4 − 8 − x.

4. Determinat, i câte numere naturale de două cifre distincte nu sunt divizibile


cu 5.

5. În reperul cartezian xOy se consideră punctele A(−3, 0) s, i B(1, 2). Determi-
nat, i ecuat, ia mediatoarei segmentului AB.

6. Calculat, i raza cercului circumscris unui triunghi dreptunghic, s, tiind că raza
cercului ı̂nscris ı̂n acest triunghi este egală cu 2, iar aria triunghiului este
egală cu 24.

SUBIECTUL al II-lea

x + 3y + z = 2

1. Se consideră sistemul de ecuat, ii mx + 2z = −3 , unde m s, i n sunt

x + ny + z = −1

numere ı̂ntregi.
Teste pentru examenul de Bacalaureat, specializarea matematică-informatică 67

a) Rezolvat, i sistemul pentru m = 3 s, i n = 0.


b) Pentru m = 3, determinat, i numărul ı̂ntreg n astfel ı̂ncât matricea
sistemului să aibă rangul egal cu 2.
c) Determinat, i perechile de numere ı̂ntregi m s, i n astfel ı̂ncât matricea
sistemului să aibă determinantul egal cu 2.

2. Se consideră polinomul f = X 3 + aX 2 − 21X + 27, unde a este număr real.

a) Determinat, i numărul real a, s, tiind că polinomului f este divizibil cu


X + a.
b) Pentru a = 0, calculat, i (x1 + x2 )3 + (x2 + x3 )3 + (x3 + x1 )3 , unde x1 ,
x2 s, i x3 sunt rădăcinile polinomului f .
c) Determinat, i numărul real a astfel ı̂ncât rădăcinile polinomului f să fie
numere reale ı̂n progresie geometrică.

SUBIECTUL al III-lea

1. Se consideră funct, ia f : (0, +∞) → R, f (x) = x2 ln(2x).

a) Arătat, i că f 0 (1) = ln(4e).


b) Demonstrat, i că graficul funct, iei f nu are asimptote.
c) Determinat, i valoarea minimă a funct, iei f 0 .
x−2
2. Fie funct, ia f : [0, 2] → R, f (x) = 2 . Pentru fiecare număr natural
Z 1 x − 4x
Z 2+ 5
nenul n, fie In = xn f (x) dx s, i Jn = xn f (x) dx.
0 0

a) Arătat, i că aria suprafat, ei plane√delimitate de graficul funct, iei f s, i axele


de coordonate este egală cu ln 5.
b) Arătat, i că lim nIn = f (1).
n→+∞
c) Calculat, i lim Jn .
n→+∞
68 Rezolvarea problemelor din numărul anterior

PROBLEME PENTRU CONCURSURI

Rezolvarea problemelor din numărul anterior

Clasa a V-a

MGO 41. Numerele naturale nenule sunt afis, ate ı̂n pătrăt, elele unei tabele electro-
nice astfel ı̂ncât să se formeze repetat Numărul Marii Uniri, adică 19181918 . . . , ca
mai jos. Tabela are cinci linii, numerotate de sus ı̂n jos cu 1, 2, . . . , 5, iar coloanele
sale sunt numerotate de la stânga la dreapta cu 1, 2, . . . .

1 2 3 4 5 6 7 8 9 10 11 12 ...
1 2 7 11 14 20 25 30 33 39
2 1 3 8 15 19 21 26 34 38 40
3 4 9 12 16 22 27 31 35 41
4 5 17 23 28 36 42
5 6 10 13 18 24 29 32 37 43

Pe ce linie s, i pe ce coloană se află Numărul Centenarului, 2018 ?

Costel Anghel, Slatina

Solut, ie. Numărul 1918 este afis, at repetat pe câte 37 de pătrăt, ele ale tabelei, ce
formează 10 coloane. Avem 2018 : 37 = 54 rest 20, deci până la afis, area numărului
2018 sunt afis, ate 54 de numere 1918, pe 54 · 10 = 540 de coloane, iar ı̂n al 55-lea
număr 1918 pătrăt, elul 2018 este al 20-lea, deci este situat pe linia 1 s, i coloana
540 + 7 = 547 ale tabelei.

MGO 42. Determinat, i numerele naturale nenule n pentru care numărul 1·2·3·. . .·n
se termină ı̂n exact 2018 zerouri.

Stelian Corneliu Andronescu s, i Costel Bălcău, Pites, ti

Solut, ie. Numărul de zerouri ı̂n care se termină numărul 1 · 2 · 3 · . . . · n este egal cu
puterea lui 5 din descompunerea ı̂n factori primi a acestui produs (puterea lui 2
din această descompunere fiind mai mare sau egală cu puterea lui 5), adică cu
hni h n i h n i
+ 2 + 3 + ...
5 5 5
Rezolvarea problemelor din numărul anterior 69

([x] reprezintă partea ı̂ntreagă a numărului rat, ional x), deci avem
hni hni h n i h n i h n i
+ + + + = 2018.
5 25 125 625 3125

Fie n = abcdef (5) scrierea lui n ı̂n baza 5. Obt, inem a + (5a + b) + (25a +
5b + c) + (125a + 25b + 5c + d) + (625a + 125b + 25c + 5d + e) = 2018, adică
781a + 156b + 31c + 6d + e = 2018. Cum a, b, c, d, e, f sunt cifre ı̂n baza 5,
iar 2018 : 781 = 2 rest 456, 456 : 156 = 2 rest 144, 144 : 31 = 4 rest 20,
20 : 6 = 3 rest 2, rezultă că a = 2, b = 2, c = 4, d = 3 s, i e = 2. Astfel
n = 3125 · 2 + 625 · 2 + 125 · 4 + 25 · 3 + 5 · 2 + f = 8085 + f , cu 0 ≤ f ≤ 4, deci
numerele cu proprietatea dată sunt 8085, 8086, 8087, 8088 s, i 8089,

MGO 43. Arătat, i că numărul A = 10n+2 + 112n+1 se divide cu 37, pentru orice
n ∈ N.

Florea Badea, Scornices, ti

Solut, ie. Avem A = 100 · 10n + 11 · 121n = 111 · 10n + 11 (121n − 10n ) = M111 +
11 · M(121 − 10) = M111 = M(37 · 3) = M37.

MGO 44. Câte numere naturale de forma acbabc, scrise ı̂n baza 10, sunt divizibile
cu 7?

Florea Badea, Scornices, ti s, i Costel Anghel, Slatina

Solut, ie. Deoarece A = acbabc = 100100a + 1010b + 10001c = M7 + 5b + 2c =


M7 + 2(7 + c − b), rezultă că A se divide cu 7 dacă 7 + c − b se divide cu 7, adică
pentru următoarele perechi (b, c): (0, 0), (1, 1), (2, 2), (3, 3), (4, 4), (5, 5), (6, 6),
(7, 7), (8, 8), (9, 9), (0, 7), (7, 0), (1, 8), (8, 1), (2, 9), (9, 2). Pentru fiecare dintre
cele 16 astfel de perechi (b, c), a poate fi oricare dintre cele 9 cifre nenule, deci
numărul de numere A divizibile cu 7 este egal cu 16 · 9 = 144.

MGO 45. Numărul ab scris ı̂n baza 10 se numes, te special dacă restul s, i câtul
ı̂mpărt, irii lui a prin b sunt numere naturale nenule egale. Câte numere speciale
există?

Adrian T, urcanu, Pites, ti

Solut, ie. Numărul ab este special dacă a : b = c rest c, cu 0 < c < b, adică dacă
a = bc + c = c(b + 1) cu c ≥ 1 s, i b ≥ c + 1. Pentru c = 1 rezultă că a = b + 1 cu
b ≥ 2, deci obt, inem numerele speciale 32, 43, 54, 65, 76, 87, 98. Pentru c = 2
rezultă că a = 2(b + 1) cu b ≥ 3, deci obt, inem numărul special 83. Pentru c ≥ 3
rezultă că b ≥ 4, deci a ≥ 15, fals. În concluzie, avem 8 numere speciale.
70 Rezolvarea problemelor din numărul anterior

Clasa a VI-a

MGO 46. Calculat, i suma ultimelor 503 cifre ale numărului N = 1 · 2 · 3 · . . . · 2018.
Stelian Corneliu Andronescu s, i Costel Bălcău, Pites, ti
Solut, ie. Utilizăm notat, ia 1 · 2 · 3 · . . . · n = n! (n factorial ). Numărul de zerouri ı̂n
care se termină numărul N = 1 · 2 · 3 · . . . · 2018 = 2018! este egal cu puterea lui
5
 din descompunerea
   ı̂n factori primi a acestui produs, deci cu suma numerelor
2018 403 80 16
= 403, = 80, = 16, = 3 ([x] reprezintă partea ı̂ntreagă a
5 5 5 5
numărului rat, ional x), adică cu 403 + 80 + 16 + 3 = 502. Rezultă că suma ultimelor
503 cifre ale numărului N = 2018! este egală cu ultima cifra nenulă a sa, pe care o
vom nota cu c(N ). Grupând factorii 10 câte 10 s, i descompunând multiplii lui 5
sub forma 5 · k avem 2018! = (1 · 2 · 3 · 4 · 5 · 1 · 6 · 7 · 8 · 9 · 5 · 2)(11 · 12 · 13 · 14 · 5 ·
3 · 16 · 17 · 18 · 19 · 5 · 4) · . . . · (2011 · 2012 · 2013 · 2014 · 5 · 403 · 2016 · 2017 · 2018),
deci 2018! = 5403 · 403! · (1 · 2 · 3 · 4 · 6 · 7 · 8 · 9)(11 · 12 · 13 · 14 · 16 · 17 · 18 · 19) ·
. . . · (2011 · 2012 · 2013 · 2014 · 2016 · 2017 · 2018). Scriind 5403 = 10403 : 2403 s, i
observând că produsul din ultima paranteză are ultima cifră 4, iar produsele din
celelalte paranteze au ultima cifră 6, rezultă că

c(2018!) = c c(403!) · (M10 + 4) : 2403 .




Analog, c(403!) = c c(80!) · (M10+ 6) : 280 , c(80!) = c c(16!) · (M10 + 6) : 216 ,


 

c(16!) = c c(3!) · (M10 + 4) : 23 . Dar c(3!) = 6, de unde rezultă că c(16!) =


c (6 · (M10 + 4) : 8) = 8 (numărul 6 · (M10 + 4) : 8 fiind par), prin urmare avem
c(80!) = c (8 · (M10 + 6) : 6) = 8 (216 având ultima cifră 6 iar numărul c(16!) ·
(M10 + 6) : 216 fiind par) s, i, analog, obt, inem că c(403!) = c (8 · (M10 + 6) : 6) = 8,
c(2018!) = c (8 · (M10 + 4) : 8) = 4.

MGO 47. Fie ABC un triunghi cu AB 6= AC. Determinat, i pozit, ia punctului P


|AB − AC|
situat pe bisectoarea unghiului BAC pentru care P G = , unde G este
3
centrul de greutate al triunghiului BCP .
Florea Badea, Scornices, ti s, i Costel Anghel, Slatina
Solut, ie. Fie P1 s, i P2 picioarele perpendicularelor duse din B, respectiv C pe bisec-
toarea unghiului BAC. Avem P1 6= P2 , deoarece ı̂n caz contrar AP1 ar fi bisectoare
s, i ı̂nălt, ime ı̂n triunghiul ABC s, i ar rezulta că AB = AC, fals. Demonstrăm că
P = P1 s, i P = P2 sunt singurele solut, ii ale problemei. Fie M mijlocul laturii BC.
Dacă BP1 ∩ AC = {B 0 }, atunci triunghiul BAB 0 este isoscel (deoarece AP1 este
bisectoare s, i ı̂nălt, ime), deci AB 0 = AB. Fie G1 centrul de greutate al triunghiului
BCP1 . Folosind s, i faptul că P1 M este linie mijlocie ı̂n triunghiul B 0 BC, avem
Rezolvarea problemelor din numărul anterior 71

2 2 B0C |AC − AB 0 | |AC − AB| |AB − AC|


P 1 G1 = · P 1 M = · = = = , deci
3 3 2 3 3 3
punctul P1 are proprietatea din enunt, . Analog se arată că s, i punctul P2 are
această proprietate. Reciproc, dacă un punct P are proprietatea din enunt, , atunci
3 |AB − AC|
P M = ·P G = , deci M P = M P1 = M P2 , de unde rezultă că P = P1
2 2
|AB − AC|
sau P = P2 (ı̂n caz contrar cercul de centru M s, i rază ar intersecta
2
bisectoarea unghiului BAC ı̂n trei puncte distincte, P , P1 s, i P2 , fals).

MGO 48. Arătat, i că oricare ar fi numerele naturale n, a, b, r cu 0 ≤ r ≤ 3 s, i


a ≥ b ≥ 1, numărul N = n4a+r − n4b+r este divizibil cu 30.
Marian Haiducu, Pites, ti
Solut, ie. Avem N = nr n4a − n . Cum n4 ∈ {M3, M3 + 1} s, i (M3)k = M3,
4b


(M3 + 1)k = M3 + 1, pentru orice k ∈ N∗ , rezultă că n4a − n4b = M3, deci N
se divide cu 3. Cum n4 ∈ {M10, M10 + 1, M10 + 5, M10 + 6} s, i (M10 + r)k =
M10 + r, pentru orice r ∈ {0, 1, 5, 6} s, i k ∈ N∗ , rezultă că n4a − n4b = (M10 +
r) − (M10 + r) = M10, deci N se divide cu 10. Fiind divizibil cu 3 s, i cu 10, iar 3
s, i 10 fiind prime ı̂ntre ele, rezultă că N este divizibil cu 30.

MGO 49. Determinat, i numerele naturale a s, i b, s, tiind că cel mai mare divizor
comun al lor este 1005, iar suma pătratelor lor este egală cu 10100250.
Marin Chirciu, Pites, ti
Solut, ie. Avem (a, b) = 1005, deci a = 1005x s, i b = 1005y cu (x, y) = 1. Înlocuind
ı̂n a2 +b2 = 10100250 obt, inem 10052 ·x2 +10052 ·y 2 = 10·10052 , adică x2 +y 2 = 10,
cu solut, iile (x, y) ∈ {(1, 3), (3, 1)}. Astfel (a, b) ∈ {(1005, 3015), (3015, 1005)}.

MGO 50. Fie ABC un triunghi, D un punct pe dreapta BC diferit de B s, i de C,


M un punct pe segmentul (AD), iar E s, i F picioarele perpendicularelor duse din
D pe dreptele M B, respectiv M C. S, tiind că [DE] ≡ [DF ] s, i EF kBC, arătăt, i că:
a) E ∈ (M B) s, i F ∈ (M C), iar D este mijlocul lui [BC].
b) Triunghiul ABC este isoscel, [AB] ≡ [AC].
Florin Stănescu, Găes, ti
Solut, ie. a) Avem 4M ED ≡ 4M EF (cazul C.I.), deci ^DM E ≡ ^DM F s, i
[M E] ≡ [M F ], de unde rezultă că punctele E s, i F sunt situate de o parte s, i de
alta a dreptei AD (ı̂n caz contrar am avea E = F , deci M, B, C ar fi coliniare,
fals) s, i că M D este perpendiculară pe EF ı̂n mijlocul O al lui [EF ]. Astfel avem
O ∈ (M D) (EO fiind ı̂nălt, ime ı̂n 4M ED dreptunghic ı̂n E). Cum EF kBC,
rezultă că E ∈ (M B) s, i F ∈ (M C). În 4BM C, M D este bisectoare s, i ı̂nălt, ime,
deci este s, i mediană, adică D este mijlocul lui [BC]. b) În 4BAC, AD este
ı̂nălt, ime (deoarece AD ⊥ EF s, i EF kBC) s, i mediană, deci 4BAC este isoscel,
[AB] ≡ [AC].
72 Rezolvarea problemelor din numărul anterior

Clasa a VII-a

r
25n − 1
MGO 51. Determinat, i numerele rat, ionale de forma A = , cu n ∈ N∗ .
n + 11
Marin Chirciu, Pites, ti

a 25n − 1 a2
Solut, ie. Fie A = ∈ Q, cu a, b ∈ N∗ , (a, b) = 1. Atunci = 2 , deci avem
b n + 11 b
11a2 + b2 276b2 ∗
∈ N . Dar (a, b) = 1, deci a , b2 = 1, prin
2

n= 2 2
= −11 + 2 2
25b − a  25b − a
urmare 25b2 − a2 , b2 = 1. Rezultă că (25b2 − a2 )|276 s, i cum 25b2 − a2 > 0 (deoa-
rece n ∈ N∗ ) obt, inem că (5b − a)(5b + a) ∈ {1, 2, 3, 4, 6, 12, 23, 46, 69,
 92, 138, 276}.
5b − a = 1
Cum (5b − a) + (5b + a) = 10b este divizibil cu 10, deducem că
5b + a = 69
  
5b − a = 2 a = 34 a = 68
sau , deci sau . Dar (a, b) = 1, deci singura
5b + a = 138 b=7 b = 14
34
solut, ie este a = 34, b = 7, de unde obt, inem că n = 185 s, i A = .
7
MGO 52. În exteriorul triunghiului ABC având m (^A) = 30◦ se construiesc
triunghiurile echilaterale ABD s, i ACE. Fie K simetricul punctului A fat, ă de
mijlocul segmentului [DE]. Arătat, i că ABKC = AABD + AACE − 3AABC .

Mihai Florea Dumitrescu, Potcoava

Solut, ie. ADKE este paralelogram (deoarece diagonalele au acelas, i mijloc) s, i


m(^DAE) = 150◦ , deci DK = AE = AC = CE, EK = AD = AB = BD
s, i m(^ADK) = ^AEK) = 30◦ . Astfel m(^BDK) = ^CEK) = 60◦ − 30◦ =
30◦ . Rezultă că 4BAC ≡ 4BDK ≡ 4KEC (cazul L.U.L.), deci 4BKC este
bc sin A bc
echilateral. Deoarece m (^BAC) = 30◦ , avem AABC = = s, i, conform
2 4 √
2 2 2 2 2
√ 3
Teoremei cosinusului, a = b + c − 2bc cos A = b + c − bc 3. Înmult, ind cu
√ √ √ 4
a2 3 b2 3 c2 3 3bc
obt, inem = + − , adică ABKC = AACE + AABD − 3AABC .
4 4 4 4
MGO 53. Fie ABCD un patrulater convex, a, b, c, d lungimile laturilor s, i d1 ,
d2 lungimile diagonalelor sale. Arătat, i că

a+b+c+d d2 + d22 a+b+c+d


< 1 < .
4 d1 + d2 2
Marian Haiducu, Pites, ti
Rezolvarea problemelor din numărul anterior 73

Solut, ie. Fie AB = a, BC = b, CD = c, DA = d, AC = d1 , BD = d2 s, i fie


AC ∩ BD = {O}. Conform Inegalităt, ii triunghiului avem d1 < a + b s, i d1 < c + d,
deci 2d1 < a + b + c + d, de unde rezultă că 2d21 < d1 (a + b + c + d). Analog, avem
2d22 < d2 (a + b + c + d). Prin adunare obt, inem 2(d21 + d22 ) < (a + b + c + d)(d1 + d2 ),
d2 + d22 a+b+c+d
prin urmare 1 < .
d1 + d2 2
Tot conform Inegalităt, ii triunghiului avem a < OA + OB, b < OB + OC,
c < OC + OD s, i d < OD + OA, de unde prin adunare obt, inem a + b + c + d <
a+b+c+d d1 + d2 d1 + d2 d2 + d22
2(d1 + d2 ), deci < . Cum ≤ 1 (inegalitatea
4 2 2 d1 + d2
2 2 2 2
fiind echivalentă cu (d1 + d2 ) ≤ 2(d1 + d2 ), adică cu (d1 − d2 ) ≥ 0), rezultă că
a+b+c+d d2 + d22
< 1 .
4 d1 + d2

MGO 54. Fie ABC un triunghi circumscris unui cerc C de centru I. Cercul C
este tangent laturilor BC, CA, AB ı̂n punctele D, E, respectiv F s, i intersectează
segmentele [AI], [BI], [CI] ı̂n punctele M , N , respectiv P . Arătăt, i că triunghiurile
DEF s, i M N P au acelas, i centru de greutate dacă s, i numai dacă triunghiul ABC
este echilateral.

Marin Ionescu, Pites, ti

Solut, ie. Dacă triunghiul ABC este echilateral, atunci triunghiurile DIE, EIF ,
F ID, M IN , N IP s, i P IM sunt congruente (cazul L.U.L.), deci triunghiurile DEF
s, i M N P sunt echilaterale s, i au centrul I.
Reciproc, să considerăm că triunghiurile DEF s, i M N P au acelas, i centru de
greutate G. Cum ele au s, i acelas, i centru I al cercului circumscris, utilizând dreapta
lui Euler rezultă că ele au s, i acelas, i ortocentru, s, i anume punctul H ∈ [IG cu
IH = 3IG. Deoarece 4AEI ≡ 4AF I (cazul I.U.) rezultă că ^AIE ≡ ^AIF ,
_ _
prin urmare m(M E) = m(M F ), de unde obt, inem ^M DE ≡ ^M DF . Analog,
_ _ _ _
avem m(N D) = m(N F ) s, i m(P E) = m(P D), deci, notând M D ∩ N P = {Q},
rezultă că
_ _ _
m(M E) + m(P E) + m(N D) 180◦
m(^M QP ) = = = 90◦ ,
2 2
adică M D ⊥ N P . Deducem că bisectoarele triunghiului DEF coincid cu ı̂nălt, imile
triunghiului M N P , deci centrul cercului ı̂nscris ı̂n triunghiul DEF coincide cu
ortocentrul H al triunghiului M N P . Dar H este s, i ortocentru pentru triunghiul
DEF , deci acest triunghi este echilateral (s, i H = I). Atunci m(^DIE) =
m(^EIF ) = m(^F ID) = 120◦ , deci m(^A) = m(^B) = m(^C) = 180◦ − 120◦ =
60◦ , adică triunghiul ABC este echilateral.
74 Rezolvarea problemelor din numărul anterior

n(n + 1)
MGO 55. Fie numerele reale a1 , a2 , . . . , an având suma egală cu . Dacă
p p p 2
2 2 2 2 2 2
(a1 + 1) + (a2 + 2) + (a2 + 2) +(a3 + 3) + . . . + (a n + n) + (a1 + 1) ≤
√ 3 n+1
n(n + 1) 2, să se arate că a1 − 1 + 2 a2 − + . . . + n an − = 0.
2 2
Sorin Ulmeanu s, i Costel Bălcău, Pites, ti
Solut, ie. Notăm cu E expresia r din membrul stâng al inegalităt, ii din ipoteză.
x2 + y 2 x+y
Utilizând Inegalitatea mediilor, ≥ , pentru orice x, y ∈ R, cu
2 2
1 1
egalitate dacă s, i numai dacă x = y, rezultă că √ · E ≥ (a1 + 1) + (a2 +
2 2 √
2) + (a2 + 2) + (a3 + 3) + . . . + (an + n) + (a1 + 1) , deci E ≥ 2(a1 + a2 +
√ n(n + 1) n(n + 1) √

. . . + an + 1 + 2 + . . . + n) = 2 + = n(n + 1) 2. Dar,
2√ 2 √
conform ipotezei, avem s, i E ≤ n(n + 1) 2. Rezultă că E = n(n + 1) 2, adică
inegalităt, ile anterioare devin egalităt, i. Deducem că a1 + 1 = a2 + 2 = a3 +
3 = . . . = an + n. Rezultă că a1 + 1 = a2 + 2 = a3 + 3 = . . . = an + n =
1
(a1 + 1 + a2 + 2 + a3 + 3 + . . . + an + n) = n + 1, deci a1 = n, a2 = n − 1,
n
a3 = n −  2, . . . , an 
= 1, adică
 ak = n + 1 − k,  ∀k ∈ {1, 2, . . . , n}.2 Prin urmare,
k+1 k+1 (2n + 1)k − 3k
avem k ak − =k n+1−k− = , deci a1 − 1 +
2  2 2
(2n + 1) − 3 · 12 (2n + 1) · 2 − 3 · 22
  
3 n+1
2 a2 − +. . .+n an − = + +. . .+
2 2 2 2 
(2n + 1)n − 3n2 (2n + 1) 1 + 2 + . . . + n − 3 12 + 22 + . . . + n2 2n + 1
= = ·
2 2 2
n(n + 1) 3 n(n + 1)(2n + 1)
− · = 0.
2 2 6

Clasa a VIII-a

MGO 56. Fie V ABCD s, i V AEF două piramide regulate de vârf V astfel ı̂ncât
ı̂nălt, imile lor sunt congruente s, i EF kBD. Calculat, i raportul dintre volumele celor
două piramide.
Florea Badea, Scornices, ti s, i Costel Anghel, Slatina
Solut, ie. Notăm centrele bazelor piramidelor V ABCD s, i V AEF cu O, respectiv O0 .
Demonstrăm că AO = AO0 . Egalitatea fiind evidentă pentru O = O0 , considerăm
cazul O = 6 O0 . Avem (V AO0 ) ⊥ EF s, i EF kBD, deci (V AO0 ) ⊥ BD. Dar s, i
(V AO) ⊥ BD, deci (V AO0 ) = (V AO), adică punctele V, A, O, O0 sunt coplanare.
Triunghiul OV O0 fiind isoscel, avem ^V OO0 ≡ ^V O0 O. Cum s, i ^V OA ≡ ^V O0 A
Rezolvarea problemelor din numărul anterior 75

(unghiuri drepte), rezultă că ^AOO0 ≡ ^AO0 O, deci triunghiul OAO0 este
√ isoscel,
VV ABCD AABCD 2AO 2 8 3
cu AO = AO0 . Prin urmare, avem = = 2
√ = .
VV AEF AAEF 3AO 3 9
4
MGO 57. Reprezentat, i grafic, ı̂ntr-un sistem de coordonate xOy, solut, iile ecuat, iei

x3 + y 3 + (x + y)3 + 50(x + y) + 15xy = 500, x, y ∈ R.

Marin Chirciu, Pites, ti

Solut, ie. Egalitatea din enunt, este echivalentă cu (x + y − 5)(2x2 + xy + 2y 2 + 10x +


10y + 100) = 0. Cum 2x2 + xy + 2y 2 + 10x + 10y + 100 = (x + 5)2 + (y + 5)2 +
 y 2 3y 2
x+ + + 50 > 0, obt, inem că x + y − 5 = 0 (ecuat, ia unei dreapte).
2 4
MGO 58. Se consideră cubul ABCDA0 B 0 C 0 D0 , E mijlocul segmentului [AA0 ] s, i
F ∈ C 0 D0 astfel ı̂ncât EB ⊥ DF . Calculat, i cosinusul unghiului dintre planele
(EBD) s, i (F BD).

Mihai Florea Dumitrescu, Potcoava

Solut, ie. În pătratul ABB 0 A0 se arată că EB ⊥ AF 0 , unde F 0 este mijlocul lui
[A0 B 0 ]. Rezultă că DF kAF 0 , deci F este mijlocul lui [C 0 D0 ]. Fie M mijlocul lui
[CD] s, i M P ⊥ BD, P ∈ BD, deci P este mijlocul lui [OD], unde {O} = AC ∩BD.
Conform Teoremei celor trei perpendiculare rezultă că F P ⊥ BD. Fie N mijlocul
lui [ED]. Avem N P kEO, deci N P ⊥ √ BD. Rezultă că ^ ((EBD), (F√ BD)) =
a 2 √ a 3
^N P F . Fie AB = a. Aven AO = , EO = EA2 + AO2 = , deci
√ 2 √ 2
EO a 3 CO a 2
NP = = . De asemenea, avem F M = a, M P = = , deci F P =
2 4 √ 2 4 √
√ 3a 2 0 0 A 0C 0 a 2
FM2 + MP2 = . Fie Q mijlocul lui [A D ]. Avem QF = = ,
4 √ 2 2
A0 E + D 0 D 3a p a 17
QN = = , deci F N = QF 2 + QN 2 = . Aplicând Teorema
2 4 4 √
NP + FP − FN2
2 2 6
cosinusului ı̂n 4N P F avem cos(^N P F ) = = .
2N P · F P 9
1 1 1
MGO 59. Fie n ∈ N, n ≥ 2 s, i x1 , x2 , . . . , xn > 0 astfel ı̂ncât + +. . .+ =
x1 x2 xn
n. Arătăt, i că 2 2 2 2
x1 x2 + x2 x3 + . . . + xn−1 xn + xn x1 ≥ 2(x1 + x2 + . . . + xn ) − n.

Marin Ionescu, Pites, ti

Solut, ie. Folosind egalitatea din


 enunt
 , s, i Inegalitatea
 mediilor,
 avemx21 x2 + x22 x3 +
1 1 1
. . . + x2n x1 + n = x21 x2 + + x22 x3 + + . . . + x2n x1 + ≥ 2x1 + 2x2 +
x2 x3 x1
. . . + 2xn .
76 Rezolvarea problemelor din numărul anterior

2 2
MGO 60. Aflat, i a, b, c, d > 0, s, tiind că a+b+c+d = 2 s, i+ +
2a + b + c 2b + c + d
2 2 1 2 1
+ = + + .
2c + d + a 2d + a + b (a + b)(c + d) (a + c)(b + d) (a + d)(b + c)
Florin Stănescu, Găes, ti
x2 y 2 (x + y)2
Solut, ie. Utilizând Inegalitatea lui Bergström, + ≥ , unde z, t > 0,
z t z+t
x y 2 1 4
cu egalitate dacă s, i numai dacă = , avem = · ≤
  z t 2a + b + c 2 a + b + a+c
1 1 1
+ , cu egalitate dacă s, i numai dacă b = c. Analog, obt, inem
2 a + b a +c   
2 1 1 1 2 1 1 1 2
≤ + , ≤ + , ≤
2b+ c + d 2 b+ c b + d 2c + d + a 2 c+a c+d 2d + a + b
1 1 1
+ . Adunând membru cu membru aceste inegalităt, i s, i utilizând
2 d+a d+b
2 2 2
faptul că a + b + c + d = 2 rezultă că + + +
   2a + b + c 2b + c+ d 2c + d +a
2 1 1 1 1 1 1 1 1
≤ + + + + + =
2d + a + b 2 a+b c+d a+c b+d 2 a+d b+c
1 2 1
+ + . Dar, conform ipotezei, această
(a + b)(c + d) (a + c)(b + d) (a + d)(b + c)
inegalitate devine egalitate, deci s, i inegalităt, ile adunate devin egalităt, i. Deducem
1
că a = b = c = d = .
2

Clasa a IX-a

MGO 61. Se consideră triunghiul ABC s, i punctele M ∈ (AB), N ∈ (AC), P ∈


(BC) astfel ı̂ncât AM = CP , AN = BP , BM = CN . Fie A0 , B 0 , C 0 mijloacele
segmentelor [BC], [AC], respectiv [AB], iar M 0 , N 0 , P 0 mijloacele segmentelor
ha hb hc
[N P ], [M P ], respectiv [M N ]. Arătat, i că A0 P 0 · B 0 N 0 · C 0 M 0 = , unde
8
ha , hb , hc reprezintă lungimile ı̂nălt, imilor triunghiului ABC.
Mihai Florea Dumitrescu, Potcoava
Solut, ie. Notând BM = CN = x, AM = CP = y s, i AN = BP = z, avem
y + z = a, x + z = b, x + y = c, deci x = p − a, y = p − b, z = p − c.
−−
0
→0 −−→0 −−→0 1 −−→ −→ 1 −−→ −−→ 1 −−→ −→
Avem P A = AA − AP = AB + AC − AM + AN = AB + AC −
 2
 2 2
1 p − b −−→ p − c −→ p − a −−→ p − a −→
· AB + · AC = · AB + · AC. Rezultă că A0 P 0 =
2 c r b 2c 2b
−− → p − a 1 −−→ 2 1 −→ 2 2 −−→ −→ p − a √
P 0 A0 = · · AB + · AC + · AB · AC = · 2 + 2 cos A =
2 c b bc 2
Rezolvarea problemelor din numărul anterior 77
r r
A p(p − a) 0 0 p(p − b)
(p − a) cos = (p − a) . Analog, avem B N = (p − b) s, i
2 r bc ac
p(p − c) p(p − a)(p − b)(p − c)
C 0 M 0 = (p−c) . Rezultă că A0 P 0 ·B 0 N 0 ·C 0 M 0 = ·
ab abc
p S3 ha hb hc
p(p − a)(p − b)(p − c) = = .
abc 8
MGO 62. Fie x, y, z ∈ [0, 2] astfel ı̂ncât xy + yz + zx + xyz = 4. Determinat, i
4 − xy 4 − yz 4 − zx
valorile extreme ale expresiei + + .
4 + xy 4 + yz 4 + zx
Leonard Mihai Giugiuc, România s, i Michael Rozenberg, Israel
Solut, ia 1 (Anh Duy, Vietnam). Cu substitut, iile
2 − 2a 2 − 2b 2 − 2c
x= , y= , z=
1+a 1+b 1+c
P 4 − xy P a + b
avem a, b, c ∈ [0, 1], a + b + c = 1 s, i S = = . Demonstrăm că
4 + xy 1 + ab
9 X a+b
≤ ≤ 2.
5 1 + ab
P a+b Pa+b
Avem S = ≤ = 2(a + b + c) = 2. Notând q = ab + bc + ca s, i
1 + ab 1
9 r(q − 9r) + 3r(3q + 2) − (4q − 1)
r = abc, obt, inem că S = + . Cum q − 9r =
5 5(q + r2 + r + 1)
(a + b + c)(ab + bc + ca) − 9abc ≥ 0 (conform Inegalităt, ii mediilor ), rezultă că
9 3r(3q + 2) − (4q − 1)
S≥ + , (1). Avem (a + b + c)2 ≥ 3(ab + bc + ca), deci
5 5(q + r2 + r + 1)
1 1 9 1 1
q ≤ . Cazul 1. 0 ≤ q ≤ . Din (1) rezultă că S ≥ . Cazul 2. <q ≤ .
3 4 5 4 3
Din (a2 + b2 + c2 )2 + 6abc(a + b + c) ≥ (a + b + c)2 (ab + bc + ca) (Inegalitatea lui
Schur pentru t = 2) rezultă 6r ≥ (1 − q)(4q − 1), prin urmare din (1) obt, inem
9 (1 − q)(4q − 1)(3q + 2) − 2(4q − 1) 9 q(4q − 1)(1 − 3q) 9
S≥ + = + ≥ .
5 10(q + r2 + r + 1) 5 10(q + r2 + r + 1) 5
Solut, ia 2 (Toang Huc Khein, software developer, Marea Britanie). Condit, iile
xy + yz + zx + xyz = 4 s, i x, y, z > 0 implică existent, a a trei variabile a, b, c > 0
astfel ı̂ncât
2a 2b 2c
x= , y= , z=
b+c c+a a+b
a b b c c a a b c
(deoarece · + · + · +2· · · = 1).
b+c c+a c+a a+b a+b b+c b+c c+a a+b
Restrict, iile x, y, z ≤ 2 implică a + b ≥ c, b + c ≥ a, c + a ≥ b. Astfel a, b s, i c sunt
lungimile laturilor unui triunghi (posibil degenerat). Avem
X 4 − xy X a(a + b + c)
= .
4 + xy a(a + b + c) + 2bc
78 Rezolvarea problemelor din numărul anterior

Pentru a determina marginea superioară a acestei sume, observăm că avem


a(a + b + c) a(b + c) abc(b + c − a)
= − , de unde
a(a + b + c) + 2bc ab + bc + ca (ab + bc + ca) [a(a + b + c) + 2bc]
P a(a + b + c) P a(b + c)
rezultă că ≤ = 2. Egalitatea are loc pentru
a(a + b + c) + 2bc ab + bc + ca
abc = 0, adică (x, y, z) = (0, 2, 2) s, i permutările sale.
Pentru a determina marginea inferioară, utilizăm substitut, iile lui Ravi :
1 1 1
X = (a + b − c), Y = (b + c − a), Z = (c + a − b). Avem
2 2 2
X a(a + b + c) X (X + Y )(X + Y + Z)
= .
a(a + b + c) + 2bc (X + Y + Z)2 + XY
9
Pentru X = Y = Z suma are valoarea . Arătăm că aceasta este chiar margi-
5
P X +Y 9
nea inferioară, adică (X + Y + Z) 2
≥ sau, echivalent,
Pn hP i h(X + Y + Z) + ioXY Q 5h P i
2 P 2
( X)2 + XY .
P
5 ( X) (X + Y ) ( X) + Y Z ( X) + ZX ≥ 9
P 6
Efectuând
P 4 P ı̂nmult , irile, expresiaP 3din membrul stâng
P este
P egală cu 10 ( X) +
5 ( X) ( XY ) + 15XY Z ( X) P + 10XYP Z ( X) P( XY ), iar expresiaP din
membrul drept este egală cu 9 ( X)6 + 9 ( X)4 ( PXY ) + 9XY Z ( P X)3 +
9X 2 Y 2 Z 2P, deci reducând termenii
P inegalitatea devine ( X)6 + 6XY Z ( X)3 +
( X)4 ( XY ) + 9X 2 Y 2 Z 2 . Efectuând
P P
10XY Z ( X) ( PXY ) ≥ 4P P 4 2 calculele
sePobt, ine forma X + 2 XY (X 4 + Y 4 ) + 3X 2 Y 2 Z 2 ≥
6 X (Y + Z 2 ) +
4 X 3 Y 3P . Această inegalitate este suma dintre Inegalitatea lui Schur de gradul
alP treilea X 2 (X 2 − YP 2 )(X 2 − Z 2 ) ≥ 0 si Inegalitatea mediilor aplicată astfel:
, P
2 XY (X + Y ) ≥ 2 XY · 2X 2 Y 2 = 4 X 3 Y 3 . Egalitatea are loc
4 4
 pentru
2 2
X = Y = Z sau X = Y, Z = 0, adică (x, y, z) = (1, 1, 1) sau (x, y, z) = 2, ,
3 3
9
s, i permutările sale. În concluzie, valorile extreme ale expresiei date sunt s, i 2.
5
MGO 63. Fie ABC s, i A0 B 0 C 0 două triunghiuri asemenea astfel ı̂ncât vectorii
−−→0 −−→0 −−→
AA , BB s, i CC 0 nu sunt tot, i coliniari s, i au suma ~0. Să se arate că cercurile
ı̂nscrise ı̂n cele două triunghiuri au acelas, i centru dacă s, i numai dacă triunghiurile
sunt echilaterale.
Marin Ionescu, Pites, ti
Solut, ie. Fie G s, i G0 centrele de greutate ale triunghiurilor ABC, respectiv A0 B 0 C 0 ,
iar I, respectiv I 0 centrele cercurilor ı̂nscrise ı̂n cele două triunghiuri. Conform
−−→ −−→ −−→
ipotezei, AA0 + BB 0 + CC 0 = ~0, (1). Rezultă că G = G0 . Dacă triunghiurile
sunt echilaterale, atunci I = G s, i I 0 = G0 , deci I = I 0 . Reciproc, să considerăm că
−→ a −→ b −−→ c −−→ −→ −−→ −−→ ~
I = I 0 . Avem GI = · GA + · GB + · GC s, i GA + GB + GC = 0, deci
2p 2p 2p
−→ a − c −→ b − c −−→ −→ a0 − c0 −−→0 b0 − c0 −−→0
GI = · GA+ · GB, (2). Analog, GI = · GA + · GB , (3).
2p 2p 2p0 2p0
Rezolvarea problemelor din numărul anterior 79

a−c a0 − c0 b−c
Dar, deoarece 4ABC ∼ 4A0 B 0 C 0 , rezultă că = = α s, i =
2p 2p0 2p
0
b −c 0 −−→ −−→ −−→
0
= β, deci din (2) s, i (3) obt, inem că αAA0 + β BB 0 = ~0. Cum vectorii AA0 s, i
2p
−−→0 −−→ −−→ −−→
BB nu sunt coliniari (ı̂n caz contrar, din (1) ar rezulta că AA0 , BB 0 s, i CC 0 ar
fi tot, i coliniari, contradict, ie cu ipoteza), rezultă că α = β = 0, deci a = b = c s, i
a0 = b0 = c0 , adică triunghiurile ABC s, i A0 B 0 C 0 sunt echilaterale.

MGO 64. Să se determine numerele reale x, y, z, t, u, v cu proprietatea că


  
2 2 2 11 2 2 2 11
x+y+z+t+u+v = x +y +z + y +z +t + z 2 + t2 +
12 12
   
2 11 2 2 2 11 2 2 2 11 2 2 2 11
+u + t +u +v + u +v +x + v +x +y + .
12 12 12 12
Daniel Jinga s, i Costel Bălcău, Pites, ti

Solut, ie (Leonard Mihai Giugiuc, Drobeta Turnu Severin). Notăm cu S s, i E


expresiile din membrul stâng, respectiv drept  al egalităt, ii din enunt
 , . Aplicând
11
Inegalitatea Cauchy-Buniakowski-Schwarz avem x2 + y 2 + z 2 + t2 + u2 + v 2 +
   12 
11 1 1 1 5 1 1 1 5
= x2 + y 2 + z 2 + + + + + + + t2 + u2 + v 2 + ≥
12 36 36 36 6 36 36 36 6
S+5 2 S+5 6
   
. Procedând analog, deducem că E ≥ . Dar S = E > 0, deci
6 6
6
S−1 6
  
S+5
utilizând Inegalitatea lui Bernoulli avem E ≥ = 1+ ≥ S.
6 6
Cum E = S, rezultă că toate inegalităt, ile anterioare devin egalităt, i, deci S = 1 s, i
1 1 1
6x = 6y = 6z = = = = 1 (s, i analoagele), adică x = y = z = t = u =
6t 6u 6v
1
v= .
6

MGO 65. Determinat, i funct, iile f : N → N cu proprietatea că

f (x + y) = f (x) + f (y) + 2xy(2x2 + 3xy + 2y 2 ), ∀x, y ∈ N.

Sorin Ulmeanu, Pites, ti

Solut, ie. Funct, ia f0 (n) = n4 verifică proprietatea dată, deoarece (x+y)4 = xu +y 4 +


2xy(2x2 + 3xy + 2y 2 ), ∀x, y ∈ N. Fie funct, ia g : N → Z, g(n) = f (n) − n4 , ∀n ∈ N.
Atunci egalitatea din enunt, devine g(x + y) = g(x) + g(y), ∀x, y ∈ N s, i se arată
prin induct, ie după n ∈ N că g(n) = ng(1), ∀n ∈ N. Notând g(1) = k, rezultă că
f (n) = n4 + kn, ∀n ∈ N, cu k ∈ N (deoarece f (0) = k). Evident, orice funct, ie de
această formă verifică proprietatea dată.
80 Rezolvarea problemelor din numărul anterior

Clasa a X-a

MGO 66. Rezolvat, i ı̂n mult, imea numerelor reale inecuat, ia

5x + 9x + 10x + 12x
> 1.
3x + 4x + 6x + 8x + 15x
Mihai Florea Dumitrescu, Potcoava
Solut, ie. Inecuat, ia devine, succesiv: 15x + 8x + 6x + 4x + 3x− 12 x − 10x − 9x − 5x < 0

4 x
  x 
3
(numitorul fiind pozitiv); (5x − 4x − 3x )(3x − 2x − 1) < 0; 1 − − 1−
 x  x  5 5
2 1
− < 0. Notând expresiile din cele două paranteze drepte cu f (x),
3 3
respectiv g(x), rezultă că f s, i g sunt funct, ii strictcrescătoare pe R,cu f (2) = 0
f (x) > 0 f (x) < 0
s, i g(1) = 0. Astfel avem: f (x)g(x) < 0 ⇔ sau
g(x) < 0 g(x) > 0
   
f (x) > f (2) f (x) < f (2) x>2 x<2
⇔ sau ⇔ (fals) sau ⇔
g(x) < g(1) g(x) > g(1) x<1 x>1
x ∈ (1, 2).
1 1 1
MGO 67. Fie a, b, c > 0 astfel ı̂ncât a + b + c = + + . Demonstrat, i că
a b c
p √ √ √
a2 + b2 + c2 + 2 2 − 3 ≥ 2 2abc.
Leonard Mihai Giugiuc, Drobeta Turnu Severin
abc (a + b + c) ab + bc + ca
Solut, ie. Evident, 1 = s, i abc = . Deci inegalitatea
ab + bc + ca a+b+c
cerută se rescrie ca
p  √ √ 
r
abc (a + b + c) √

ab + bc + ca

a2 + b2 + c2 + 2 2 − 3 ≥2 2 , (1).
ab + bc + ca a+b+c

Demonstrăm că această inegalitate are loc pentru orice a, b, c > 0. Putem pre-
3a
supune, fără a restrânge generalitatea, că a + b + c = 3 (se ia a0 = ,
a+b+c
3b 3c
b0 = , c0 = , deci a0 + b0 + c0 = 3, iar din (1) pentru (a0 , b0 , c0 ) se
a+b+c a+b+c
obt, ine (1) pentru (a, b, c)). Cum 3 (ab + bc + ca) ≤ (a + b + c)2 , există t ∈ [0, 1)
astfel ı̂ncât ab + bc + ca = 3 1 − t2 , deci a2 + b2 + c2 = 3 1 + 2t2 .

 
1
Cazul 1. t ∈ 0, . Conform Teoremei 3 de la pag. 41 din RMGO 1/2018
2
(http://rmgo.upit.ro), valoarea minimă a produsului abc este (1 + t)2 (1 − 2t),
Rezolvarea problemelor din numărul anterior 81

ea fiind atinsă ı̂n (1 + t, 1 + t, 1 − 2t) s, i permutările sale. Astfel este suficient


√ √  (1 − 2t) (1 + t) √
r
p
≥ 2 2 1 − t2 . Prin

să arătăm că 3 (1 + 2t2 ) + 2 2 − 3
1−t
√ √ √ p
r
2
1 − 2t
ridicare la pătrat, aceasta devine 2 3 2 2 − 3 (1 + 2t ) (1 + t) ≥ ·
 2 √ √ √ 1−t
2t 2t2 − t − 6 + 2 3 2 2 − 3 (1 + t) . Cum 2t2 2t2 − t − 6 ≤ 0, este su-
   
r
p
2
1 − 2t
ficient să arătăm că (1 + 2t ) (1 + t) ≥ (1 + t) , inegalitate echivalentă
1−t
cu 1 + 2t2 (1 − t) ≥ (1 − 2t) (1 + t), deci cu 4t2 ≥ 2t3 , adevărat.


√ √  abc (a + b + c)
  r
1
Cazul 2. t ∈ , 1 . Atunci 2 2 − 3 > 0, deci este
2 p √ ab + bc + ca
suficient să arătăm că 3 (1 + 2t2 ) ≥ 2 2 1 − t2 , adică 2t2 − 5 4t2 − 1 ≤ 0,
  

adevărat.

MGO 68. Determinat, i numerele reale pozitive k pentru care inegalitatea


√ √ √ √
ka + 1 + kb + 1 + kc + 1 ≥ 3 k + 1

are loc pentru orice a, b, c ∈ [0, ∞) astfel ı̂ncât a + b + c = ab + bc + ca > 0.


Leonard Mihai Giugiuc, România, Michael Rozenberg, Israel
s, i Valmir Krasniqi, Kosovo
Solut, ie.
√ a = 0 √ s, i b = c = 2 verifică a + b + c = ab + bc + ca > 0, deci
1 + 2 2k + 1 ≥ 3 k + 1, de unde obt, inem k ≤ 24. Reciproc, vom demonstra că
orice k ∈ (0, 24] satisface cerint, a problemei. Fie a + b + c = ab + bc + ac = w > 0.
Deoarece (a + b + c)2 ≥ 3 (ab + bc + ac), avem w2 ≥ 3w, deci w ≥ 3. Putem
presupune, fără a restrânge generalitatea, că bc = max {ab, bc, ac}. Atunci bc ≥ 1.
b + c − bc
Evident, a = . Notăm b + c = 2s s, i bc = p2 . Avem s ≥ p (Inegali-
b+c−1
2s − p2
tatea mediilor ), p ≥ 1 s, i a = . Cazul 1. 1 ≤ p ≤ 2. Considerăm funct, ia
2s − 1
2t − p2
fp (t) = , t ∈ [p, ∞). Deoarece p2 ≥ 1, rezultă că fp este crescătoare,
2t − 1
2p − p2
deci fp (s) ≥ fp (p), prin urmare a ≥ . Aplicând Inegalitatea medii-
√ √ p 2p − 1 p
lor avem kb + 1 + kc + 1 ≥ 2 4
(kb + 1) (kc + 1) = 2 4
k 2 bc + k (b + c) + 1 =
p
4
p4
√ √
2 2 2 2
2 k p + 2ks + 1 ≥ 2s k p + 2kp + 1 = 2 kp + 1. Prin urmare, ka + 1 +
√ √ √
   
2−p 2−p k
kb + 1 + kc + 1 ≥ kp + 1 + 2 kp + 1. Dar kp = ·
2p − 1 2p − 1 p
2
     
2−p p k 2−p 2−p
p2 s, i ≥ 1, deci · p2 ≥ k . Astfel, avem
s 2p −1 2p − 1 p
s 2p − 1 p
√ √ √
   
2−p 2−p
kp + 1 + 2 kp + 1 ≥ k + 1 + 2 kp + 1, deci ka + 1 +
2p − 1 p
82 Rezolvarea problemelor din numărul anterior
s 
√ √ √

2−p
kb + 1 + kc + 1 ≥ k + 1 + 2 kp + 1. Astfel este suficient să arătăm
p
s 
√ √ √

2−p
că k + 1 + 2 kp + 1 ≥ 3 k + 1, (1). Notând kp + 1 = x, avem
p
√ x2 − 1 2k + 1 − x2
   
√  2−p
x∈ k + 1, 2k + 1 , p = ,k +1 = k +1 s, i astfel
s  k p x2 − 1
2k + 1 − x2 √

inegalitatea (1) devine k 2
+ 1 ≥ 3 k + 1 − 2x, (2). Deoarece
x −1
√ √ √ 
3 k + 1 − 2x > 0 pentru orice x ∈ k + 1, 2k + 1 , rezultă că putem ridica
la pătrat ambii membri ı̂n inegalitatea (2) s, i obt, inem inegalitatea echivalentă
√ 2 √
2x2 − 2x k + 1 − k − 4 ≤ 0, (3). Deoarece rădăcinile ecuat, iei

x− k+1
√ p √ p
√ k + 1 − 3 (k + 3) k + 1 + 3 (k + 3)
2x2 −2x k + 1−k−4 = 0 sunt <0< ,
√ p 2 2
k + 1 + 3 (k + 3) √
iar ≥ 2k + 1 pentru orice k ∈ (0, 24], rezultă că avem
√ 2 √
2x2 − 2x k + 1 − k − 4 ≤ 0 pentru orice x ∈
√ 
k√+ 1, 2k + √1 s, i astfel
√inegalita-
tea (3)
√ este adevărată.
√ Cazul 2. √p > 2. Avem ka + 1 + kb + 1 + kc + 1 ≥
1 + 2 kp + 1 > 1 + 2 2k + 1 ≥ 3 k + 1. Demonstrat, ia este ı̂ncheiată. Observăm
că egalitatea are loc pentru (1, 1, 1), dacă 0 < k < 24, respectiv pentru (1, 1, 1)
sau (0, 2, 2) s, i permutările sale, dacă k = 24.

MGO 69. Rezolvat, i ı̂n mult, imea numerelor reale ecuat, ia


cos2 x sin2 x
sin2 x = cos2 x .
Daniel Jinga, Pites, ti
Solut, ie. Fie t = sin2 x ∈ [0, 1], deci cos2 x = 1 − t ∈ [0, 1]. Ecuat, ia dată devine
t1−t = (1 − t)t . Evident, t = 0 s, i t = 1 nu sunt solut, ii, deci t ∈ (0, 1). Prin
ln t ln(1 − t)
logaritmare ecuat, ia devine (1 − t) ln t = t ln(1 − t), adică = . Fie
t 1−t
ln t 1 1
f : (0, 1) → R, f (t) = . Dacă 0 < x < y < 1 atunci > > 0 s, i
t x y
− ln x − ln y
ln x < ln y < 0, deci − ln x > − ln y > 0, prin urmare > , adică
x y
f )x) < f (y). Rezultă că funct, ia f este strict crescătoare, deci injectivă. Astfel
ecuat, ia devine: f (t) = f (1 − t) ⇔ t = 1 − t ⇔ sin2 x = cos2 x ⇔ tg 2 x = 1 ⇔
(2k + 1)π
tg x = ±1 ⇔ x = , cu k ∈ Z.
4
 2
 a = b(2b − c)
MGO 70. Rezolvat, i ı̂n C3 sistemul b2 = c(2c − a) .
 2
c = a(2a − b)
Florin Stănescu, Găes, ti
Rezolvarea problemelor din numărul anterior 83

Solut, ie. Dacă a = 0, atunci din a treia ecuat, ie rezultă c = 0, iar din a doua ecuat, ie
rezultă b = 0, deci obt, inem solut, ia a = b =c = 0. Fie a 6= 0, deci s, i b 6= 0,
 (a − b)(a + b) = b(b − c)
c 6= 0. Sistemul poate fi rescris sub forma (b − c)(b + c) = c(c − a) , (1).
(c − a)(c + a) = a(a − b)

Dacă a = b, atunci din prima ecuat, ie rezultă b = c, deci a = b = c, care este
solut, ie. Aceeas, i solut, ie se obt, ine s, i dacă b = c sau c = a. Fie acum a = 6 b 6= c 6= a,
2 2 2
a, b, c 6= 0. Adunând ecuat, iile sistemului init, ial avem a + b + c = ab + bc + ca.
Această egalitate poate fi rescrisă sub forma (a − b)2 = (b − c)(c − a), deci
|a − b|3 = |a − b| · |b − c| · |c − a|. Analog se obt, ine că |b − c|3 = |a − b| · |b − c| · |c − a|
s, i |c − a|3 = |a − b| · |b − c| · |c − a|, deci |a − b| = |b − c| = |c − a|. Atunci din
(1) rezultă că |a + b| = |b|, |b + c| = |c| s, i |c + a| = |a|. Utilizând identitatea
|a+b|2 +|b+c| 2 2 2 2 2 2
 +|c+a| = |a+b+c| +|a| +|b| +|c| , rezultă că a+b+c = 0. Atunci
 −c(a − b) = b(b − c)
(1) devine −a(b − c) = c(c − a) , deci prin ı̂nmult, ire obt, inem că −abc = abc,
−b(c − a) = a(a − b)

adică abc = 0, fals. În concluzie, sistemul dat are solut, iile (α, α, α), cu α ∈ C.

Clasa a XI-a

1 π
MGO 71. Se consideră un triunghi ABC astfel ı̂ncât arccos √ ≤ A, B, C ≤ .
3 2
1 1 1 12
Demonstrat, i că + + ≥ .
1 + cos2 A 1 + cos2 B 1 + cos2 C 5
Când are loc egalitatea?
Leonard Mihai Giugiuc, România s, i Michael Rozenberg, Israel
1
Solut, ie. Notând 2A = u, 2B = v s, i 2C = w avem π − arccos ≤ u, v, w ≤ π,
3
u + v + w = 2π şi inegalitatea se rescrie ca
1 1 1 6
+ + ≥ .
3 + cos u 3 + cos v 3 + cos w 5
Vom utiliza următorul rezultat ce extinde Inegalitatea lui Jensen:
Teorema funct, iilor semi-convexe (Vasile Cı̂rtoaje, 2004) (Teorema de
la pag. 3 din Mathematical inequalities, Volumul 4: Extensions and refine-
ments of Jensen’s inequality, Editura Universităt, ii Petrol-Gaze din Ploies, ti, 2018;
http://ac.upg-ploiesti.ro/vcirtoaje/MI_VOLUME4.pdf) Fie I ⊆ R un interval, s
un punct interior al lui I s, i f : I → R o funct, ie convexă pe I∩ (−∞, s] sau pe I  ∩
a1 + a2 + . . . + an
[s, +∞). Atunci inegalitatea f (a1 )+f (a2 )+. . .+f (an ) ≥ nf
n
84 Rezolvarea problemelor din numărul anterior

are loc pentru orice a1 , a2 , . . . , an ∈ I a.ı̂. a1 + a2 + . . . + an = ns dacă s, i numai


dacă f (x) + (n − 1)f (y) ≥ nf (s) pentru orice x, y ∈ I a.ı̂. x + (n − 1)y = ns.
 
1 1
Considerăm funct, ia f : π − arccos , π → R, f (x) = . Avem
3 3 + cos x
− cos2 x + 3 cos x + 2

1 2π
f 00 (x) = 3
> 0 ∀x ∈ π − arccos , . Conform teoremei
(3 + cos x) 3 3   
u+v+w 2π
de mai sus, inegalitatea f (u) + f (v) + f (w) ≥ 3f = 3f =
  3 3
6 1
are loc pentru orice u, v, w ∈ π − arccos , π cu u + v + w = 2π dacă s, i
5 3  
6 1
numai dacă 2f (t) + f (2π − 2t) ≥ pentru orice t ∈ π − arccos , π astfel
  5 3
1 2 1 6
ı̂ncât 2π − 2t ∈ π − arccos , π , adică + ≥ , pentru orice
 3  3 + cos t 3 + cos 2t 5
1 π 1 1 1
t ∈ π − arccos , + arccos . Notând cos t = x avem x ≤ − s, i inegalitatea
3 2 2 3 3
2 1 6 2
de arătat devine + ≥ , adică (2x + 1) (3x + 1) ≤ 0, adevărat.
3 + x 2 (x2 + 1) 5
π π π  1
De aici deducem s, i cazurile de egalitate (A, B, C) = , , , pentru x = − ,
 3 3 3 2
1 1 1
respectiv (A, B, C) = arccos √ , arccos √ , arccos s, i permutările, pentru
3 3 3
1
x = − . Ment, ionăm că o altă solut, ie, mult mai laborioasă, bazată pe Inegalitatea
3
lui Karamata, a fost dată de Yury Yucra Limachi, Peru.
MGO 72. Fie matricea A ∈ Mn (C), n ≥ 2, astfel ı̂ncât (A∗ )2 6= On , unde A∗
reprezintă matricea adjunctă a lui A. Să se arate că rang (Ak ) = rang (A), pentru
orice k ∈ N∗ .
Marin Ionescu, Pites, ti
Solut, ie (Alexandru Daniel Pı̂rvuceanu, elev, C.N. ,,Traian”, Drobeta Turnu Severin).
Evident, rang (A) > n − 2, pentru că altfel A∗ = On deci s, i (A∗ )2 = On , fals.
Cazul 1. rang (A) = n, adică det A 6= 0. Avem det(Ak ) = (det A)k 6= 0, deci
rang Ak = n = rang (A). Cazul 2. rang (A) = n − 1, adică det A = 0.
Din Inegalitatea lui Sylvester rang (A · A∗ ) ≥ rang (A) + rang (A∗ ) − n s, i din
A · A∗ = det A · In = On obt, inem rang (A∗ ) ≤ 1. Dar A∗ 6= On , deci rang (A∗ ) =
1. Rezultă că (A∗ )2 = tr A∗ · A∗ , prin urmare avem tr A∗ 6= 0 s, i (A∗ )k =
(tr A∗ )k−1 · A∗ 6= On , pentru orice k ≥ 2. Conform Inegalităt, ii lui Sylvester avem
rang A2 = rang (A · A) ≥ rang (A) + rang (A) − n = n − 2, iar det(A2 ) =  0,
2 2

deci rang A ∈ {n − 2, n − 1}. Presupunând prin absurd că rang A =
2 ∗ = O , deci (A∗ )2 = A2 ∗ = O , fals. Prin urmare
 
n − 2, am avea A n n
rang A2 = n − 1 = rang (A). Presupunem acum că rang Ak = n − 1 s, i
 

demonstrăm că rang Ak+1 = n − 1, unde k ≥ 2 este arbitrar. Procedând


analog ca mai sus, obt, inem rang Ak+1 ∈ {n − 2, n − 1}, iar presupunând prin
Rezolvarea problemelor din numărul anterior 85

absurd că rang Ak+1 ∗ )k+1 = Ak+1 ∗ = O , fals. Prin


 
= n − 2 am avea (A n
k+1 = n − 1. Conform principiului inductiei matematice, obtinem

urmare rang A , ,
rang Ak = n − 1, pentru orice k ≥ 2 s, i astfel rang Ak = n − 1 = rang (A).
 

MGO 73. Fie a, b ∈ R, a < b s, i f : [a, b] → [a, b] o funct, ie derivabilă cu


f (a) = b s, i f (b) = a. Arătat, i că există c1 , c2 ∈ (a, b), c1 6= c2 , astfel ı̂ncât
f 0 (c1 ) + f 0 (c2 ) = −2.
Daniel Jinga, Pites, ti
 
a+b
Solut, ie. Aplicând Teorema lui Lagrange funct, iei f pe intervalele a, s, i
      2
a+b a+b a+b
, b rezultă că există c1 ∈ a, s, i c2 ∈ , b astfel ı̂ncât
2   2   2
a+b a+b
f −b a−f
2 2
f 0 (c1 ) = s, i f 0 (c2 ) = . Astfel avem c1 6= c2 s, i
a+b a+b
−a b−
2 2
0 0 a−b
f (c1 ) + f (c2 ) = = −2.
b−a
2
MGO 74. Fie (xn )n≥1 un s, ir crescător astfel ı̂ncât xn2 = n pentru orice n ≥ 1.
xn
Arătat, i că √ tinde către 1.
n
Cristinel Mortici, Târgovis, te
Solut, ie (Leonard Mihai Giugiuc, Drobeta Turnu Severin). Pentru orice n ∈
√ √ √ √ 2 √ 2
N, n ≥ 1 avem, succesiv: [ n] ≤ n < [ n] + 1; [ √n] ≤ n < ([ √ n] + 1) ;
√ √ [ n] xn [ n] + 1
x √n 2 ≤ xn ≤ x √n +1 2 ; [ n] ≤ xn ≤ [ n] + 1; √ ≤ √ ≤ √ ;
[ ] ([ ] ) n n n
√ √
n−1 xn n+1 xn
√ <√ ≤ √ . Aplicând Criteriul cles, telui rezultă că lim √ = 1.
n n n n→∞ n
MGO 75. Arătat, i că dacă matricele A, B ∈ M2 (C) verifică relat, iile tr (AB) 6= 0
s, i AB 2 A + BA2 B = 2(AB)2 , atunci AB = BA.
Rămâne concluzia adevărată dacă se renunt, ă la ipoteza tr (AB) 6= 0?
Florin Stănescu, Găes, ti
Solut, ie. Conform Teoremei Hamilton-Cayley, pentru orice matrice X ∈ M2 (C)
avem X 2 −tr X ·X +det X ·I2 = O2 . Rezultă că tr X 2 −(tr X)2 +2 det X = 0, prin
(tr X)2 − tr X 2
urmare det X = . Luând X = AB − BA s, i utilizând egalitatea
2
[tr (AB − BA)]2 − tr (AB − BA)2
tr (AB) = tr (BA), avem det (AB − BA) = =
2
2 2 2
tr (AB) − tr (AB A) − tr (BA B) + tr (BA) 2
− = tr (A2 B 2 ) − tr (AB)2 . Dar
2
86 Rezolvarea problemelor din numărul anterior

din a doua relat, ie din enunt, obt, inem 2tr (AB)2 = tr (AB 2 A) + tr (BA2 B), adică
tr (AB)2 = tr (A2 B 2 ). Astfel det(AB − BA) = 0. Conform Teoremei Hamilton-
Cayley pentru X = AB − BA obt, inem (AB − BA)2 = O2 , deci (AB)2 − (AB 2 A +
BA2 B)+(BA)2 = O2 . Cum AB 2 A+BA2 B = 2(AB)2 , rezultă că (AB)2 = (BA)2 ,
de unde utilizând din nou Teorema Hamilton-Cayley avem tr (AB) · (AB) −
det(AB) · I2 = tr (BA) · (BA) − det(BA) · I2 , deci tr (AB)(AB − BA) = O2 . Cum
tr (AB) 6= 0 rezultă că AB = BA.
Dacă se renunt, ă la ipoteza tr (AB)6= 0, egalitatea
 AB
 = BAnu este neapărat
1 0 0 1
adevărată. De exemplu, pentru A = s, i B = avem B 2 = O2 ,
0 0 0 0
AB = B, BA = O2 , deci 2 (AB)2 = AB 2 A+BA2 B = O2 s, i AB 6= BA.

Clasa a XII-a

MGO 76. Determinat, i corpurile finite K cu proprietatea că există x, y ∈ K astfel


ı̂ncât (x + y)−1 = x−1 + y −1 .
Stelian Corneliu Andronescu s, i Costel Bălcău, Pites, ti
Solut, ie. Conform Teoremei lui Wedderbrun, orice corp finit este comutativ. De
asemenea, se cunoas, te că orice corp finit K are pn elemente, unde p este carac-
teristica lui K s, i este un număr prim, iar n este un număr natural nenul. Fie
(x + y)−1 = x−1 + y −1 . Prin ı̂nmult, ire cu xy(x + y) obt, inem xy = (x + y)2 , adică
x2 +xy+y 2 = 0. Cazul 1. x = y. Atunci 3x2 = 0, deci 3 = 0, adică p = 3. Reciproc,
ı̂n orice corp K cu 3n elemente, n ≥ 1, avem 3 = 0, deci (1 + 1)−1 = 1−1 + 1−1 .
Cazul 2. x 6= y. Atunci x3 − y 3 = (x − y)(x2 + xy + y 2 ) = 0, deci x3 = y 3 ,
3
adică xy −1 = 1. Conform Teoremei lui Lagrange rezultă că pn − 1 = M3, deci
p = M3 + 1, n arbitrar sau p = M3 + 2, n par. Reciproc, pentru orice corp K
cu pn elemente cu p număr prim de forma M3 + 1 s, i n ≥ 1 sau p număr prim de
forma M3 + 2 s, i n ≥ 2 număr par, avem pn − 1 = M3, deci conform Teoremei
lui Cauchy există x ∈ K \ {0, 1} astfel ı̂ncât x3 = 1, deci x2 + x + 1 = 0, adică
x = (x + 1)2 s, i prin ı̂nmult, ire cu x−1 (x + 1)−1 obt, inem (x + 1)−1 = x−1 + 1−1 .

MGO 77. Determinat, i valoarea minimă k ∈ N \ {2018} cu proprietatea că există


un polinom f ∈ Z[X] s, i nis, te numere ı̂ntregi m1 , m2 , . . . , m2018 distincte două câte
două astfel ı̂ncât f (mi ) = 2018 pentru orice i ∈ {1, 2, . . . , 2017} s, i f (m2018 ) = k.
Stelian Corneliu Andronescu s, i Costel Bălcău, Pites, ti
Solut, ie. Fie f un polinom cu proprietatea din enunt, s, i fie g(x) = f (x) − 2018.
Atunci g(x) = (x − m1 )(x − m2 ) · . . . · (x − m2017 )h(x), cu h ∈ Z[X]. Deoarece
g(m2018 ) = f (m2018 ) − 2018 = k − 2018, obt, inem

k = 2018 + (m2018 − m1 )(m2018 − m2 ) · . . . · (m2018 − m2017 )h(m2018 ),


Rezolvarea problemelor din numărul anterior 87

toti cei 2018 factori fiind numere ı̂ntregi s, i, cu except, ia lui h(m2018 ), distinct, i doi
câte doi. Astfel valoarea minimă k ∈ N\{2018} ce verifică această egalitate este k =
2018+1·(−1)·2·(−2)·. . .·1008·(−1008)·1009·1 = 1008!·1009!+2018. Ea se atinge, de
exemplu, pentru f (x) = 2018 + (x + 1)(x − 1) · . . . · (x + 1008)(x − 1008)(x + 1009),
m1 = −1, m2 = 1, m3 = −2, m4 = 2, . . . , m2015 = −1008, m2016 = 1008,
m2017 = −1009 s, i m2018 = 0.
Z ln x √
(cos t − 3m sin t) 1 + e4mt
MGO 78. Determinat, i x > 1 s, tiind că dt = 0,
− ln x e3mt
unde m este un număr real fixat.
Marin Chirciu, Pites, ti
Solut, ie. Utilizând metoda integrării prin părt, i, ecuat, ia din enunt, devine 0 =
sin t 0 p ln x
Z ln x  Z ln x
emt sin t

4mt
sin t p 4mt
3mt
1 + e dt = · 1 + e − 2m √ dt.
− ln x e e3mt − ln x − ln x 1 + e4mt
emt sin t
Funct, ia f (t) = √ este impară, deci ultima integrală este egală cu zero.
1 + e4mt
sin(ln x) √ √
4m + x3m sin(ln x) 1 + x−4m = 0;
Ecuat, ia devine, succesiv: · 1 + x
√  x3m
sin(ln x) 1 + x4m 1 + x4m
= 0; sin(ln x) = 0; x = ekπ , k ∈ N∗ .
x3m
MGO 79. Fie α > β > 0 s, i a, b, c, d ≥ 0 astfel ı̂ncât a + b + c + d = 2α + β s, i
a2 + b2 + c2 + d2 = 2α2 + β 2 . Arătat, i că a3 + b3 + c3 + d3 ≥ 2α3 + β 3 .
Leonard Mihai Giugiuc, Drobeta Turnu Severin

Solut, ie. Conform Formulei lui Newton, a3 + b3 + c3 + d3 = (a + b + c + d) a2 + b2 +

c2 +d2 −(ab + bc + cd + da + ac + bd) (a + b + c + d)+3 (abc + abd + acd + bcd),
iar ab + bc + cd + da + ac + bd = α (α + 2β), deci inegalitatea cerută este echivalentă
cu abc + abd + acd + bcd ≥ α2 β. Presupunem prin reducere la absurd că abc + abd +
acd + bcd < α2 β. Fie f : [0, ∞) → R, f (x) = (x − a) (x − b) (x − c) (x − d) =
x4 − (2α + β) x3 + α (α + 2β) x2 − mx + p, unde m = abc + abd + acd + bcd
s, i p = abcd. De asemenea, fie g : [0, ∞) → R, g (x) = x (x − β) (x − α)2 =
x4 −(2α + β) x3 +α (α + 2β) x2 −α2 βx. Observăm că f (x) = g (x)+ α2 β − m x+
p. Pentru orice γ ∈ {a, b, c,d} \ {0} avem 0 = f (γ) = g (γ) + α2 β − m γ +
p, deci g (γ) = − α2 β − m γ − p < 0, de unde rezultă că γ ∈ (0, β). Prin
urmare {a, b, c, d} ⊂ [0, β). Din Teorema lui Rolle deducem că punctele critice
ale lui f sunt situatep ı̂n intervalul [0, β). De asemenea, p punctele critice ale lui
2α + 3β − 4α2 − 4αβ + 9β 2 2α + 3β + 4α2 − 4αβ + 9β 2
g sunt < < α s, i
p 8 p8
2α + 3β − 4α2 − 4αβ + 9β 2 2α + 3β + 4α2 − 4αβ + 9β 2
avem <β < . Fie
8 8 0
 critice ale lui f s, i fie δ ∈ {y1 , y2 , y3 }. Atunci 0 = f (δ) =
y1 , y2 s, i y3 punctele
g 0 (δ) + α2 β − m , deci g 0 (δ) = − α2 β − m < 0, de unde rezultă că avem
88 Rezolvarea problemelor din numărul anterior
p p !
2α + 3β − 4α2 − 4αβ + 9β 2 2α + 3β + 4α2 − 4αβ + 9β 2
δ< sau δ ∈ ,α .
8 8
p
2α + 3β − 4α2 − 4αβ + 9β 2 3 (2α + β)
Dar δ < β, deci δ < . Cum y1 +y2 +y3 = ,
 8 p  4
3 (2α + β) 3 2α + 3β − 4α2 − 4αβ + 9β 2
ar rezulta că < , fals.
4 8
Observăm că egalitatea are loc pentru (α, α, β, 0) s, i permutările sale. Mai mult,
se poate arăta că acestea sunt singurele configurat, ii pentru care abc + abd + acd +
bcd = α2 β.

MGO 80. Fie α, β ∈ R, n ∈ N∗ s, i funct, ia f : (0, ∞) → R, f (x) = α ln x −


Z a 2
xn − 1 x +1
β· n . Calculat, i 1 · f (x) dx, unde a > 0 este fixat.
x +1 x2
a
Daniel Jinga, Pites, ti
1 − xn
 
1 1
Solut, ie. Avem f = α ln − β · = −f (x), pentru orice x > 0. Cu
x x 1 + xn
Z a 2
1 x +1
schimbarea de variabilă x = , integrala din enunt, devine I = 1 ·f (x) dx =
t x2
a
1 1
Z +1 
1
 
1
 Z a 2
t +1

1
 Z a 2
t +1
a t2 ·f · − 2 dt = 1 · f dt = − · f (t) dt =
1 t t t 2 t 1 t2
a
t2 a a
−I, deci I = 0.
Probleme propuse 89

Probleme propuse

Clasa a V-a

MGO 81. În câte moduri se poate scrie 2020 ca sumă de numere impare consecu-
tive?
Costel Anghel, Slatina s, i Florea Badea, Scornices, ti

MGO 82. Arătat, i că numerele 45n s, i 45n + 5n au acelas, i număr de cifre, pentru
orice număr natural n.
Costel Anghel, Slatina s, i Florea Badea, Scornices, ti

MGO 83. Determinat, i ultimele două cifre ale numărului N = 7A , unde


A = p21 + p22 + . . . + p22019 , p1 , p2 , . . . , p2019 fiind numere naturale impare.
Stelian Corneliu Andronescu s, i Costel Bălcău, Pites, ti

MGO 84. Se consideră numerele 1000, 1001, 1002, . . . , 2019.


a) Câte pătrate perfecte se găsesc printre numerele date?
b) Calculat, i suma resturilor ı̂mpărt, irilor numerelor date prin 13.
***

MGO 85. Câte numere de 3 cifre au proprietatea că prin adunare cu răsturnatul
se obt, ine un număr de 4 cifre identice?
Mihai Florea Dumitrescu, Potcoava

Clasa a VI-a

MGO 86. Fie A o mult, ime de numere naturale care satisface condit, iile:
i) 1 ∈ A;
ii) Dacă x ∈ A, atunci 5x ∈ A;
iii) Dacă 7x − 1 ∈ A, atunci x ∈ A.
Arătat, i că 13 ∈ A.
Marin Chirciu, Pites, ti
90 Probleme propuse

MGO 87. a) Determinat, i ultimele cinci cifre ale numărului 72019 , atunci când
este scris ı̂n sistemul binar.
b) Aceeas, i cerint, ă pentru numărul N = 12019 + 22019 + . . . · +20192019 .
Stelian Corneliu Andronescu s, i Costel Bălcău, Pites, ti

MGO 88. Se consideră numerele a, b, c, d ∈ Q∗ astfel ı̂ncât


a 3b 4c 5d
= = = 6= −1.
3b + 4c + 5d a + 4c + 5d a + 3b + 5d a + 3b + 4c
 
1 1 1 1
Calculat, i (a + b + c + d) + + + .
a b c d
Costel Anghel, Slatina s, i Florea Badea, Scornices, ti

MGO 89. Fie ABC un triunghi echilateral s, i punctele D ∈ (BC), P ∈ (AB)


astfel ı̂ncât m (^BAD) = m (^ADP ) = x. Determinat, i valoarea lui x astfel ı̂ncât
perpendiculara din punctul P pe dreapta AD să treacă prin mijlocul laturii AC.
Mihai Florea Dumitrescu, Potcoava

MGO 90. Fie triunghiul ABC cu AB = AC s, i m (^A) = 80◦ . Se consideră


punctul D ı̂n semiplanul determinat de dreapta AB s, i care nu cont, ine punctul C,
astfel ı̂ncât m (^BCD) = 40◦ s, i CD = BC. Calculat, i măsura unghiului BAD.
Costel Anghel, Slatina s, i Florea Badea, Scornices, ti

Clasa a VII-a

MGO 91. Determinat, i perechile (x, y) de numere ı̂ntregi cu proprietatea că


1255 + x2 = 2y .
Costel Anghel, Slatina s, i Florea Badea, Scornices, ti
2 2
MGO r 92. Fie numerele rat, ionale pozitive x s, i y astfel ı̂ncât 2(x − y) + 4y = 3xy
11x + 3y 2x + 3y
s, i ∈ Q. Calculat, i valoarea raportului .
7x + 2y 4x + 5y
Marin Chirciu, Pites, ti

MGO 93. Determinat, i n ∈ N∗ pentru care numărul 1·20 +2·21 +3·22 +. . .+n·2n−1
este pătrat perfect.
Stelian Corneliu Andronescu s, i Costel Bălcău, Pites, ti
Probleme propuse 91

MGO 94. Fie ABCD un patrulater convex, T un punct ı̂n interiorul său, iar
M, N, P s, i Q mijloacele laturilor AB, BC, CD, respectiv DA. Dacă AAM T Q = a
s, i ACP T N = c, calculat, i aria patrulaterului ABCD ı̂n funct, ie de a s, i c.
Florea Badea, Scornices, ti
MGO 95. Fie triunghiul ABC cu AB = AC s, i m (^A) = 100◦ . Se consideră
punctul D ı̂n semiplanul determinat de dreapta AB s, i care nu cont, ine punctul C,
astfel ı̂ncât m (^BCD) = 20◦ s, i BD = BC. Calculat, i măsura unghiului ADC.
Costel Anghel, Slatina

Clasa a VIII-a

MGO 96. Fie a, b, c ∈ Q+ astfel ı̂ncât ab + bc + ca = a − b + 2. Demonstrat, i că


p
(a2 + 2a + 2)(b2 − 2b + 2)(c2 + 1) ∈ Q.
Costel Anghel, Slatina s, i Florea Badea, Scornices, ti
r
a2 + b2 + c2
MGO 97. Fie a, b, c > 0 cu a + b + c = 1. Arătat, i că + 18abc ≤ 1.
3
Marin Ionescu, Pites, ti
MGO 98. Arătat, i că pentru orice a, b, c > 0 are loc inegalitatea
a(a − b)(2b + c) b(b − c)(2c + a) c(c − a)(2a + b)
+ + ≥ 0.
b + 2c c + 2a a + 2b
Ardak Mirzakhmedov, Kazahstan
MGO 99. Un trunchi de piramidă regulată este sect, ionat cu un plan paralel
cu bazele astfel ı̂ncât volumele celor două trunchiuri obt, inute să fie egale. Dacă
notăm cu L s, i l lungimile laturilor bazelor trunchiului dat s, i cu m lungimea laturii
poligonului de sect, iune, arătat, i că volumul cubului de muchie m este egal cu media
aritmetică a volumelor cuburilor de muchii L s, i l.
***
MGO 100. Fie a > 0 un număr real fixat. Determinat, i valorile x ∈ R pentru care
există două piramide V ABCD s, i V A0 B 0 C 0 D0 astfel ı̂ncât ABCD este un pătrat,
A0 B 0 C 0 D0 este un romb, AB = A0 B 0 = a, m (^B 0 A0 D0 ) = 60◦ , triunghiurile V AB
s, i V A0 B 0 sunt echilaterale, iar perimetrele triunghiurilor V CD s, i V C 0 D0 sunt egale
fiecare cu x.
Stelian Corneliu Andronescu s, i Costel Bălcău, Pites, ti
92 Probleme propuse

Clasa a IX-a

MGO 101. Pentru orice numere reale pozitive a1 , a2 , . . . , an s, i orice număr natural
k mai mare ca 1 are loc inegalitatea

(ak1 + n − 1)(ak2 + n − 1) · . . . · (akn + n − 1) ≥ nn−k (a1 + a2 + . . . + an )k .


Ardak Mirzakhmedov, Kazahstan

MGO 102. Să se determine valoarea minimă a expresiei

2(x3 + y 3 + z 3 ) + 3xyz
E(x, y, z) =
x2 y + xy 2 + y 2 z + yz 2 + z 2 x + zx2
când x, y, z ∈ (0, ∞).
Dan Nedeianu, Drobeta Turnu Severin s, i Sorin Ulmeanu, Pites, ti

MGO 103. Rezolvat, i ı̂n R × R ecuat, ia

(sin3 x + 2)(cos3 y − 3 sin2 y − 3) + (cos y − 3)(2 cos2 x − sin x − 2) = 0.


Mihai Florea Dumitrescu, Potcoava

MGO 104. Fie ABC un triunghi neechilateral ı̂nscris ı̂ntr-un cerc de centru O
s, i circumscris unui cerc de centru I. Notăm cu M , N s, i P centrele cercurilor
exı̂nscrise triunghiului ABC. Să se arate că centrul de greutate al triunghiului
M N P se află pe dreapta OI.
Marin Ionescu, Pites, ti
Y ra X ra
MGO 105. Arătat, i că ı̂n orice triunghi ABC avem 4 · − = 1.
ha ha
Daniel Jinga, Pites, ti

Clasa a X-a

MGO 106. Fie x, y, z ∈ (0, ∞) astfel ı̂ncât x + y + z = xyz. Demonstrat, i că



x y z 3 3
+ + ≤ .
1 + x2 1 + y 2 1 + z 2 4

Când are loc egalitatea?


Alexandru Daniel Pı̂rvuceanu, elev, Drobeta Turnu Severin
Probleme propuse 93

MGO 107. Fie a > 0, a 6= 1. Rezolvat, i ı̂n mult, imea numerelor reale ecuat, ia

loga (x − 2a − 1) = log2a+1 (2x + 2a2 − 1).


Marin Chirciu, Pites, ti

MGO 108. Fie numerele reale a, b s, i c astfel ı̂ncât

th (a)th (b) + th (b)th (c) + th (c)th (a) = 1

ex − e−x
(unde th (x) = reprezintă tangenta hiperbolică a numărului real x).
ex + e−x
a) Demonstrat, i că a, b, c < 0 sau a, b, c > 0.

1 2 + th (a) + th (b) + th (c) + th (a)th (b)th (c)
b) Arătat, i că a + b + c = ln .
2 2 − th (a) + th (b) + th (c) + th (a)th (b)th (c)
Michel Bataille, Frant, a s, i Leonard Mihai Giugiuc, România

MGO 109. Fie z1 , z2 , z3 ∈ C∗ astfel ı̂ncât |z1 +z2 | = |z1 |+|z2 |, |z1 +z3 | = |z1 |+|z3 |
s, i z2 + z3 = 2z1 . Determinat, i mult, imea tuturor valorilor posibile pentru numărul
z 2 · z2 + z3 · z3
.
z1 · z 1
Mihai Florea Dumitrescu, Potcoava

MGO 110. Rezolvat, i ı̂n mult, imea numerelor reale ecuat, ia

cos2 x + cos2 2x · sin2 3x + sin2 4x = 1.


Daniel Jinga, Pites, ti

Clasa a XI-a

MGO 111. Fie A ∈ Mn (C) astfel ı̂ncât A6 = A + In . Demonstrat, i că matricea


A2 + A + In este inversabilă.
Cristinel Mortici, Viforâta

MGO 112. Fie a, b ∈ C∗ astfel ı̂ncât a 6= ±b s, i A, B ∈ M4 (C) astfel ı̂ncât


det(aAB + bBA) = det(aBA + bAB). Demonstrat, i că
 
det (x + a)AB + (b − x)BA = det (x + a)BA + (b − x)AB , ∀x ∈ C.
Daniel Jinga, Pites, ti
94 Probleme propuse

MGO 113. Se consideră matricea A ∈ Mn (C), n ≥ 2, astfel ı̂ncât det A = 0 s, i


det(In + A∗ ) = 1, unde A∗ este matricea adjunctă a lui A. Arătat, i că (A∗ )2 = On .
Marin Ionescu, Pites, ti
MGO 114. Fie (an )n≥1 o progresie aritmetică având primul termen a1 = 1 s, i
1 1 1
a a
an1 + an2 + . . . + anan
rat, ia r ∈ N∗ . Calculat, i lim .
n→∞ an
Dan S, tefan Marinescu, Hunedoara s, i Leonard Mihai Giugiuc, Drobeta Turnu Severin
MGO 115. a) Fie a, b, c, d ≥ 0 cu a2 + b2 + c2 + d2 = 2. Demonstrat, i că
a + b + c + d − (abc + abd + acd + bcd) ≤ 2.
b) Arătat, i că pentru orice k < 1 există a, b, c, d ≥ 0 cu a2 + b2 + c2 + d2 = 2 astfel
ı̂ncât a + b + c + d − k(abc + abd + acd + bcd) > 2.
Leonard Mihai Giugiuc, Drobeta Turnu Severin

Clasa a XII-a

MGO 116. Fie a, b ∈ Z∗23 . Calculat, i probabilitatea ca alegând un endomorfism f


al grupului (Z∗23 , ·) acesta să verifice relat, ia f (a) = b.
Stelian Corneliu Andronescu s, i Costel Bălcău, Pites, ti
MGO 117. Fie p un număr prim de forma 4k + 1. Demonstrat, i că polinomul
f = X p−1 + aX + b
1 ∈ Zp [X] este reductibil, pentru orice a ∈ Zp .
Rămâne afirmat, ia adevărată pentru numerele prime p de forma 4k+3?
***

 a+b+c+d=4
MGO 118. Rezolvat, i ı̂n R4 sistemul a2 + b2 + c2 + d2 = 52 .
 4
a + b4 + c4 + d4 = 868
Florentin Vis, escu, Bucures, ti s, i Leonard Mihai Giugiuc, Drobeta Turnu Severin
(sin x − x cos x) sinn−2 x + xn−2
Z   π
MGO 119. Calculat, i dx, x ∈ 0, , unde
(sin x + x)n 2
n este un număr natural fixat.
Daniel Jinga, Pites, ti
MGO 120. Fie f : [0, 1] → R o funct, ie de două ori derivabilă, cu derivata a doua
continuă, astfel ı̂ncât f (0) = 0, f (1) = (a − 1)2 , f 0 (0) = a2 s, i f 0 (1) = a2 − 4a + 3,
Z 1
2
unde a ∈ R. Demonstrat, i că f 00 (x) dx ≥ 4(4a2 − 6a + 3).
0
Marin Chirciu, Pites, ti
Revistă sponsorizată de SC VISTORIA LUX SRL

S-ar putea să vă placă și